Download as doc, pdf, or txt
Download as doc, pdf, or txt
You are on page 1of 188

නයිනර්ස් : අහින්ෂක භාලාපෝරො

Final exam GENETICS SET 1-1


࿿93࿿h In which stage of the meiotic division does crossing over take place
I prophase
II prophase
II anaphase
I anaphase
࿿94࿿h During telophase all of the above mentioned takes place EXCEPT
← Nuclear envelope forms
← the chromosome becomes less dense and more thread like
← DNA is duplicated
← The chromosomes are released from the microtubules
࿿95࿿h If at the end of meiosis, the daughter cell has 12 chromosomes, how many
chromosomes were in the mother cell?
← 3
← 6
← 12
← 24
࿿96࿿h After which of the following divisions will there be 23 chromosomes and 46
chromatids in the cell?
← Mitosis
← I meiotic division
← II meiotic division
← Amitosis
࿿97࿿h An important and distinct feature of meiosis is
← Recombination
← Chromosome condensation
← Chromosome congression
← Spindle formation
࿿98࿿h During the process of oogenesis
← Four ovum are produced
← Twi polar bodies are produced
← Two ovum and two polar bodies are produced
← Only one ovum is produced.
࿿99࿿h The human haploid genome is composed of approximately
← 6 billion pairs of DNA and 2000 protein coding genes
← 6 million pairs of DNA and 2000 protein coding genes
← 3 million pairs of DNA and 2000 protein coding genes
← 3 billion pairs of DNA and 20000 protein coding genes
࿿100࿿h Which of the following units correspond to the I thousand nucleotide base pairs in the
genomic DNA?
← I microbase
← I milibase
← I kilobase
← I megabase
a By which of the following bonds are DNA nucleotides og the same strand connected with
each other?
a Hydrogen bond
b Sugar phosphate bonds
c Hydrophobic bonds
d Peptide bonds
b What is the complementary strand for the DNA of the following RNA
strand 5’AGCCAUUGCACG 3’ ?
a 3’ CGUGCUUUGGCU 5’
b 3’ ATGCGTTTACCGA 3’
c 5’ CGUGCAAUGGCU 3’
d 3’ TCGGTTTCGTGC 5’
c DNA exist in a double-stranded form whereas RNA is mainly a single stranded molecule. What
is the likely reason for DNA being double stranded?
a DNA cannot exist in single stranded form
b RNA strands cannot form base pairs
c Double stranded DNA is more stable structure
d It is easier to replicate double stranded DNA than single stranded RNA
d Which of the following statements is true regarding Okazaki fragments?
a They are synthesized by ligase
b They are short fragments of DNA joined by DNA ligase
c They add nucleotides to the elongating DNA
d They are short fragments of RNA joined by DNA ligase
e When examining the genetic code, it is apparent that
a There can be more than one amino acid for a particular codon
b AUG is a terminating codon
c There can be more than one codon for a particular amino acid
d The code is ambiguous in that the same codon can code for two or more amino acids
f Which of the following makes use of RNA template to synthesize DNA?
a DNA polymerase
b RNA polymerase
c Reverse transcriptase
d DNA dependent RNA polymerase
g Which statement about base substitutions is true
a Can not affect splicing
b Are always pathogenic
c Can affect gene expression
d Result in frameshift mutations.
h During protein synthesis an anticodon on tRNA pairs with
a DNA nucleotide bases
b rRNA nucleotide bases
c mRNA nucleotide bases
d other tRNA nucleotide bases
i Which statement about transcription is true
a Describes the production of polypeptides from mRNA template
a Produces single-stranded mRNA using the sense DNA strand as template
b Is regulated by transcription factors that bind to the 3’UTR
c Precedes 5’ capping and polyadenylation
a Which of the following processes take part in the nucleus of a cell?
a RNA processing
b Posttranslational processing
c Translation
d Binding amino acids with transport RNA
b Which of the following unprocessed RNA transcript of gene collinear with the genomic DNA
and contains introns as well as exons?
a tRNA
b rRNA
c mature RNA
d primary RNA
c What effect can methylation of CpG islands in the promoter region have on gene expression?
a Transcription level will be reduced
b Transcription level will sharply increase
c It will affect translation
d It will affect plicing process
d During somatic rearrangement which part of the DNA is excised?
a Exons
b Introns
c Gene segments
d Promotor sequence
e Which of the following is NOT an outcome of posttranslational modifications of a protein
a Removal of N terminus amino acid
b Addition of metals to create tertiary or quaternary structures
c Addition of phosphate groups
d Removal of introns
f Which of the following types of histones does NOT constitute an octamer?
a H1
b H2A and H2B
c H3
d H4
g Which of the following is the fundamental unit of chromatin organization?
a Solenoid
b Scaffold
c Matrix
d Histone
h Which of the following contain protein encoding genes?
a Heterochromatin
b Euchromatin
c Telomeric regions
d Centromeric regions
← Approximately 5% of the cytosine residues are methylated in the genome of any given
eukaryote. In what ways is DNA methylation related to genetic regulation?
← There is an inverse relationship between the degree of methylation of DNA and the
degree of gene expression
← Methylation of DNA promotes upregulation of gene expression
← To the methylated regions of DNA specific transcription factor proteins attached
← Active chromatin configuration is characterized by methylation od CpG
← Which of the following elements are present in the processed pseudogenes?
← Anticodons
← Introns
← Exons
← Enhancers
← Which is NOTtrue regarding variable number tandem repeats (VNTRs)?
← They are about 15 to 100 base pairs long
← They are transposable elements
← They are found within and between genes
← They are useful for identification of individuals
← Which of the following does NOT regulate normal gene expression?
← Enhancer
← Silencer
← Promoter
← “Cap”
← Transposable sequences fall into a unique group of repetitive DNA sequences that are
identified by their ability to
← Stop viruses from infecting a cell;
← Move from place to place within the genome
← Interfere with telomere function
← Prevent proper chromosome segregation during meiosis
← What are two modular elements that appear as consensus sequences upstream from
RNA polymerase II transcription start sites
← Microsatellites and transposons
← rDNA and nucleolar organizers
← TATA and CAAT
← TTAA and CCTT
← The mechanism of silencing of genes through epigenetics occurs
← Specifically at (C) in RNA located in CpG islands
← Specifically on amino acids on histone tails
← Specifically at cytosine (C) in DNA located in areas where strings of C are located
← Specifically at cytosine (C) in DNA located in CpG islands
← A mutation is defined as:
← A change in an organism’s DNA sequence
← The growth of an abnormal cell structure
← The changing of a cell from one type to another
← A way of changing mRNA to proteins
← Why are male mutation rates higher than female
← Greater number of germ cell divisions
← More cells so more DNA
← The Y chromosome is unstable
← Men have only 1 X chromosome
← Which of the following would result in the highest rate of mutation?
← Small genes, few hot spots, old age
← Large genes, few hot spots, old age
← Small genes, many hot spots, young age
← Large genes, many hot spots, old age
← Which of the following statements regarding spontaneous mutation rates is FALSE?
← The spontaneous mutation rate cannot be measured in complex animals such as
mammals
← The spontaneous mutation rate varies from gene to gene within an organism
← The spontaneous mutation rate is very high in some organisms, but they have
efficient repair system
← The spontaneous mutation rate varies considerably among different organisms
← Which of the following illustrates a single point mutation on a segment of DNA which reads
TACTCCGGTG?
← TACACGCGCTG
← TACAUGTG
← TAGACGGTC
← TTCACGGAG
← A frameshift mutation is one of the most sever types of mutations because
← Mpre than one gene is affected
← They occur only in gametes
← Translation is stopped
← More than one amino acid or entire proteins are affected
← What is the consequence of a nonsense mutation?
← Prevents DNA replication
← Codes for the wrong amino acid
← Deletes promoter region
← Introduces a stop codon
← When the number of bases involved is not a multiple of three (i.e., is not an integral number of
codons), and when it occurs in a coding sequence, the reading frame is altered beginning at
the point of the insertion or deletion:
← Nonsense mutations
← Silence mutations
← Frameshift mutation
← Missense mutations
← The mutation, causing sickle-cell disease is GAG GTG in the beta globin gene. This is an
example of
← Frameshift mutation
← Missense mutation
← Nonsense mutation
← Dynamic mutation
← What is the name of the sequence that is most common in a population?
← Private sequence
← Variant sequence
← Reference sequence
← All of the above
← Which of the following is NOTtrue about indels?
← The number of repeats correspond to the number of alleles
← Their DNA sequences may vary from 2bp to 200bp
← There are hundreds to thousands of indels in the genome
← Indels are never inherited
← In a case when two related individuals in a family have the same disease, they are said to be
← Concordant
← Discordant
← Quantitative traits
← Qualitative traits
← Which of the following biomolecule has self-repair mechanisms?
← DNA, RNA and protein
← DNA and RNA
← DNA only
← DNA and proteins
← A polymorphism is
← Any change in the DNA sequence
← The most common variation of a gene or marker sequence
← The least common variation of a gene or marker sequence
← A variation of gene or marker sequence present in >1% of the population
← Which of the following is directly involved in DNA repair?
← Topoisomerase
← Ribosomes
← DNA polymerases
← Telomereases
← What type of enzyme removes damaged DNA from the rest of the DNA molecule?
← Polymerase
← Nuclease
← Ligase
← Primase
← Which polymorphism is represented with a multiple alleles and used for identity testing
← STRs
← SNPs
← LINE-1
← CNVs
← An individual’s ABO blood type is normally determined by:
← Genetic inheritance and environmental influences during life
← Environmental influences alone
← The inheritance of 1 of 3 possible alleles (A,B or O) from each parent
← Whether mother has been injected by RhIG during pregnancy or not

← An Rh “-” female gave birth to an Rh “+” child. She was not injected Rh immune globulin after the
delivery. Her husband is Rh “+”. Will she have problems with her second pregnancy? (remember, Rh “-“
is a negative trait)
← Yes. She will definitely have problems

← Galactosemia is an autosomal recessive disorder. Symptoms include Failure to thrive vomiting


jaundice hepatomegaly and cataracts. it is most commonly due to A deficiency of galactose-phosphate
uridyltransferase (GALT). A man and woman, who are both known to be carriers for galactosemia,
marry and have a child. To their relief the standard newborn screening test results which measures the
blood level of galactose and galactose-1-p, shows that their child is notaffected. Based upon this
information, what is the probability that the child is normal phenotypically?

← 25%

← 50%

←75%
← Tay-sachs disease is inherited as an autosomal recessive inheritance,characterises deficiency of the
enzyme hexoseaminidase A. Symptoms include blindness and retardation. Onset of symptoms begins at
about six months of age and death results in early childhood. What’s the probability that a homozygous
normal manand a carrier femalewill have a child with Tay-Sachs disease?
← 0%

← 25%

← 50%

d.75%
B. Hemophilia is a disease in which patients lack a clotting factor in the blood (factor VIII). It is
inherited as anX-linked recessive trait.What’s the probability of a hemophilic male and a carrier
femaleproducing ahemophilic son?
← 0%

←25%

← 50%

d.75%

A Red-green colour blindness also known as daltonism is a relatively benign condition. The gene
responsible for this condition is located on theX chromosome and is inherited as a recessive trait.
Approximately 7-10% men and 0.5-1% of women are affected. What’s the chance of a Daltonic father
and a normal mother will produce a colour-blind son?
← 0%

← 25%

← 50%

d.100%
← Robin is affected by an autosomal dominantdisorder inherited from her mother. She is married to
Chad who is unaffected and has no history of the disorder in his family. Robin and Chad have two
unaffected children. Studies suggest that for every 100 individuals who inherit mutations in the gene
of interest, only 50 actually show symptoms. The new mutation rate for this disorder is essentially
zero. Based on this what is the probability that their next child will present with the clinical signsof the
disease?
← 3/4

← 1/2

← 1/4

← 1/8

← Which of the following symptoms best describes a disease caused by one or two mutant alleles?

← monogenic

a which of the following definitions best corresponds to an individual’s genotype, which is expressed in
its morphological characteristic?
c phenotype

b ethnic differences in disease frequencies is more apparent for:

c. Autosomal dominant conditions


d. Autosomal recessive conditions
e. X-linked recessive conditions
f. Sex chromosome aneuploidies

c the presence in an individual or a tissue of at least 2 cells lines that differ genetically but are
derived froma single xygote is referred to as:
A Mosiachism
B Trinucleotide repeat expansions
C Imprinting
D Mitochondrial inheritance

A. the frequency of expression of a phenotype is less than 100%- that is, when some of those who
have the appropriate genotype completely fail to express it- the gene is said to show:
c. reduced penetrance

A. Nearly every individual infected with neurofibromatosis type 1 (NF1) exhibits clinical symptoms.
Some, however may present with café au lait spots and Lisch nodules, while others have life
threatening tumors surrounding the spinal cord. These represent an example of which of the following?
← variable expressivity
← reduced penetrance
← locus heterogeneity
← sex-influenced expression

B. which of the following definitions best corresponds to the disease that is caused by different types
of mutations in the same locus?
← allelic heterogeneity

C. In 1991, it was discovered that fragile X syndrome was caused by a mutation in the fragile X mental
retardation 1 gene (FMR1) gene. An area of CGG trinucleotide repeats just upstream of the coding area
was found to be variable in size. All the following regarding the FMR1 gene are true except:
A. "Premutations" may expand to full mutations in future generations

B. Offspring of male carriers inherit a permutation

C. Offspring of female carriers may inherit a permutation or a full mutation

← Individuals with premutation will also have intellectual disability,

A. which of the following is true about incomplete dominance?

a. the hybrid shows a phenotype that is intermediate between the two homozygotes

← the expression of the disease phenotype depends on whether the mutant allele has been inherited
from the father or from the mother, a phenomenon known as:
← x linked recessive inheritance

← autosomal recessive

c.
d. genomic imprinting
← which of the following diseases are not characterized by anticipation?

← Alzheimer’s disease

← which of the following is characteristic to the mitochondrial type of inheritance?

A the disease is transmitted only maternally


A. the disease appears to develop at an earlier age when it is transmitted, a phenomenon referred to as

A anticipation

B. which of the following statements is true about polygenic trait?

← Several different genes influence one trait


← One gene influence many traits
← Several genes influence many traits
← Several alleles exist for one trait

C. which of the following is true about inheritance of complex diseases?

← the disease is more common in close relatives of the proband than in far relatives

← which of the following is true about conditions that show a multifactorial inheritance pattern?

b.

c. The recurrence risk is higher if the if the proband is of the less commonly affected sex

d.

← which of the following is true of a concordant trait between twins?

← it’s more common in males to have the trait


← it’s more common for females to have the trait
← it’s more common for the older twin to have the trait
← both share the trait

← which of the following is true about the modifier gene?

← it’s a nonallelic gene that changes the manifestation of another gene

← which of the following is true about cleft lip/palate(CL/P)?

← syndromic is always inherited in a single-gene disorder

← it causes mental retardation

← Most of the patients with cleft lip/palate are males

← in isolated cases of CL/P the recurrence risk in relatives does not always correlate
with the proband’s severity
← an 80-year-old man presented with impairment of higher intellectual function and alterations in
mood and behaviour. His family reported progressive disorientation and memory loss over the last 6
months. There is no family history of dementia. The patient was tentatively diagnosed with Alzheimer
disease. which one of the following best describes Alzheimer disease?
← it’s associated with the accumulation of amyloid precursor protein

← it’s associated with the deposition of neurotoxic amyloid β peptide aggregates


← which statement s true regarding Alzheimer disease patients?

c. ε4 allele icreases the risk of developing Alzheimer disease but it does not cause the disease

← which of the following may cause cerebral vein thrombosis?

← any of the above

← which of the following statements regarding venous thrombosis is true?

← Factor V leiden and the prothrombin G20210A variant are common cause of venous
thrombosis
d. the factor V Leiden mutation causes reduced expression of factor V gene

← which process is primarily responsible for aneuploidy?

← meiotic non disjunction

← which of the following will be most likely to produce an imbalance in the amount of essential
genetic material for the carrier?
← Robertsonian translocation
← Pericentric inversion
← Reciprocal translocation
← Isochromosome

← this type of acrocentric chromosome has __________ attached to their short arms by stalks

← Satellites
← Fragile sites
← LINE sequences
← Alu sequences

← to be stable, a rearranged chromosome must have which of the following?

← No inverted segments
← Centromere and two telomeres
← Centromere
← Balanced composition(no losses or gains of genetic material)

← in which phase of mitosis can high quality G or R banding of the chromosomes be accomplished?

← metaphase

← which of the following chromosomes correspond to dicentric chromosomes

← Duplicated
← Homological
← Abnormal
← Diploid
← which of the following is a chromosomal mutation?

← aneuploidy

← short arms of acrocentric chromosomes contains

← tRNA genes
← rRNA genes
← telomerace genes
← none of the above

← which of the following is characterized by the junction of two acrocentric chromosomes near
the centromere with the deletion of short arms?
← Robertsonian translocation

← you have sent a blood sample of a dismorphic baby to the laboratory for chromosomal analysis.
The answer from the ab says that the baby 46,XY karyotype del(18)(q12). Whats the reason for the
babay’s parents to also do a blood analysis?
← to determine whether anomaly is acquired or inherited

← which of the following does not cause Down’s syndrome?

← Reciprocal translocation between 21q and chromosome 15

← Fluoresence insitu hybridization using the whole chromosome (painting) or specific locus
probes enables routine detection of:
← trisomy

← which method is used to detect relative copy number gains and losses in a genome wide manner
by hybridization?
← CGH array

← where are pseudoautosomal regions (PAR) located on?

← PAR1 and PAR2 are located on distal parts of both arms of X and Y chromosomes
← PAR1 and PAR2 are located close to the centromeres of both X and Y chromosomes
← PAR1 is located at the distal part and PAR2 is located near the centromere of both X and
Y chromosomes
← PAR1 is located near the centromere and PAR2 is located at the distal part of both X and
Y chromosomes
← which of the following is true about X inactivation?

← embryos with 2 active X cannot survive

← which of the following statements is correct?

← X chromosome inactivation all genes of one X are switched off


← which of the following statements is correct?

← X chromosome inactivation all genes of one X are switched off

← when is a Barr body formed

← early embryogenesis

← dosage compensation in mammals typically involves the random inactivation of one of the two
X chromosomes relatively early in development. In a such X chromosome involve:
← XIST gene

← which is not a feature of Kleinfelter syndrome?

← Short height
← Gynecomastia
← Long, thin fingers arms and legs
← Small testes

← marcus and Carmen have a child with ambiguous external genetalia. The internal genitalia are
female and chromosome analysis is 46,XX. Biochemical studies reveal a deficiency of 21-hydroxylase.
What is the most likely diagnosis?
← congenital adrenal hyperplasia
1. During normal adult life the major form of hemoglobin produced in red blood cells:
a. α2 2
b. α2β2
c. α2ε2
d. α2γ2
← Which of the following statements is FALSE?
← The hemoglobinopathies are the most common single-gene disorders in humans
← More than 5% of world’s population are carriers of genes responsible for disorders of Hb
← There are 2 α globulin and 2 β globulin genes per diploid genome
← The tertiary structure of Hb has been highly conserved during evolution
← Which statement about α chains is true?
← The genes for α and α-like chains are clustered tandemly on chromosome 16
← The genes for α chains are on chromosome 11 and the genes for α-like chains
on chromosome 16
← The genes for α like chains are clustered tandemly on chromosome 11
← The genes or α chains are on chromosome 16 and the genes for α-like chains
on chromosome 24
← Which statement about β chains is true?
← The genes for β and β-like chains are clustered tandemly on chromosome 16
← The genes for β chains are on chromosome 16 and the genes for β-like chains
on chromosome 11
← The genes for β and β-like chains are clustered tandemly on chromosome 11.
← The genes for β chains are on chromosome 16 and the genes for β-like chains
on chromosome 24.
← The majority of iron present in the body is present as
← Ferritin
← Myoglobin
← Hemoglobin
← Heme containing enzymes
← Where is the major site of gobin synthesis during 3-8 weeks of gestation?
← Liver
← Yolk Sac
← Spleen
← All of the above
← Where is the major site of hematopoesis in adult life?
← Liver
← Spleen
← Bone Marrow
← All of the above
← Which of the following is the most common type of normal human Hb:
← Hb A
← Hb F
← Hb H
← Hb S
← Which of the following is false concerning the hemoglobin molecule?
a. It contains amino acids
b. It contains iron
c. It can bind O2 molecules
d. It is found in humans only
10. The tertiary structure of protein is the
a. Bonding together of several polypeptide chains by weak bonds
b. Order in which amino acids are joined in a polypeptide chain
c. Unique three-dimensional shape of the fully folded polypeptide
d. Organization of a polypeptide chain into a α helix or β pleated sheet
11. Which of the following is FALSE about fetal Hb:
a. Hb F accounts for about 70% of total haemoglobin at birth
b. Hb F accounts for about 20% of total haemoglobin in adult life
c. Switching of γ-globin gene to β-globin gene takes place after the birth
d. Hb F is the predominant haemoglobin throughout fetal life
12. Wh ich of the following corresponds to Hb F:
a. 2 γ2
b. α2 γ2
c. γ2 β2
d. ε2 γ2
← In the condition hereditary persistence of foetal haemoglobin there is:
← Increased production of α chains
← Decreased production of γ chains
← Increased production of β chains
← Increased production of γ chains
← Which of the following statement is correct:
← Persistence of fetal Hb into adult life is an acquired disorder;
← Throughout fetal life it is the liver that produces most of the body’s Hb
← Persistence of fetal Hb into adult life is a benign condition
← After the birth spleen is the primary place of hematopoiesis
← Which change in the polypeptide chain is characteristic to Hb S:
← β chain:Glu6Val
← β chain:Glu6Lys
← β chain:His92Tyr
← β chain:Glu2Lys
← Which of the following statement is correct:
← Many Hb variants are harmless
← The types of mutation occurring in the hemoglobinopathies are very limited
← The tertiary structure of Hb has undergone many changes during evolution
← Point (missense) mutations are the usual cause of abnormal Hb in the sickling disorders
← Which statement about sickle-cell disease is true:
← The sickling effect of red blood cells is the result of abnormal Hb binding with the
red blood cell membrane;
← Life-threatening thrombosis can occur;
← Splenic infarction may occur but this has little clinical consequence
← Deletions are the usual cause of abnormal Hb in the sickling disorders
← Which statement about sickle cell disease is true
← The sickling effect of red blood corpuscles is the result of abnormal Hb binding with
the red blood cell membrane
← Hb S differs from normal Hb A by a single gene amino-acid substitution
← Splenic infarction may occur but this has little clinical consequence
← Point (missense) mutations are the usual cause of abnormal Hb in the sickling disorders
← The worldwide distribution of sickle cell anemia and β-thalassemia coincides with that of:
← Influenza
← Cholera
← Multiple sclerosis
← Malaria
← Sickle cell anemia is caused by a change in the amino acid sequence of the two beta chains
in the haemoglobin molecule. Hoe many amino acids have been changed in each beta chain,
compared to normal hemoglobin?
← 1
← 19
← Hundreds
← Thousands
← What causes the anemia in sickle cell disease?
← An inability of the red cell to reduce organic peroxides
← An abnormal haemoglobin which polymerizes and irreversibly injures the red cell
← Inability of red blood cells to transport oxygen
← Insufficient Hb A and excess unpaired α chains
← Which change in the polypeptide chain is characteristic to Hb C
← β chain:Glu6Val
← β chain:Glu6Lys
← β chain:His92Tyr
← β chain:Glu22Lys
← Which of the following is characteristic to Hb C;
← Heinz bodies damage the red blood cell membrane
← It is less soluble than Hb A and tends to crystallize in red blood cells
← Mutation allows heme to drop out of its pocket
← They aggregate, block blood flow and cause local ischemia
← Persons with Hb SC disease are referred to as
← Compound heterozygotes
← True homozygotes
← Heterozygotes
← Hemizygous
← Patients with Hb SC disease:
← Frequently die in utero from complications of the hemoglobinopathy
← Have a different mutation in each copy of their β-globin gene, though both are in the
same codon
← Never experience sickle cell crises
← Could not have a child with sickle cell disease
← Which of the following about Hb Hammersmith is FALSE:
← It is an unstable Hb that causes denaturation of haemoglobin tetramer
← The polypeptide chain mutation is β chain:Phe22Ser
← It tends to crystallize in red blood cells
← Mutation allows heme to drop out of its pocket
← Which of the following about Hb Kempsey is false?
← It has high oxygen affinity
← It has low oxygen affinity
← Henz bodies are formed that damage the red blood cell membrane
← Mutations prevent oxygen-related movement between the chains and it can not
give oxygen to tissues
← Which of the following chromosomal pathology is caused by the duplication of part of
the chromosome?
← Partial trisomy
← Partial monosomy
← X chromosome trisomy
← 12thchromosome trisomy
← In some chromosomes, the centromere is not in the middle of the chromosome, however there
is no significant difference in the length of the arms. Which of the following corresponds to this
type of chromosome?
← Metacentric
← Submetacentric
← Acrocentric
← Telocentric
← In some chromosomes the centromere is located on one end of the chromosome and such
chromosomes have only one arm. Which of the following corresponds to this type of
chromosome?
← Metacentric
← Submetacentric
← Acrocentric
← Telocentric
← Which of the following chromosomal trisomies are compatible with life?
← 18 chromosome trisomy
← 17 chromosome trisomy
← 19 chromosome trisomy
← 20 chromosome trisomy
← In which of the following organisms telocentric chromosomes are not seen in normal condition
← Humans
← Flies
← Birds
← Snakes
← Which structural anomaly is followed by rearrangement results from breakage
of nonhomological chromosome exchange of broken-off segments?
← Inversions
← Duplications
← Reciprocal translocations
← Deletions
← Which of the following genome anomalies are most common?
← Polyploidy
← Euploidy
← Aneuploidy
← Monoploidy
← Which of the following causes chromosomal diseases?
← Gene mutations
← Genome mutations
← Gene interaction
← Simultaneous action of multiple genes
← Which of the following types of chromosomes correspond to the dicentric chromosomes?
← Duplicated
← Homological
← Abnormal
← Diploid
← Which of the following combinations of Robertsonian translocation is most common?
← 13q14q and 15q22q
← 13q14q and 16q15q
← 13q14q and 14q21q
← 13q14q and 11q12q
← In which phase of mitosis high quality G or R banding of the chromosomes can be accomplished?
← Interphase
← Prophase
← Anaphase
← Telophase
← 24 types of chromosomes have been identified in humans. Which of the following methods
is used for their identification?
← Biochemical
← Population-statistical
← Cytogenetic
← Cloning
← Which type of inheritance is characterized with the transmission of a disease from parents
to children in every generation?
← Autosomal-recessive
← X-linked
← Y-linked
← Autosomal-dominant
← Which karyotype corresponds to the turner syndrome?
← 42,XXX
← 47,XXY
← 46,XX
← 45,XO
← Which of the following is a chromosomal mutation?
← Inversion
← Triploidy
← Aneuploidy
← Polypoidy
← Which of the following is a chromosomal mutation?
← Inversion
← Triploidy
← Aneuploidy
← Polyploidy
← Which of the following karyotypes is a characteristic to patau syndrome?
← 47,XX,18*
← 47,XX,13*
← 47,XXY
← 45,XO
← Which pathology of a karyotype is lethal?
← X chromosome monosomy
← Trisomy in sex chromosomes
← Monosomy in autosomes
← Trisomy in autosomes
← Imprinting is a normal process, which is caused by the chromatin changes in germline cells
of only one parent. This change may be
← Covalent modification
← Modification of RNA
← Modification of DNA
← Modification of RNA polymerase
← Which of the following types of mutations causes aneuploidy and polyploidy
← Gene
← Genome
← Chromosomal
← Any type of mutation
← Which of the following medical examinations should a couple undergo, who have a son with CF?
← Cytogenetic
← Biochemical
← X ray
← Endoscopic
← Which is the name of a normal gene, which, when activated by a mutation, is transformed to an
oncogene.
← Suppressor
← Regulator
← Modificator
← Proto-oncogene
← Upon karyotype analysis, Michael is found to be 47, XYY. The additional Y chromosome
likely arose as a result of which of the following events?
← Cystic fibrosis (CF)
← Galactosemia
← Huntington disease
← Tay-Sachs disease
← In applying the Hardy-Weinberg equilibrium the following assumptions are made.
← The population is small
← There is no consanguinity
← New mutations do happen
← There is significant movement of population.
← Anticipation is characteristic of conditions caused by
← Microdeletions
← Mitochondrial inheritance
← Genomic imprinting
← Trinucleotide repeat expansions
← Match the following clinical description – flat occiput, Brushfield spots, atrioventricular canal
cardiac defect, single palmar creases - with the most likely chromosome abnormality
← Trisomy 13
← Trisomy 18
← Trisomy 21
← Monosomy X
← Match the following clinical description – second trimester abortus with cystic hygroma and
massive hydrops – with the most likely chromosome abnormality:
← Trisomy 13
← Trisomy 18
← Trisomy 21
← Monosomy X
← At the early stage of development X inactivation takes place in the somatic cells of
normal women. What is the aim of this process?
← It suppresses X-linked genes
← It activates X-linked genes
← It allows variability of X-linked genes
← It allows equal distribution of X-linked genes in both sexes
← Hydatidiform Moles is an anomaly of placenta. Most of the cases are diploid with 46,XX
karyotype. From which parent do these chromosomes come from?
← 40% are maternal
← 50% are maternal and 50% are paternal
← All are maternal
← All are paternal
← Which of the following is characterized by the junction of two acrocentric chromosomes
near the centromere with the deletion of short arms?
← Deletion
← Duplication
← Reciprocal translocation
← Robertsonian translocation
← You have sent a blood sample of a dysmorphic baby to the laboratory for the chromosomal
analysis. The answer from the lab says that the baby 46,XY karyotype, del(18)(q12). What is
the reason the baby’s parents should also do a blood analysis?
← To determine whether anomaly is SeZenilia or not
← To determine whether anomaly is paternal
← To determine whether anomaly is maternal
← To determine if there is a family history of the disease.
← Which of the following statements is true concerning the function of proto-oncogenes:
← Proto-oncogenes serve as a signal for cellular apoptosis
← Proto-oncogenes are components of cell growth pathways
← Proto-oncogenes are cell checkpoint regulators
← Proto-oncogenes repair DNA damage across the genome
← Which disease is most commonly associated with a mutation induced cell-membrane
transport protein alteration.
← Huntington disease
← Galactosemia
← Cystic fibrosis
← Phenylketonuria
← Which of the following properties of a gene is characteristic to many types of cancer?
← Duplications
← Amplification
← Insertion
← Translocations
← Which of the following processes will develop if the growth of cancer cells becomes
uncontrolled?
← Invasion will occupy neighbor tissues
← There will be no invasion at all
← Invasion will occupy all tissues
← Invasion will be suppressed
← What is the number of nucleotide base paired in human haploid genome?
← 6 million
← 6 billion
← 3 million
← 3 billion
← Which of the following is characteristic to the mitochondrial type of inheritance
← The disease is more often in males
← The disease is transmitted only maternally
← All children of the affected males are also affected
← Only females are affected
← In which of the following conditions cytogenic analysis is used?
← Cataract
← Anemia
← Congenital anomalies
← Diabetes
← Which of the following is the coding gene of telomerase?
← Topoisomerase
← Helicase
← Reverse transcriptase
← RNS polymerase
← Which of the following processes is characterized by the uncontrolled proliferation of the cells
← Neoplasia
← Hyperplasia
← Aplasia
← Hypoplasia
← Which of the following forms of trisomy are most common in live births?
← Trisomy 16
← Trisomy 17
← Trisomy 18
← Trisomy 21
← In some chromosomes the centromere is in the middle of the chromosome, and the arms are
of equal size. Which of the following corresponds to this type of chromosome?
← Metacentric
← Submetacentric
← Acrocentric
← Telocentric
← In some chromosomes, the centromere is very close to one end of the chromosome, and
one arm is much longer than the other. Which of the following corresponds to this type of
chromosome?
← Metacentric
← Submetacentric
← Acrocentric
← Telocentric
← In general, full monosomies are not compatible with life. Which of the following
chromosomal monosomies is an exception?
← X chromosome monosomy
← 21 chromosome monosomy
← 22 chromosome monosomy
← 17 chromosome monosomy
← Which of the following chromosomal trisomies are compatible with life?
← Trisomy 17
← Trisomy 19
← Trisomy 20
← Trisomy 13
← Which of the following chromosomal trisomies are compatible with life?
← Trisomy 12
← Trisomy 17
← Trisomy 19
← Trisomy 21
← In which of the following diseases cytogenic analysis is recommended most often.
← PKU
← Galactosemia
← Malignant neoplasia
← Diabetes mellitus
← 24 types of chromosomes have been identified in humans. Which of the following methods
used for their identification?
← Biochemical
← Population-statistical
← Special karyotyping
← Cloning
← Which method is used for the identification of mosaicism?
← Cytogenetical Neha says.
← Twin studies
← Population-statistical
← Biochemical
← Despite the fact that the reason of aneuploidy has not been studied completely, it is well known
that is most common chromosomal mechanism is:
← Mitotic nondisjunction in prophase
← Mitotic nondisjunction in anaphase II
← Meiotic nondisjunction in prophase
← Meiotic nondisjunction in anaphase
← Which of the following types of inheritance is characterized by the fact that the sons of
the affected fathers are healthy and the daughters are affected?
← X linked dominant
← X linked recessive
← Autosomal dominant
← Autosomal recessive
← Which of the following is a genomic mutation?
← Inversion
← Deletion
← Aneuploidy
← Chromosomal
← In women what age is considered a risk factor for having a child with chromosomal anomaly?
← 20-25
← 25-30
← 30-35
← 35-40
← Which of the following karyotypes is characteristic of Kleinfelter syndrome?
← 45,XO
← 47,XXX
← 47,XXY
← 47,XYY
← Which of the following karyotypes is characteristic to Edwards syndrome?
← 46,XY,21*
← 47,XXY
← 47,XX,12*
← 47,XX,18*
← Which of the following cell type is best for cytogenetic analysis
← Bone marrow cells
← Liver cells
← Heart cells
← Muscle cells
← The mutation in which of the following chromosomes causes patau syndrome?
← 13
← 18
← 21
← 5
← Insertion is the type of nonreciprocal translocation which is met when one segment of
the chromosome goes/shifts to:
← The end of its own chromosome
← The beginning of its own chromosome
← The middle of its own chromosome
← The other chromosome and maintains it’s orientation
← Which of the following processes causes strong proliferation of cells?
← Aplasia
← Neoplasia
← Hypoplasia
← Hyperplasia
← Mutation of oncogenes, their overexpression or amplification in somatic cells may cause:
← Neoplastic transformation
← Hyperplastic transformation
← Hypoplastic transformation
← Aplasic transformation
← The first apoptotic gene associated with cancer was identified in which of the following typed
of cancer?
← Lymphoma
← Lypoma
← Sarcoma
← Myoma
← Which of the following mechanisms is distorted during autoimmune lymphoproliferation
syndrome?
← Photoreactivation
← SOS-reparation
← Lymphocyte apoptosis
← Excessive reparation
← Which of the following are onocomires?
← Transport RNA
← Information RNA
← Micro RNA
← Ribosomal RNA
← Which one of the following statements best describes most malignant neoplasms
← They are associated with constitutional chromosomal abnormalities
← They are of multifunctional etiology
← They are due to an inherited mutation of an oncogene
d. They result
from activation of tumor suppressor genes
92.
SET 2-2
Friday, July 17, 2020 11:13 PM

← What is the complementary strand for the DNA of the following RNA strand: 5'
GCACGUUUACCGA 3'?
←3' AUGCGUUUACCGA 3';
←3' CGTGCAAATGGCT 5';
←3' AGCCAUUUGCGUA 5';
←3' UACGCAAAUGGCU 5'.

← What is different about interphase before and after meiosis I?

← There is DNA replication before, but not after, meiosis I


← DNA replication occurs at double the rate before meiosis I;
← DNA replication occurs at half the rate before meiosis 1;
← There is DNA replication after, but not before, meiosis I.

← The human haploid genome is composed of approximately:

←6billion base pairs of DNA and 20000 protein coding ge


←3 billion base pairs of DNA and 20000 protein coding genes;
←3 million base pairs of DNA and 20000 protein coding genes;
←6 million base pairs of DNA and 2000 protein coding genes.

← DNA exists in a double-stranded form whereas RNA is mainly a single stranded molecule. What
is the likely reason for DNA being double stranded?

← DNA cannot exist in the single stranded fornyf


← Double stranded DNA is a more stable structure;
← RNA strands cannot form base pairs;
← It is easier to replicate double stranded DNA than single stranded RNA.

← Okazaki fragments:

←Remove the primer;


←Are formed in the leading strand;
←Are formed in the lagging strand;
←Are synthesized by primase.

← Which of the following statements about mitochondrial genome is true:

←There are 13 tRNA gene in the mitochondrial genome;


←There are 22 tRNA gene in the mitochondrial genome;
←They are 13 protein coding gene in the mitochondrial genome;
←There are 22 protein coding gene products from the mitochondrial genome.

← Which of the following stages is characterised by the replication of DNA?

←Metaphase of mitosis;
B. Stage between two meiotic divisions;
C. Stage before I meiotic division;
D. Prophase of mitosis.

← Which statement about base substitutions is true:

← Result in frameshift mutations;


← Can not affect splicing;
← Are always pathogenic;
←May result in nonsense mutations.

← Which statement about transcription is true


.
A. Occurs in the nucleus.
B. Describes the production of polypeptides from the mRNA template;
C. Produces single-stranded mRNA using the sense DNA strand as a template;
D. Is regulated by transcription factors that bind to the 3' UTR.

← Which of the following processes take part in a nucleus of a cell?

← Binding of amino acids with transport RNA;


← Translation;
← Peptide bonds between amino acids;
←Transcription

← Which of the following statements about DNA replication is TRUE?

← Both the leading and lagging strands are replicated continuously;


← The leading strand is replicated discontinuously, while the lagging strand is
replicated continuously;
← The leading strand is replicated continuously, while the lagging strand is
replicated discontinuously;
←Both the leading and lagging strands are replicated discontinuously.

← RNA processing varies in different cells. This is an example of

← Transcriptional control of gene expression;


← Translational control of gene expression;
← Post-transcriptional control of gene expression;
←Post-translational control of gene expression.

← Which of the following processes take part in the cytoplasm-of a cell?

← Transcription;
← RNA processing
← Binding of ammo acids with transport RNA;
←RNA splicing

← What is an exon?

← It is a segment of a split gene that codes for amino acids in a protein;


← It is a DNA spacer between genes;
← It is a segment of a split gene that has no corresponding amino acids in a protein;
←It is a regulatory gene.
15. Which of the following does not regulate normal gene expression?

← Promotor;
B.;
← Locus control region Enhancer;
←Poly-A tail.

← Which of the following undergoes processing?


← t-RNA;
← DNA
← m-RNA
←r-RNA.

← Which of the following does NOT occur as a result of post translational modifications of a
protein?

← Addition of phosphate groups;


← Addition of metals to create tertiary or quaternary structures;
← Removal of introns;
←Removal of N-terminus amino acid.

← What effect can methylation of CpG iclands in the promoter region have on gene expression?
← Transcription level will be reduced
← Transcription level is sharply increased;
← It will affect translation;
←It will affect splicing process.

← Which of the following processes causes diversity of antibodies?

← Transcription;
← Posttranslational processing;
← Translation;
←Somatic reconstruction.
← What is the first stage of chromatin packing?

← Formation of a solenoid;
← Coiling around nucleosomes;
← Looping of 300-nm fibers;
←Formation of a 300-nm fiber.

← Which of the following is true about histone proteins:

← They bind to the base pairs of DNA;


← They contain methionine and valine;
← They are positively charged;
←There are about 20 histone types associated with DNA.

← Which of the following statements is true regarding transposons?


← They are sequences of mRNA that can move around in the genome;
← They are the most abundant type of repeat in the genome;
← They exist in corn, but are not found in the human genome;
← All of the above are true regarding transposons.

← An octamer of 4 histones complexed with DNA forms:


←Endosome
←Nucleosome;
←Mesosome;
←Centromere.

← Which of the following is NOT an example of repetitive DNA?

← Telomeric DNA;
←Promotor region;
← Centromeric DNA;
← SINE DNA.

← In eukaryotes, many genes may have to interact with each other, requiring more interacting
elements than can fit around a single promoter. This physical limitation is overcome by:

← Alternating promoters and operators;


← Distant sites in a chromosome controlling transcription of a gene;
← The use of very long promoters;
← Placing promoters on both sides of each gene.

← Pseudogenes are:
← Nonfunctioning genes;
← Recent beneficial mutations;
← Randomly transposed genes;
← Artificial DNA sequences constructed by geneticists and inserted into the DNA of
living organisms.

← When considering the initiation of transcription, one often finds consensus sequences located in
the region of the DNA where RNA polymerase(s) binds. Which of the following is a common
consensus sequence?

← TTTTAAAA;
← GGTTC;
← TATA;
← Satellite DNAs.

← In addition to highly repetitive and unique DNA sequences, a third category of DNA
sequences exists. What is it called, and which types of elements are involved in it?

← Composite DNA; telomeres and heterochromatin;


← Dominant DNA; euchromatin and heterochromatin;
← Multiple gene family DN'A; hemoglobin;
← Moderately repetitive DNA; SINES, LINEs, and VNTRs.

← X chromosome inactivation is a process by which one of two X chromosomes in human


females is
condensed and inactivated to prevent overexpression of X-linked genes. What would most likely be
true about the degree of DNA methylation and histone acetylation on the inactivated X
chromosome?

← Cytosines in CG is lands would be hyperacetylated and histone proteins


would be hypomethylated;
← Cytosines in CG is lands would be hypomethylated and histone proteins
would be acetylated;
← Cytosins in CG is lands would be hypermethylated and histone proteins
would be deacetylated;
← Cytosines in CG is lands would be deacetylated and histone proteins
would be hypermethylated.
← Approximately 5% of the cytosine residues are methylated in the genome of any given eukaryote. In
what way is DNA methylation related to genetic regulation?
← There is an inverse relationship between the degree of methylation of DNA and the
degree of gene expression;
←Methylation of DNA promotes upregulation of gene expression;
←To the methylated regions of DNA specific transcription factor proteins attached;
←Active chromatin configuration is characterized by methylation of CpG.

← DNA methylation may be a significant mode of genetic regulation in eukaryotes. Methylation


refers to:
← Altering RNA polymerase activity by methylation;
←Addition of methyl groups to the cytosine of CG doublets;
C. Altering translational activity, especially of highly methylated tRNAs;
D. Changes in DNA-DNA hydrogen binding.

← Which of the following processes is called a segregation between homological chromosomes


during meiosis?

←Fusion;
← Disjunction;
←Exchange of the segments;
←Formation of bivalents.

← In what way do mitosis and meiosis differ?

← Centrioles form during mitosis, they do not form during meiosis;


← Replication of DNA precedes mitosis, it does nor precede meiosis;
← Mitosis produces cells which are identical, meiosis produces cells which are
not identical;
← Chromosomal spindles shorten during mitosis, they do not during meiosis.

← During metaphase all of the following takes place EXCEPT:


←Chromosomes are in their most condensed form;
←Chromosomes move to opposite spindle poles;
←Microtubules attach to sister chromatids of each chromosome; .
←Chromosomes line up on the spindle equator.

← If at the end of meiosis, the daughter cell has 12 chromosomes, how many chromosomes were in
the mother cell?

←3;
←6;
←12;
D.24.
← Antiparallel strands of a DNA molecule mean that:

← The phosphate groups at the start of two DNA strands are in opposite position (pole);;
←One strand turns anti-clockwise;
←One strand turns clockwise position;
←The phosphate groups of two DNA strands, at their ends, share the same
← A genotype in which two different mutant alleles of the same gene are present, rather than one
normal and one mutant is referred to as:
←Homozygote;
←Compound heterozygote;
←Hemizygote;
← Heterozygote.

← Tay-Sachs disease shows autosomal recessive inheritance. Parents of a newly diagnosed affected
child arc referred for genetic counseling. It would be correct to tell them that:

← the probability that their next child will be affected is 1 in 2;


← the probability that the older unaffected sister of the affected child is a carrier is I;
← the fact that their last child was affected means that their next three children will not
be affected;
← the probability that each parent is a carrier is I.
← Huntington's disease (HD) is inherited as an autosomal dominant.Patients first exhibit symptoms as
adults
(90% between the ages 25 and 60, with the average in the early 40s). They suffer a deterioration of the
central system and lose mental abilities and motor control of the limbs. The disease is fatal
within about 10
years after the onset of symptoms. What is the probability two individuals homozygous for
HDwill have a child with HD?
← 25%
← 50%
← 75%
← 100

← Galactosemia is an autosomal recessivedisorder. Symptoms include failure to thrive, vomiting,


jaundice, hepatomegaly, and cataracts. It is most commonly due to a deficiency of galactose-
phosphate uridyltransferase (GALT). A man and woman, who are both known to be carriers for
galactosemia, marry and have a child.To their relief,the standard newborn screening test
results, which measures the blood level of galactose and galactose-I -P, shows that their child is
affected. Based upon this information, what is the probability that the child has the disease?
← 0%;
B. 25%
← 50%;
←75%.

← Ellis-van Creveld syndrome is a type of dwarfism inherited as an autosomal recessive disease. It is


rare in the general population of the United States, but it is unusually common in the Amish people
of Pennsylvania. People with this disease have a normal body length, but have shortened limbs and
appendages as well as extra fingers or toes. Dick suffers from Ellis-van Creveld syndrome. Margie has
a normal stature, but her father suffered from Ellis-van Creveld syndrome. What iS the probability
that Dick and Margie will produce a dwarf child?
←0%;
←25%;
C. 50%;
← 100%.

← An autosomal dominant trait exhibited by some Caucasians is called "woolly hair". Mutation in this
allele
produces extremely brittle hair that breaks off before it grows very Iong. What is the probability two
individuals heterozygous for this trait having a normal child?

0%;
B.25%;
50%;
75%.

࿿101࿿h The probability that a gene will have any phenotypic expression at all is referred to as:

Expressivity;
Penetrance;
reduced penetrance;
variable expressivity.

࿿102࿿h The severity of a disease differs in people who have the same genotype. In this case
the phenotype is said to have:

Penetrance;
Expressivity;
Reduced penetrance; !

Variable expressivity.
࿿103࿿h What is called a set of alleles at a locus or locus clusters in chromosomes?
Wild Wild type;
Polymorphism;
Homozygote;
Haplotype

࿿104࿿h Which statement about X-linked inheritance is true:


← The condition cannot be passed from an affected father to his son;
← The condition can be passed from an affected father to his son;
← When dominant, females are usually as severely affected as males;
← The risk of germline mosaicism does not need to be considered.

࿿105࿿h Which term is used to describe differences in gene expression based on the parent of
origin of an allele?

imprinting;
Sex-determination;
Parental determination;
Sex-linked dominance.
࿿106࿿h In familial hypercholesterolemia, individuals homozygous for the allele causing the
disorder completely lack receptors on liver cells that take up cholesterol from the bloodstream.
Heterozygotes have one-half the number of receptors while individuals homozygous for the normal
allele are phenotypically normal. This is an example of:

Codominance;
Incomplete dominance;
Epistasis;
Complete dominance.

࿿107࿿h Which of the following clinical phenotypes is characteristic to the mutations of mtDNA?
Neural and parenchymal
disorders;
Disorders of the sensory organs;
Neuromuscular disorders;
Skeletal

࿿108࿿h The presence in an individual or a tissue of at least two cell lines that differ
genetically but are derived from a single zygote is referred to as:
Mosaicism;
Trinucleotide repeat expansions;
imprinting;
Mitochondria! inheritance.

࿿109࿿h Which of the following is a consequence of mutation?

Prevention of protein formation;


Lowering in the amount of a protein;
Increases in the amount of a protein;
Any of the above can occur.

࿿110࿿h Which of the following is characteristic to the expansions of unstable repeat sequences?

Locus heterogeneity;
Anticipation;
Haploinsufficiency;
Mozaicism.

࿿111࿿h The mutations are mainly responsible for

Increasing the population rate; B.


Variation in organisms;
C. Constancy in organisms;
D. Maintaining genetic continuity.

࿿112࿿h Most known human mutations:

Are caused by radiation;


Appear to be spontaneous;
Are due to viruses;
Are germline.

࿿113࿿h Those mutations that arise in the absence of known mutagen are known as:
Spontaneous mutations;
Fused mutations;
Induced mutations;
None of the above.

࿿114࿿h A nonsense mutation introduced into the DNA sequence of a gene may:
cause premature termination of the mRNA;
Shorten the length of the protein encoded by the gene;
Have no effect on the transcript or protein made;
Cause a shift in reading frame.
࿿115࿿h A single nucleotide substitution (or point mutation) in a DNA sequence can alter the
code in a triplet of bases and cause the replacement of one amino acid by another in the gene
product.

Nonsense
mutations
← Missensce
mutations;
← Framcshift mutation;
← Silence mutations.
࿿116࿿h Which of the following illustrates a single point mutation on a segment of
DNA which reads GTGGCACAT?

← GTGGGACAU;
← GTGGCAGAT;
← TACACGGTG;
← GTGGGCCACAT.

࿿117࿿h A nucleotide deletion in DNA replication:


← CAUSES the amino acids inserted after the deletion to be incorrect;
Causes all of the amino acids cf the protein to be incorrect;
Causes one amino acid of the protein to be incorrect;
D.Causes the amino acids insened before the deletion to be incorrect.
࿿118࿿h Substitution of one or the other of two bases at one location is referred to as:
Indel;
Single Nucleotide polymorphism SNP;
Simple;
Short tandem repeat polymorphism STRP.
࿿119࿿h What's the name of the sequence that is the most common in a population?
Reference sequence;
Variant sequence;
Private sequence;
All of the above.
࿿120࿿h The mutation, cusing sickle-cell disease is GAG --> GTG in the beta-globin gene
. This is an example of

Frameshift mutation;
Missense mutation;
Nonsense mutation;
Dynamic mutation.

࿿121࿿h Population carries two or more different alleles at a particular locus.


Which of the following definitions best corresponds to the above mentioned
statement?

Polymorphism;
Hctcrozygotc;
Homozygote;
Compound.

࿿122࿿h When only one member of the pair of relatives is affected and the other is not, the relatives
are:

Concordant;
B.Discordant;
C. Quantitative traits;
D. Qualitative trait.
࿿123࿿h Because of its high level of informativity, the Federal Bureau of Investigation (FBI) uses a set of
13 of this type of marker for identity testing.
Variable number of tandem repeats
(VNTR);
Single nucleotide polymorphisms (SNPs);
Copy number polymorphisms (CNPs);
AIl of above.
࿿124࿿h When should RhIG he injected to the Rh"-" woman?

← After miscarriage or medical abortion;


← After amniocentesis or chorionic villus sampling
← After delivery during the first 72 hours;
D. all of the above
Which of the following is true about indels?

A.they are crucial for identity testing;


B. There are several millions of indels in the genome;
C. They have only 2 alleles;?????
D. The DNA sequences of indels are approximately 200-500 bp.

࿿125࿿h VNTRs are:


Used in DNA fingerprinting;
Variable in number;
Tandem repeats;
All of the above are correct.
࿿126࿿h Which of the following is directly involved in DNA repair:
Topoisomerase;
Ribosoines;
DNA polymerases;
Telomerases.

࿿127࿿h Which of the following statements is true of DNA damage?


Any DNA damage results in diseases such as cancer;
Most DNA damage is repaired by the cell;
← Any DNA damage is caused by physical, chemical or biological agents;
← Most DNA damage is advantageous to the cell.

࿿128࿿h According to the X-linked genes males are:

40.Hemizygotes;
41. Homozygotes;
42. Heterozygotes;
43.Compounds.

࿿129࿿h DNA proofreading reparation takes place during which of the following process?

40.During GI phase;
41. During DNA synthesis;
42. After DNA replication .
43.A and C.

࿿130࿿h Which of the following statements about twins and twin studies is FALSE?

a. Dizygotic twins are an important control for any study using monozygotic twins;
b. Twin studies have been used to try to determine the role of heredity versus
environment;
← if the concordance values for monozygotic twins are much higher than for dizygotic
twins, the trait probably has a strong genetic basis;
← D. Dizygotic twins are as genetically similar as monozygotic twins,
࿿131࿿h If you have an aunt or uncle with a polygenic disorder:
← You will be more likely to develop the disorder than your mother or father;
← You are at increased risk of developing the disorder than the average
member of your population;
← You arc not at risk to have children with the disorder;
← You will eventually develop that disorder.
࿿132࿿h Which of the following statements about normal phenotypic variation is true?

Most normal characteristics in bays are inherited from their fathers and most normal
characteristics in girls are inherited from their mothers;
The genes involved are the same in all populations;
Environmental factors play a role;
Most normal characteristics such as eye or hair color are transmitted as
autosomal recessive traits.

࿿133࿿h Which of the following statements is TRUE about concordance?

We can talk about concordance only when considering monozygotic twins;


Concordance means when any two individuals develop the same disease;
← Concordance always results from the same genotypes;
← Concordance may be a result of genetic and/or environmental factors.

࿿134࿿h Which of the following is a risk factor for Alzheimer's disease?


← Oral contraceptives;
← Arthritis;
← Use of anticonvulsants;
← Head trauma.

࿿135࿿h Which of the following is NOT a teratogen (an environmental agent that causes
malformation of an embryo or fetus) for cleft lip/palate?

Anticonvulsants (anti-epilepsy drugs);


Rubella virus;
Maternal diabete;
Oral contraceptives.

࿿136࿿h Which of the following is NOT a symptom of placental artery thrombosis?


Premature separation of the placenta from the uterine wall;
Intrauterine growth retardation;
Increased levels of alpha-fetoprotein (AFP) in amniotic fluid;
Preeclampsia.

࿿137࿿h Which of the following is TRUE of a concordant trait between twins?


← It is more common for the males to have the trait;
← It is more common for the females to have the trait;
← It is more common for the older twin to have the trait;
← Both share the trait.

࿿138࿿h In which of the following conditions cytogenetic analysis is used?

← Cataract;
← Anemia;
← Congenital anomalies:
← Diabetes.
࿿139࿿h Which of the following statements about venous thrombosis is true:

← Testing for factor V Leiden and the prothrombin variant will change the
management of a patient with venous thrombosis;
← Male are five time more likely to be affected with venous thrombosis than females;
← The factor V Leiden mutation causes reduced expression of the factor V gene; ?????
← The prothrombin variant causes increased expression of the prothrombin gene.

࿿140࿿h An 80-year-old man presented with impairment of higher intellectual function and
alterations in mood and behavior. his family reported progressive disorientation and memory
loss over the last 6 months. There is no family history of dementia. The patient was
tentatively diagnosed with Alzheimer disease. Which one of the following best describes
Alzheimer disease?
← It is caused by the infectious beta sheet form of host-cell protein;
← It is an environmentally produced disease not influenced; by the genetics of the individual;
← It is associated with the accumulation of amyloid precursor protein;
D. It is associated with the deposition of neurotoxic amyloid p peptide aggregates;
࿿141࿿h what type of chromosomal aberration is indicated by the following cytogenetic
abbreviation: i(X) (ql0)?
← Isochromosome;
← X monosomy;
← Transversion;
← Terminal deletion.

࿿142࿿h Which of the following processes, characteristic to the homological


chromosomes, may cause a genetic disease?

← Crossing over;
← Conjugation;
← Formation of bivalents;
← Nondisjuntion.

࿿143࿿h In general, full monosomies are not compatible with life. Which of
the following chromosomal monosomies is an exception?
← X chromosome monosomy;
← 21 chromosome monosomy;
← 22 chromosome monosomy;
← 17 chromosome monosomy.

࿿144࿿h In which of the following diseases is cytogenetic analysis most frequently


recommended?

← PKU;
← Galactozemia;
C.Malignant neoplasia;
D. Diabetes mellitus.
࿿145࿿h You have sent a blood sample of a dismorphic baby to the lab for the chromosomal
analysis. The answer from
the lab says that the baby has 46,XY,del(18)(q12) karyotype. What type of anomaly is in the
karyotype?

The arms of chromosome 18 are equally long;


The arms of chromosome 18 arc equally short;
One arm of chromosome 18 is very long compared to the other;
D. The long arm of chromosome 18 is shorter that it should be.

࿿146࿿h Which the following causes the bipotential gonad to differentiate into one of the
sexes?
40. Presence of SRY on the Y chromosome leads to male development;
41. resence of SRY on the X chromosome leads to female development;
42. Recombined PAR regions of X and Y lead to male development;
43. Formation of female gonads in the absence of Y chromosome
inhibits male development.
࿿147࿿h Chromosomal aberrations include a change in the sequence of genes on a
chromosome. This is known as:
←a carrier;
← an inversion;
← duplication;
←a translocation.

࿿148࿿h Marcus and Carmen have a child with ambiguous external genitalia. The internal
genitalia are female and chromosome analysis is 46,XX. Biochemical studies reveal a
deficiency of 21-hydroxylase. What is the likely diagnosis?

← Congenital adrenal hyperplasia;


← Camptomelic dysplasia;
← Androgen insensitivity;
← Turner syndrome.
࿿149࿿h Which of the following statement is correct:
← X chromosome inactivation, all the genes of one X are switched off;
← Male fetal development is solely dependent on normal functioning of the SRY gene;
← Female fetal development is solely dependent on normal functioning of the SRY gene;
← X chromosome inactivation may be linked to discordance in monozygotic twin pairs.

࿿150࿿h What is Barr body?


← Inactive Y
← inactive X;
← Active X;
D. Active Y.

࿿151࿿h Which method is used to detect relative copy number gains


and losses in a genome-wide manner by hybridization:
CGH array;
Fluorescent in situ hybridization (FISH) ;
G-banding;
Allele specific oligonucleotide hybridization.

࿿152࿿h Which of the following statement is correct:

In most males with karyotype 46.XX the SRY gene is present and found on one of the
X chromosomes;
X chromosome inactivation, all the genes of one X are switched off;
Male fetal development is solely dependent on normal functioning of the SRY gene;
Female fetal development is solely dependent on normal functioning of the SRY gene.

࿿153࿿h In some chromosomes the centromere is not in the middle of the chromosome,
however there is no significant difference in the length of the arms. Which of the following
corresponds to this type of chromosomes
A. Metacentric;
← Submetacentric;
← Acrocentric;
← Telocentric.

࿿154࿿h Fluorescence in situ hybridization using whole chromosome (painting) or specific


locus probes enables routine detection of:

← Gene expression;
← Triploidy;
← Gene amplification;
← Reciprocal translocation.

࿿155࿿h Which of the following definitions corresponds to the description given as: the
chromosome constitution of individuals:

← Genotype;
← Genome;
← Karyotype;
← Phenotype.

࿿156࿿h What happens to the extra X chromosome in patients with an additional X


chromosome?
← Almost entire chromosome is inactivated;
← p arm is inactivated;
← q arm is inactivated;
←Region near the centromere is inactivated.

࿿157࿿h Dosage compensation in mammals typically involves the random inactivation of one of
the two X chromosomes relatively early in development. In a such X chromosome inactivation
involve:

FMRI gene;
SOX gene;
SRY gene;
XIST gene
SET 2 NEW

࿿158࿿h Which of the following processes is called a segregation between


homological chromosomes during meiosis?

Exchange of the segments

Disjunction

Formation of bivalents

Fusion

࿿159࿿h Which enzyme serves to destabilize the DNA double helix in order to
open it up, creating a replication fork?

DNA endonuclease

DNA polymerase

DNA Helicase

DNA ligase

࿿160࿿h The process of meiosis produces four cells with non-identical


chromosomes. This diversification occurs during

࿿161࿿h Prophase I
࿿162࿿h Prophase II
࿿163࿿h Metaphase II
࿿164࿿h Telophase I

4. If 2n=8, for a particular cell, then the chromosome number in egg cell after meiosis would be

࿿165࿿h 4
࿿166࿿h 8
࿿167࿿h 10
࿿168࿿h 12
5. Which of the following processes does nottake place during somatic reconstruction?
࿿169࿿h Excision of certain DNA sequences
࿿170࿿h Insertion of excised parts in another region of the DNA
࿿171࿿h Splicing and processing of the RNA
࿿172࿿h New construction of genesin the cells

6. Chromosome pairs are identical in both sexes are known as

࿿173࿿h linkage groups


࿿174࿿h Autosomes
࿿175࿿h Carriers
࿿176࿿h Pedigrees

during metaphase all of the above mentioned takes place except

࿿177࿿h Chromosomes line up on the spindle equator


࿿178࿿h Microtubules attach to sister chromatids of each chromosome
࿿179࿿h chromosomes are in their most condensed form
࿿180࿿h Chromosomes move to opposite spindle poles

࿿181࿿h In a Cell, tissue and organism, existence of which of the following


substances at a particular period of time is called a proteome?

࿿182࿿h Carbohydrates
࿿183࿿h Lipids
࿿184࿿h Proteins
࿿185࿿h DNA

9. Meiosis increases genetic variability. Some of this variability is produced in a zygote by

࿿186࿿h Reducing the number of chromosomes after fertilization


࿿187࿿h bearing paternal and maternal chromosomes during fertilization
࿿188࿿h increasing the number of chromosomes after fertilization
࿿189࿿h normal mitotic divisions

10. During Protein synthesis, an anticodon on tRNA pairs with

࿿190࿿h rRNA nucleotide bases


࿿191࿿h DNA nucleotide bases
࿿192࿿h mRNA nucleotide bases
࿿193࿿h Other tRNA nucleotide bases
࿿194࿿h which of the following corresponds to the stage of the cell cycle,
where each chromosome is composed of two chromatids in preparation for
Mitosis?

࿿195࿿h S
࿿196࿿h G1
࿿197࿿h G2
࿿198࿿h M

12. the somatic cells of human females contain

࿿199࿿h 22 pairs of autosomes and two X chromosomes


࿿200࿿h 11 pairs of sex chromosomes and two X chromosomes
࿿201࿿h 22 pairs of sex chromosomes and two X chromosomes
࿿202࿿h 11 pairs of autosomes and two X chromosomes

13. what is an intron?


࿿203࿿h this is a region of DNA a located between genes that is not translated into protein
࿿204࿿h it is a regulatory gene
࿿205࿿h it is a region of DNA located within a gene that is not translated into protein
࿿206࿿h it is a structural gene

14. which of the following is a gene non coding region?

࿿207࿿h Exon
࿿208࿿h Intron
࿿209࿿h Codon
࿿210࿿h Anticodon

࿿211࿿h which of the following is stage of the cell cycle, when chromosomes are
in their most decondensed state

࿿212࿿h Metaphase
࿿213࿿h Anaphase
࿿214࿿h Interphase
࿿215࿿h Prophase

࿿216࿿h one round of DNA synthesis followed by two rounds of chromosome


segregation and cell division is referred to as

࿿217࿿h Metaphase
࿿218࿿h Mitosis
࿿219࿿h Interphase
࿿220࿿h Meiosis
17. First step in the polymerase chain reaction is

࿿221࿿h Denaturation
࿿222࿿h primer extension
࿿223࿿h Annealing
࿿224࿿h Cooling

18. which of the following is not a part of the chromosome?

࿿225࿿h Long arm


࿿226࿿h short arm
࿿227࿿h Centromere
࿿228࿿h Centriole

19. which of the following phases does not belong to Interphase?

࿿229࿿h S
࿿230࿿h G2
࿿231࿿h G1
࿿232࿿h M

࿿233࿿h synthesis of proteins can take place quickly by multiple ribosomes being able
to attach themselves to one mRNA chain. An mRNA chain with multiple ribosomes is called a

࿿234࿿h Peroxisome
࿿235࿿h Lysosome
࿿236࿿h Centrosome
࿿237࿿h Polysome

21. the following statements about gametogenesis are all true except

࿿238࿿h for stem cells are produced from each diploid cell
࿿239࿿h oogenesis produces one ovum and one polar body
࿿240࿿h sperm cells are produced during spermatogenesis
࿿241࿿h Polar bodies get only a tiny amount of cytoplasm

࿿242࿿h which of the following methods described below mentioned process; a


filter to which RNAhas been transferred After gel electrophoresis to separate the RNA
molecules by size, named for the compass point, as a pun on Southern blot; also, the act
of generating such a filter and hybridizing it to a specific probe.

࿿243࿿h Southern blot


࿿244࿿h Northern blot
࿿245࿿h Western blot
࿿246࿿h PCR
࿿247࿿h series of 20 to 200 adenine ribonucleotides there is added to the 3’ end
transcribed RNA molecule that helps in the transportation of mRNA out of the nucleus is
called;

࿿248࿿h Shine-Dalgarno sequence


࿿249࿿h Cap
࿿250࿿h Poly A-tail
࿿251࿿h Trailer

࿿252࿿h fluorescence in situ hybridization using whole chromosome (painting) or


specific locus probes enables routine Detection of

࿿253࿿h gene amplification


࿿254࿿h Trisomy
࿿255࿿h gene expression
࿿256࿿h Triploidy

25. which of the following divisions is known as the reductional division of a cell?

࿿257࿿h mitotic prophase


࿿258࿿h II meiotic division
࿿259࿿h I meiotic division
࿿260࿿h Mitotic anaphase

࿿261࿿h technological revolution that solve the problem of obtaining DNA in sufficient
quantity and purity for detailed analysis is referred to as

࿿262࿿h Southern blot


࿿263࿿h Northern blot
࿿264࿿h Hybridization
࿿265࿿h polymerase chain reaction (PCR)

࿿266࿿h the primary structural unit of the chromatin - nucleosome consists of the
146 BP. DNA wrapped twice around a core, which is composed of the histone proteins.
which of the following histone proteins are not responsible for the formation of the
nucleosome?

࿿267࿿h H2A
࿿268࿿h H3
࿿269࿿h H1
࿿270࿿h H4

28. types of nucleic acid hybridization include

࿿271࿿h Biochemical
࿿272࿿h Cytogenetic
࿿273࿿h western blotting
࿿274࿿h Southern blotting
࿿275࿿h which of the following processes take part in the

cytoplasm of a cell? A. binding of amino acids with transport RNA

B. transcription C.

RNA processing D.

RNA splicing

࿿276࿿h A Promoter is

࿿277࿿h start signal for transcription


࿿278࿿h binding site for DNA polymerase
࿿279࿿h stop signal for transcription
࿿280࿿h none of these

31. by which of the following bonds are two strands of DNA connected with each other?

࿿281࿿h peptide bonds


࿿282࿿h hydrophilic bonds
࿿283࿿h hydrogen bonds
࿿284࿿h hydrophobic Bonds

32. during DNA replication what enzyme adds complementary bases?

࿿285࿿h Helicase
࿿286࿿h Synthesase
࿿287࿿h Replicase
࿿288࿿h DNA polymerase

33. genescan be mapped to sub chromosomal regions using

࿿289࿿h fluorescence in situ hybridization (FISH)


࿿290࿿h Pulsed Field gel electrophoresis (PFGE)
࿿291࿿h fluorescence activated chromosome sorting
࿿292࿿h somatic cell hybridization
34. which statement does not describe Mitosis?
࿿293࿿h in some multicellular organisms it allows for asexual reproduction
࿿294࿿h it is a type of division that produces identical cells
࿿295࿿h it is a process which duplicate the hereditary material
࿿296࿿h it is a type of division that may produce generation after generation of identical cells

࿿297࿿h the unit within the nucleus that contains a protein Complex of two H2A, H2B,
H3 and H4 histone proteins with DNA wrapped around the complex in two turns is called a

࿿298࿿h Ribosome
࿿299࿿h Nucleosome
࿿300࿿h Centrosome
࿿301࿿h Histosome

36. how many nucleotide base pairs of the DNA correspond to 1 Megabase?

࿿302࿿h 100bp
࿿303࿿h 1000bp
࿿304࿿h 10 000bp
࿿305࿿h 1 000 000bp

࿿306࿿h which of the following describes a process of formation of


phosphodiester Bond when an enzyme binds two double stranded DNA sequences
together?

࿿307࿿h Amplification
࿿308࿿h Hybridization
࿿309࿿h Ligation
࿿310࿿h Restriction

38. which of the following is true about single-stranded binding proteins

࿿311࿿h bind to the single stranded DNA and keep them separate in order to
maintain the stability of the replication fork
࿿312࿿h bind to the single stranded DNA and prevents supercoiling of the double helix
࿿313࿿h attaches a small RNA primer to the single stranded DNA to act as a substitute
3’-OH for DNA polymerase to begin synthesis from
࿿314࿿h monitor the speed of replication

39. which of the following undergoes processing?

࿿315࿿h DNA
࿿316࿿h tRNA
࿿317࿿h mRNA
࿿318࿿h rRNA
40. which of the following processes take part in the nucleus of a cell?

࿿319࿿h Translation
࿿320࿿h post translational processing
࿿321࿿h RNA splicing
࿿322࿿h binding of amino acids with transport RNA

41. in a DNA molecule what type of linkage links the nucleotides together?

࿿323࿿h peptide bonds


࿿324࿿h Ester linkages
࿿325࿿h hydrogen bonds
࿿326࿿h phosphodiester linkages

࿿327࿿h if the sequence of bases in DNA is TAGC, then the sequence of

bases in RNA will be AUCG

࿿328࿿h which statement applies to restriction enzymes

࿿329࿿h they are used in Southern blotting


࿿330࿿h they are viral in origin
࿿331࿿h they are western blotting
࿿332࿿h they are also called restriction exonucleases

44. which of the following processes does not take place during splicing in eukaryotes
࿿333࿿h binding of exons
࿿334࿿h cut of introns
࿿335࿿h binding of introns
࿿336࿿h cut off exons

࿿337࿿h what is the length of the mitochondrial


DNA?
࿿338࿿h16 000 bp
࿿339࿿h 19 000 bp
࿿340࿿h 30 000 bp
࿿341࿿h 20 000 bp

࿿342࿿h in situ hybridization using whole chromosome (painting) for specific probes
enables routine detection of

࿿343࿿h Gene Expression


࿿344࿿h subtelomeric deletion
࿿345࿿h Triploidy
࿿346࿿h Gene amplification
࿿347࿿h which of the following processes refers to the crossing over
between the homological chromosomes?

࿿348࿿h formation of bivalents


࿿349࿿h Disjunction
࿿350࿿h Exchange of the segments
࿿351࿿h Fusion

48. which of the following statements is true about meiosis in humans?

࿿352࿿h it takes roughly two weeks for a cell to go through all phases of meiosis
࿿353࿿h Sperm and ova are not identical to the parent cells that produced them
࿿354࿿h the process begins in males and females at puberty
࿿355࿿h Females produce far more gametes than do males

49. What is the complementary strand of DNA to the DNA sequence 3’GTAGCCGTAACGTAT5’

࿿356࿿h 5’CATCGGCATTGCATA3’

50. discovery of okazaki fragments suggest that DNA synthesis is

࿿357࿿h Semi conservative


࿿358࿿h Discontinuous
࿿359࿿h Continuous
࿿360࿿h 3’ to 5’

࿿361࿿h Haemophilia A is disease in which patients lack a clotting factor in the blood
(factor
VIII). It i is inherited as an x-linked recessive trait. Eric has a normal blood. His wife Jane is
the daughter of heamophilic fatherand a homozygous normal mother. What are the
chances that Eric and Jane will have a daughter who is a carrier?
࿿362࿿h 0%
࿿363࿿h 25%
࿿364࿿h 50%
࿿365࿿h 75%
࿿366࿿h Elizabeth is a 25-year-old woman with color blindness, an X Linked recessive
disorder. Aside from being colour blind, her medical history is unremarkable. Her father is
also color blind. There is no history of color blindness on her mother’s side of the family.
What is the likely cause of Elizabeth's color blindness?

࿿367࿿h Turner syndrome


࿿368࿿h XY sex reversal
࿿369࿿h Skewed X inactivation
࿿370࿿h Klinefelter syndrome

࿿371࿿h In some chromosomes the centromere is in the middle of the chromosome,


and the arms are of equal size. Which of the following corresponds to the type of
chromosome?
࿿372࿿h Submetacentric
࿿373࿿h Acrocentric
࿿374࿿h Metacentric
࿿375࿿h Telocentric

࿿376࿿h Which of the following is the most common autosomal recessive disorder
of white children with incidence of /2000?
࿿377࿿h Cystic fibrosis
࿿378࿿h Phenylketonuria
࿿379࿿h Tay-Sachs disease
࿿380࿿h Sickle Cell anaemia

࿿381࿿h Select the most closely associated kind of genetic disease, when there is a
mechanism responsible for most common diseases of adulthood, such as coronary artery
disease and non- insulin dependent diabetes
࿿382࿿h X linked recessive
࿿383࿿h autosomal recessive
࿿384࿿h multifactorial inheritance
࿿385࿿h Autosomal dominant

࿿386࿿h Severity of a disease differs in people who have the same genotype. In
This case the phenotype is said to have
࿿387࿿h Variable expressivity
࿿388࿿h Penetrance
࿿389࿿h Expressivity
࿿390࿿h reduced penetrance
57. Which statement about the autosomal recessive inheritance is true?
࿿391࿿h Usually only a single generation has affected individuals
࿿392࿿h Angelman syndrome follows this pattern
࿿393࿿h if both parents are Carriers the risk at conception that any child might be a
carrier is
25%
࿿394࿿h Females are more likely to be affected than males

58. which disease is caused by a defect in protein hemoglobin?


࿿395࿿h Polyploidy
࿿396࿿h Sickle Cell anaemia
࿿397࿿h Achondroplasia
࿿398࿿h Aneuploidy

59. which of the following types of mutation does not change the meaning of a codon?
࿿399࿿h Nonsense
࿿400࿿h Frameshift
࿿401࿿h Missense
࿿402࿿h Samesense

60. Which of the following genetic diseases affect males almost exclusively?
࿿403࿿h Autosomal recessive
࿿404࿿h X linked recessive
࿿405࿿h chromosome abnormality
࿿406࿿h autosomal dominant

61. which of the following corresponds to the alternative form of a gene?


࿿407࿿h Allele
࿿408࿿h Compound
࿿409࿿h None Allele
࿿410࿿h Hemizygote

62. In X linked recessive inheritance


࿿411࿿h The gonadal mosaicism risk in Duchenne Muscular dystrophy may be as high as 10%
࿿412࿿h The sons of a Female carrier have one in four chance of being affected
࿿413࿿h the mother of an affected male is an obligate carrier
࿿414࿿h A dummy consultant refers to an individual in a Pedigree who is ignored
when it comes to calculating risk

63. how many mutant alleles does a compound heterozygote have in different loci?
࿿415࿿h 2
࿿416࿿h 3
࿿417࿿h 4
࿿418࿿h 5
64. which of the following statements is true
࿿419࿿h myotonic Dystrophy is a cause of neonatal hypotonia
࿿420࿿h In HD, non-penetrance of the disease may be associated with low repeat
abnormal alleles?
࿿421࿿h from the onset of HD, the average duration of the illness until a terminal event
is 25 to 30 years
࿿422࿿h In Huntington Disease (HD) an earlier age of onset in the of spring is more
likely if the gene is passed from an affected mother rather than an affected father
65. Which of the following statements is true
࿿423࿿h normal karyotyping is a good way of diagnosing fragile X syndrome in girls
࿿424࿿h fragile X syndrome is a single well defined condition
࿿425࿿h in fragile X syndrome the triplet repeat does not change in size
significantly when passed from father to daughter
࿿426࿿h FISH analysis using multi telomeric probes diagnose about 25% of
nonspecific learning difficulties

࿿427࿿h Which of the following antigens is present on the erythrocytes of


people with O blood group?
࿿428࿿h B
࿿429࿿h None
࿿430࿿h mA
࿿431࿿h AB

67. what is the consequence of a nonsense mutation?


࿿432࿿h Deletes promoter region
࿿433࿿h codes for the wrong amino acid
࿿434࿿h introduces stop codon
࿿435࿿h prevents DNA replication

68. Which of the following diseases is characterized by the paternal genomic imprinting?
࿿436࿿h myotonic dystrophy
࿿437࿿h Angel man syndrome
࿿438࿿h prader willi syndrome
࿿439࿿h fragile X syndrome
69. which of the following are true of mother-fetus Rh incompatibility problem?
࿿440࿿h medical treatment can be nearly 100% effective in preventing such problems
࿿441࿿h all of the above
࿿442࿿h they can be prevented by injecting RH immune globulin into the Mother's blood system
࿿443࿿h they are much less likely to occur during the first pregnancy compared
to later pregnancies

࿿444࿿h Which of the following diseases is characterized by a dynamic mutation with


trinucleotide repeat amplification in the transcribed, but not in the translatedregion of the
gene?
࿿445࿿h Frederic ataxia?
࿿446࿿h fragile X syndrome
࿿447࿿h Huntington’s disease
࿿448࿿h Alzheimer's disease

71. which of the following definitions best describes the degreeof phenotypes expression?
࿿449࿿h Pleiotropy
࿿450࿿h Penetrance
࿿451࿿h Expressivity
࿿452࿿h incomplete penetrance

࿿453࿿h Which of the following type of inheritance is characterized by the


fact that the phenotypically normal members do not transmit the phenotype to
their children?
࿿454࿿h Autosomal recessive inheritance
࿿455࿿h X linked recessive inheritance
࿿456࿿h Genomic imprinting
࿿457࿿h autosomal dominant inheritance

࿿458࿿h which of the following type of inheritance is characterized by the fact


that the affected males with normal mates have no affected sons and no normal
daughters
࿿459࿿h Autosomal recessive
࿿460࿿h X linked recessive
࿿461࿿h autosomal dominant
࿿462࿿h X linked dominant

74. which of the following statements is true


࿿463࿿h cystic fibrosis and haemophilia and unlikely cam indicates for gene therapy
࿿464࿿h Neurofibromatosis Type 1(NF1) sometimes skips generations
࿿465࿿h In the CFTR gene a modifying intragenic polymorphism affect the phenotype?
࿿466࿿h cataracts can be a feature of NF1 but not NF2
75. If a frameshift mutation causes a stop codon to be inserted into the DNA sequence
࿿467࿿h the phenotype will change but not the genotype
࿿468࿿h the resulting protein will be too short and non-functional
࿿469࿿h the resulting protein will not be affected
࿿470࿿h the resulting protein will be too long and nonfunctional

࿿471࿿h what is the name for the nucleotides change / alteration that changes one
pyrimidine base with another pyrimidine base?
࿿472࿿h Transition
࿿473࿿h Transversion
࿿474࿿h Inversion
࿿475࿿h Deletion
࿿476࿿h Tay Sachs disease is inherited as an autosomal recessive,characterizes deficiency
of the enzyme hexosaminidase A. Symptoms include blindness and retardation. Onset of
symptoms begin at about 6 months of age and experience in early childhood NIT counselling at
of at-risk populations have reduced the frequency of the disease. Alicia’s parents are both
normalbut she had a sister who died after Tay Sachs.What is theprobability that Alicia is a carrier
of the Tay Sachs allele?
࿿477࿿h 0%
࿿478࿿h 50%
࿿479࿿h 25%
࿿480࿿h 75%

78. A single allele that controls more than one character is said to be
࿿481࿿h Pleiotrophic
࿿482࿿h Polygeneic
࿿483࿿h Linked
࿿484࿿h Autotrophic

79. Which of the following diseases is caused by a new mutation?


࿿485࿿h Polygenic
࿿486࿿h Sporadic
࿿487࿿h Monogenic
࿿488࿿h Multifactorial

80. A genetic disease that is either present or absent Is referred to as


࿿489࿿h Discordance
࿿490࿿h quantitative traits
࿿491࿿h qualitative traits
࿿492࿿h Concordance
࿿493࿿h phenylketonuria is an autosomal recessive disorder. Using P as a
normal and p as a recessive allele what is the genotype of a phenylketonuric
person?
࿿494࿿h Pp
࿿495࿿h Pp
࿿496࿿h pp
࿿497࿿h Pp or pp

82. which of the following statements about normal phenotypic variation is true?
࿿498࿿h most normal characteristics in bays are inherited from their fathers and most
common characteristics in girls are inherited from their mother
࿿499࿿h genes involved are the same in all populations
࿿500࿿h environmental factors play a role
࿿501࿿h most normal characteristics such as eye or hair colour are transmitted as
autosomal recessive trait

࿿502࿿h Several inherited disorders are much more common in close-knit religious
communities such as the Amish and Ashkenazi than in the general population this is at least
partly due to the fact that
࿿503࿿h Shared environmental conditions such as diet can increase mutation rate
࿿504࿿h modern medical care is not widely available in such community
࿿505࿿h community members care for each other and such disorders a contagious
࿿506࿿h people in such communities are more likely to marry relatives

࿿507࿿h Which of the following type of inheritance is characterized by the fact


that the recurrence risk for each sib of the proband is 1 in 4?
࿿508࿿h autosomal dominant
࿿509࿿h autosomal recessive
࿿510࿿h Y linked
࿿511࿿h X linked

࿿512࿿h Population carries 2 or more different alleles at a particular locus.


Which of the following definitions best corresponds to the above mentioned
statement?
࿿513࿿h Compound
࿿514࿿h Heterozygote
࿿515࿿h Polymorphism
࿿516࿿h Homozygote

86. which of the following is a characteristic to the expansion of unstable repeat sequences?
࿿517࿿h Mosaicism
࿿518࿿h Anticipation
࿿519࿿h Locus heterogeneity
࿿520࿿h Haploinsufficiency
࿿521࿿h Which of the following definitions best corresponds to the probability that a
certain gene will be Expressed?
࿿522࿿h penetrance
࿿523࿿h Expressivity
࿿524࿿h pleiotropy
࿿525࿿h Variable Expressivity

88. Which of the following statements about mitochondrial inheritance is true?


࿿526࿿h mitochondrial conditions only affect muscle and nerve tissue
࿿527࿿h mitochondrial genes mutate more often than nuclear genes
࿿528࿿h mitochondrial diseases have nothing to do with nuclear genes
࿿529࿿h Hetereoplasmy refers to the presence of more than one mutation in mitochondria

࿿530࿿h in human’s relationship between the A and B alleles of the ABO blood
group gene is an example of
࿿531࿿h Complete dominance
࿿532࿿h Codominance
࿿533࿿h Incomplete dominance
࿿534࿿h Epistasis

࿿535࿿h A famous American folk singer Woody Guthrie died of Huntington's Disease
in 1967. Which statement below must be true?
࿿536࿿h At least one of Woody Guthrie’s parents must have had the Huntington's Disease
also
࿿537࿿h when only one member of the pair of relatives is affected and another is
not the relatives are
࿿538࿿h Discordant

࿿539࿿h the frequency of expression of a phenotype is less than 100%-that is when


some of those who have the appropriate genotype completely fail to express it-the gene
is said to show
࿿540࿿h Expressivity
࿿541࿿h reduced penetrance
࿿542࿿h Penetrance
࿿543࿿h Variable Expressivity
93. substitution of one or the other of two bases at one location is referred to as
࿿544࿿h short tandem repeat polymorphism STRP
࿿545࿿h single nucleotide polymorphism SNP
࿿546࿿h Simple
࿿547࿿h Indel

94. Which of the following definitions best describes the location of a gene in a chromosome?
࿿548࿿h Locus
࿿549࿿h Site
࿿550࿿h Promoter
࿿551࿿h Primer

95. couples who have one or more and ancestors in common are called
࿿552࿿h Kindred
࿿553࿿h Pedigree
࿿554࿿h Sibship
࿿555࿿h Consanguineous

96. Which of the following factors is characteristic to hemochromatosis?


࿿556࿿h Incomplete penetrance and variable expressivity
࿿557࿿h Only variable expressivity
࿿558࿿h Age dependent expressivity
࿿559࿿h Only incomplete penetrance

-IN A PARALLEL WORLD-

࿿560࿿h If a frameshift mutation causes a stop codon to be inserted into the DNA
sequence:
࿿561࿿h The resulting protein will not be affected
࿿562࿿h The resulting protein will be too short and non-functional
࿿563࿿h The phenotype will change but not the genotype
࿿564࿿h The resulting protein will be too long and non-functional

97. A frameshift mutation


࿿565࿿h Joins two different proteins
࿿566࿿h Replaces one amino acid with another
࿿567࿿h Removes part of the protein
࿿568࿿h introduces a section of amino acids not normally found
98. Which of the following statements about normal phenotypic variation is true?
࿿569࿿h Most normal characteristics in bays are inherited from their fathers
and most normal characteristics in girls are inherited from their mothers
࿿570࿿h The genes involved are the same in all populations
࿿571࿿h Environmental factors play a role
࿿572࿿h Most normal characteristics such as eye or hair color are
transmitted as autosomal recessive trait

࿿573࿿h Select the most closely associated kind of genetic disease, when there is a
mechanism responsible for most common diseases of adulthood, such as coronary artery
disease and non- insulin-dependent diabetes
࿿574࿿h Autosomal recessive
࿿575࿿h Autosomal dominant
࿿576࿿h X-Linked recessive
࿿577࿿h Multifactorial inheritance

100. Which of the following diseases is caused by a new mutation?


࿿578࿿h Sporadic
࿿579࿿h Monogenic
࿿580࿿h Multifactorial
࿿581࿿h Polygenic
sha la la la laa!
1. The Human haploid Genome is composed of approximately

3 billion base pairs of DNA and 20,000 protein coding genes


3 million base pairs of DNA and 20,000 protein coding genes
6 billion base pairs of DNA and 20,000 protein coding genes
6 million base pairs of DNA and 2000 protein coding genes

࿿582࿿h Which of the following units corresponding to the 1 thousand the


nucleotides base pairs in the genomic DNA

Microbase
Kilobase
࿿583࿿h Millibase
࿿584࿿h megabase
࿿585࿿h By which of the following bonds are DNA nucleotides of same strand connect
with each other hydrogen bonds
peptide bonds
hydrophobic bonds
sugar phosphate bonds

࿿586࿿h what is the complementary strand for the DNA of the following RNA
strand: 5’ AGCCAUUUGCACG 3’?

3’ CGUGCUUUGGCU 5’
3’ ATGCGTTTACCGA 3’
5’ CGUGCAAUGGCU 3’
3’ TCGGTTTCGTGC 5’

࿿587࿿h DNA exists in a double standard form whereas RNA is mainly a single
stranded molecule. What is the likely reason for DNA being double stranded?

DNA cannot exist in the single standard form


double-stranded DNA is more stable structure
RNA strands cannot form base pairs
it is easier to replicate double-stranded DNA when single stranded RNA
࿿588࿿h In which stage of the meiotic division does crossing over take place?

I Prophase
I Anaphase
II Prophase
II anaphase

࿿589࿿h During telophase all of the above mentioned takes place EXCEPT:

DNA is duplicated
the chromosomes become less dense and more thread like
new nuclear envelopes form
chromosomes are released from the microtubules

࿿590࿿h after which of the following divisions will there be 23 chromosomes and 46
chromatids in the cell?

I meiotic division
Mitosis
II meiotic division
amitosis

࿿591࿿h an important and distinct feature of meiosis

spindle formation
chromosome condensation
chromosome congression
Recombination

࿿592࿿h During the process of oogenesis

only one ovum is produced


two ovum and two polar bodies are produced
two polar bodies are produced
4 ovum are produced

࿿593࿿h if at the end of meiosis the daughter cells has 12 chromosomes


how many chromosomes were in the mother

3
6
12
24
࿿594࿿h which of the following statements is true regarding okazaki fragments

they are synthesized by ligase


there are short fragments of RNA joined by DNA ligase
They add nucleotides to the elongating DNA
they are short fragments of DNA joined by DNA ligase

࿿595࿿h when examining genetic code it is Apparent that

there can be more than one codon for a particular amino acid
AUG is a terminating codon
there can be more than one amino acid for a particular code
The code is ambiguous in that same codon can code for two or more amino acids

࿿596࿿h which one of the following makes use of RNA template to synthesise DNA

DNA polymerase
RNA polymerase
reverse transcriptase
DNA dependent RNA polymerase

࿿597࿿h which statement about transcription is true

describe the production of polypeptides from mRNA template


Precedes 5’ capping and polyadenylation
is regulated by transcription factors that binds to the 3’ UTR
producers single stranded mRNA using the sense DNA strand as a template

࿿598࿿h which statement about base substitution is true

cannot affect splicing


are always pathogenic
can affect gene expression
Result in frameshift mutations

࿿599࿿h during Protein synthesis an anticodon on tRNA Pairs with

DNA nucleotide bases


r RNA nucleotides bases
m RNA nucleotides bases
Other t RNA nucleotides bases
࿿600࿿h which of the following processes take part in the nucleus of a cell

Translation
post translational processing
RNA processing
binding of amino acids with Transport RNA

࿿601࿿h which of the following unprocessed RNA transcript of a gene


colinears with genomic DNA And contains introns as well exons

t-RNA
r-RNA
Mature RNA
primary RNA

࿿602࿿h What effect can methylation of CpG islands in the promoter region have on gene
expression

Transcription level will be reduced


transcription level is sharply increased
it will affect translation
it will affect splicing process

࿿603࿿h during somatic rearrangement which part of the DNA is excised

Exons
Introns
gene segments
promoter sequence

࿿604࿿h which of the following types of histones do not constitute an octamere

H1
H2A
H3
H4

࿿605࿿h Which of the following fundamental unit of chromatin organisation?

Matrix
Scaffold
Solenoid
Histone
࿿606࿿h Which of the following contains protein encoding genes?

Heterochromatin
centromeric regions
Telomeric regions
Euchromatin

࿿607࿿h Approximately 5% of the cytosine Residue are methylated in the


genome of any given eukaryote. In what way is DNA methylation related to genetic
regulation

There is an inverse relationship between the degree of methylation of DNA and


the degree of gene expression
methylation of DNA promotes upregulation of gene expression
to the methylated regions of DNA specific transcription factor protein attached
himesha d ko? --- mata type karanna amathaka una - himesha

࿿608࿿h Which of the following elements are present in the processed pseudogenes?

Exons
Introns
Anticodons
Enhances

࿿609࿿h which is not true regarding variable number tandem repeats (VNTRs) ?

they are about 15 to 100 base pairs long


they are transposable elements
they are found within and between genes
they are useful for identification of individuals

࿿610࿿h which of the following is not an outcome of posttranslational modification of a


protein?

removal of N-terminus amino acid


removal of introns
addition of phosphate groups
addition of metals to create tertiary or quaternary structures

࿿611࿿h which of the following does not regulate normal gene expression

Enhancer
Silencer
Promoter
“ cap”
࿿612࿿h the mechanism of silencing of genes through epigenetics occurs

specifically at (C) in RNA located in cpg Islands


specifically at cytosine in DNA located in cpg Islands
specifically at cytosine in DNA located in areas where strings of C are located
specifically on amino acids on histone tails

࿿613࿿h why are male mutation rates higher than female

greater number of germ cell divisions


more cells so more DNA
The Y chromosome is unstable
men have only one x chromosome

࿿614࿿h What are two modular elements that appear as consensus sequences
upstream from RNA polymerase if transcription start sites

Microsatellites and transposons


r Dna and nucleolar organisers
TATA and CAAT
TTAA and CCTT

࿿615࿿h Which of the following would result in the highest rate of mutation

࿿616࿿h small genes, few hot spots, old age


࿿617࿿h large genes, many Hotspots, old age
࿿618࿿h small genes, many Hotspots, young age
࿿619࿿h large genes, few hot spots, all age

࿿620࿿h transposable sequences fall in to unique group of repetitive DNA


sequences that are identified by their ability to

stop viruses from infecting a cell


move from place to place within the genome
interfere with telomere function
prevent proper chromosome segregation during meiosis
࿿621࿿h which of the following illustrates a single point mutation on a segment of
DNA which reads TACACGGTG?

TACACGCGCTG
TTCACGCGAG
TAGACGGTC
TACAUGTG

࿿622࿿h Frameshift mutation is one of the most severe types of mutations because

more than one gene is affected


they occur only in gametes
translation is stopped
more than one amino acid or entire proteins are affected

࿿623࿿h what is the consequence of a nonsense mutation

prevents DNA replication


introduces a stop codon
delete promoter region
codes for the wrong amino acid

࿿624࿿h mutation is defined as

change in organisms DNA sequence


the growth of an abnormal cell structure
the changing of a cell from one type to another
a way of changing mRNA proteins

࿿625࿿h when the number of bases involved in in is not a multiple of three ( i.e is
not an integral number of codons) and when it occurs in a coding sequence the reading
frame is altered beginning at the point of the insertion or deletion

frameshift mutation
silence mutation
Nonsense mutation
missense mutation
࿿626࿿h what is the name of the sequence that is most common in a population

private sequence
variant sequence
reference sequence
all of the above

࿿627࿿h which of the following is not true about indels

the number of repeats correspond to the number of alleles


Indels are never inherited
there are hundreds to thousands of indels in the genome
their DNA sequences may vary from 2 pb to 200 bp

࿿628࿿h the mutation causing Sickle Cell disease is GAG → GTG in the Beta -
globin gene. This is an example of
frameshift mutation
missense mutation
nonsense mutation
dynamic mutation

࿿629࿿h which of the following statements regarding spontaneous mutation rate isfalse

the spontaneous mutation rate cannot be measured in complex animals such as


mammals
the spontaneous mutation rate where is considerably among different organisms
the spontaneous mutation rate is very high in some organisms but they have efficient
repair system
the spontaneous mutation rate varies from gene to gene within an organism

࿿630࿿h In a case when two related individuals in a family have the same disease
they are said to be

Concordant
Discordant
quantitative traits
qualitative traits
࿿631࿿h the polymorphism is

any change in the DNA sequence


variation of genomic sequence present in less than 1% of the population
the least common variation of gene or marker sequence
the most common variation of a gene or marker sequence

࿿632࿿h An Rh “ - “ female gave birth to an Rh “+” child, she was not injected Rh
immune globulin after the delivery. Her husband is Rh “+” . will she have problems with
her second pregnancy ( remember Rh “-” is a recessive trait)

Yes we will definitely have problems


it depends on the gender of the conceived child
no she will not have problems
maybe depending upon the father's phenotype

࿿633࿿h Which of the following biomolecule has self repair mechanism

DNA only
DNA and RNA
DNA RNA and proteins
DNA and proteins

࿿634࿿h which polymorphism is represented with multiple allele and use for identity
testing

STRs
SNPs
LINE-1
CNVs

࿿635࿿h Which of the following is directly involved in DNA repair

DNA polymerases
Ribosomes
Topoisomerase
Telomerase

࿿636࿿h what type of enzyme removes damaged DNA from the rest of the DNA molecule

Polymerase
Nuclear
Ligase
Primase
࿿637࿿h an individuals ABO blood type is normally determined by

the inheritance of o1 of 3 possible alleles (A, B or O) from each parent


environmental influences alone
genetic inheritance and environmental influences during life
whether mother has been injected by RhIG during the pregnancy or not

࿿638࿿h Galactosemia is an autosomal recessive disorder. Symptoms include failure


to thrive, vomiting, jaundice,hepatomegaly, and cataracts. It is most commonly due to a
deficiency of galactose-phosphate uridyltransferase(GALT).A man and woman,who are both
known to be carriers for galactosemia, marry and have a child. To their relief. the standard
newborn screening test results, which measures the blood level of galactose and galactose-
l-P, shows that their child is affected. Based upon this information, what is the probability that
the child is normal phenotypically?
0%
25%
50%
75%

࿿639࿿h Red green colour blindness also known as daltonism is a relatively benign
condition the gene responsible for this condition is located on the X chromosome and is
inherited as recessive trait approximately 7 - 0% of men 0.5 - 1% of women are
affectedWhat is the chance that a daltonic father and a normal mother will produce a
colorblind son?

0%
25%
50%
100%

࿿640࿿h Robin is affected by an autosomal dominant disorderinherited from her mother;


she is married to Ched who is unaffected and has no history of the disorder in his family.
Robin and child have two unaffected children studies suggest that for every hundred
individuals who inherited mutations in the chain of interest only 50 actually show symptoms,
the new mutation rate for this disorder is essentially zero. based on this what is the probability
that the next child will be present with the clinical signs of the disease?

¾
½
¼

࿿641࿿h Tay-Sachs disease is inherited as an autosomal recessive, deficiency of the
enzyme hexosaminidase A, Symptoms include blindness and retardation. Onset of symptoms
begins at six months of age, and death results in early childhood. What is the probability that
homozygous normal man and a carrier female will have a child Tay-Sachs disease?

0%
25%
50%
75%

࿿642࿿h Haemophilia a is a disease in which patients like a clotting factor in the blood
(factor VIII) it is inherited as an x-linked recessive trait. What is the probability of ahemophilic
male and carrier femaleproducing a hemophilic son?

࿿643࿿h 0%
࿿644࿿h 25%
࿿645࿿h 50%
࿿646࿿h 75%

࿿647࿿h Which of the following definitions best describes a disease caused by


one or two mutant alleles?
Multifactorial
Polygenic
Chromosomal
Monogenic

࿿648࿿h the presence in an individual or tissue of at least two cells lines that differ
genetically but are derived from a single zygote is referred to as

Imprinting
trinucleotide repeat expansion
Mosaicism
mitochondrial inheritance

࿿649࿿h which of the following definitions best corresponds to the disease that is
caused by different types of mutations in the same Locus

Genetic heterogeneity
phenotypic heterogeneity
locus heterogeneity
Allelic Heterogeneity
࿿650࿿h Which of the following definitions best corresponds to an individual
genotype which is expressed in its morphological characteristics

Phenotype
Homozygote
Heterozygote
Genotype

࿿651࿿h ethnic differences in disease frequencies are most Apparent for

autosomal dominant conditions


autosomal recessive conditions
X linked recessive conditions
sex chromosome aneuploidies

࿿652࿿h the frequency of expression of a phenotype is less than 100%, that is when
some of those who have the appropriate genotype completely failed to express it, the Gene
is said to show

Penetrance
Expressivity
reduced penetrance
variable Expressivity

࿿653࿿h In 1991 it was discovered that the fragile X syndrome was caused by
mutation in the fragile X mental retardation 1(FMR-1 ) gene. an area of CGG trinucleotide
repeats just upstream of the coding area was found to be variable in size. all the following
statements regarding the FMR1 gene are true EXCEPT:

“Premutations” may expand to full mutations in future generations


Individuals with premutation will also have intellectual disability
offspring of female Carriers main inherit a premutation or a full mutation
offspring of Mel carriers in heretic premutation

࿿654࿿h which of the following is true about incomplete dominance

the hybrid shows a phenotype that is intermediate between two homozygotes


the dominant allele is expressed only in heterozygotes
the hybrid expresses the traits carried by both Alleles
patients homozygous and heterozygous for a mutant allele will have equally severe
disease
࿿655࿿h Nearly every individual affected with neurofibromatosis Type 1(NF1) exhibits
clinical symptoms some however may present with cafe au lait spots with lisch nodules while
others have life threatening tumors surrounding the spinal cord, these represent an example
of which of the following

Locus heterogeneity
Reduced penetrance
Variable Expressivity
sex influenced expression

࿿656࿿h The expression of the disease phenotype depends on whether the mutant
allele has been inherited from the father or from the mother, a phenomenon known as

X linked recessive inheritance


autosomal recessive
autosomal dominant
genomic imprinting

࿿657࿿h which of the following is characteristic to the mitochondrial type of inheritance

the disease more often in males


only females are affected
all children of the affected males are also affected
the diseases transmitted only maternally

࿿658࿿h the disease appears to develop at an earlier age when it is transmitted


through the Pedigree, a phenomenon referred to as

genomic imprinting
Anticipation
autosomal dominant
X linked recessive

࿿659࿿h which of the following diseases are not characterized by anticipation

Alzheimer's disease
myotonic dystrophy
Huntington's disease
fragile X syndrome
࿿660࿿h which of the following is true about Inheritance of complex diseases

it appears in every second generation of the family


concordant rate is same in both monozygotic and dizygotic twins
the disease is more common in close relatives of the Proband than in far relatives
d ko? deeko ?- dilina, amathaka una - himesha ,ehema baane - kaveesha

࿿661࿿h which of the following statements is true about polygenic trait

several different genes influence many traits


one gene influences many traits
it is more common for the older twin to have the trait
several alleles exists for one trait

࿿662࿿h which of the following is true of a Concordant trait between twins?,


Sharing is caring - Dinethra )
࿿663࿿h it is more common for males to have the trait
࿿664࿿h it is more common for the females to have the trait
࿿665࿿h it is more common for the old twins to have the trait
࿿666࿿h bothsharethetrait
LGBTQ
࿿667࿿h which of the following is true about cleft lip/palate (CL/P)?

most of the patients with cleft lip/palate are males


it causes mental retardation
syndromic cleft lip/palate is always inherited as a single gene disorder
the isolated cases of CL/P the recurrence risk in relatives does not always correlate
with probands severity

࿿668࿿h Which of the following is true about conditions that show a multifactorial
inheritance pattern

the recurrence risk is lower if more than one family member is affected
if the expression of the disease is in the proband is more severe the recurrence is lower
the recurrence risk is higher if the proband and is of the less commonly affected sex
the recurrence risk for the disease is quite high even in remotely related relatives
࿿669࿿h which of the following is true about modifier gene

It is a gene that, according to the demands of the organism, increases or decreases the
function of a non allelic gene - himesha kiwwe A kiyala
it is a nonallelic gene that changes the manifestation of another gene- parami thama
kiwwe
it is a non allelic gene that enhances the function of another gene when certain enzymes
are present
it is a non allelic gene that inhibits the function of another gene when certain enzymes
are absent

࿿670࿿h which statement is true concerning Alzheimer's disease patients

E4 allele increases the risk of developing and sinus disease but it does not cause the
disease
all people who have E4 allele will develop the disease
all people with Alzheimer's disease have the E4 allele
everyone who carries the APOE gene will develop Alzheimer's disease

࿿671࿿h which of the following may be a cause of cerebral vein thrombosis?

mutation of the prothrombin FVL genes at the same time


Mutation of prothrombin gene and use of oral contraceptives
mutation of FVL gene and use of oral contraceptives
any of the above

࿿672࿿h which of the following statements about venous thrombosis is true

factor v leiden and the Prothrombin G20210A Variant are common causes of venous
thrombosis
testing for factor v leiden and prothrombin variant will change the management of a
patient with venous thrombosis
Males are five times more likely to be affected with venous thrombosis than females
the factor V leiden mutation causes reduced expression of the factor V gene

࿿673࿿h an 80 year old man presented with impairment of higher intellectual function in
alterations in mood and behaviour, his family reported progressive disorientation and
memory loss over the last six months there is no family history of dementia. the patient was
tentatively diagnosed with Alzheimer's disease. which one of the following best describes
Alzheimer's disease?

it is caused by the infectious beta sheet form of host-cell protection


It is associated with the deposition of neurotoxic amyloid Beta peptide aggregates
it is associated with the accumulation of precursor protein
It is an environmentally reproduce disease not influenced by the Genetics of the
individual

࿿674࿿h this type of acrocentric chromosome has ______ attach to their short arms by
stalks

Satellites
fragile sites
LINE sequences
ALU sequences

࿿675࿿h to be stable, a rearranged chromosome must have which of the following

no inverted segments
balance composition ( no losses or gains of genetic material)
Centromere
centromere and two telomeres

࿿676࿿h which process is primarily responsible for aneuploidy

Meiotic nondisjunction
unequal crossing over
formation of barr bodies
errors during replication

࿿677࿿h short arm of acrocentric chromosomes contain

t-RNA genes
r-RNA genes
Telomerase genes
None of the above

࿿678࿿h Which of the following would be most likely to produce an


imbalance in the amount of essential genetic material in the carrier?

robertsonian translocation
Isochromosome
reciprocal translocation
Pericentric inversion
࿿679࿿h in which phase of mitosis can high quality G or R banding of the
chromosome be accomplished

Interphase
Metaphase
Anaphase
Telophase

࿿680࿿h which of the following is characterized by the junction of two acrocentric


chromosomes near centromere with the deletion of short terms

Deletion
robertsonian translocation
reciprocal translocation
Duplication

࿿681࿿h which of the following types of chromosomes corresponds to the dicentric


chromosome

Duplicated
Homological
Abnormal
Diploid

࿿682࿿h Which of the following does not cause down syndrome

robertsonian translocation between 21q and chromosome 14


robertsonian translocation between 21q and chromosome 22
isochromosome 21q21q
reciprocal translocation between 21q and chromosome 15

࿿683࿿h Which of the following is chromosomal mutation

Aneuploidy
Triploidy
Inversion
Polyploidy
࿿684࿿h fluorescence in situ hybridization using whole chromosome(painting) or
specific locus probes enables routine detection of

gene expression
Trisomy
gene amplification
Triploidy

࿿685࿿h which method is used to detect relative copy number gains and
losses in a genome-wide manner by hybridization

G banding
fluorescent in situ hybridization (FISH)
CGH array
allele specific oligonucleotide hybridization

࿿686࿿h dosage compensation in mammals typically involves the random inactivation


of one of the two X chromosomes collectively in early development, in such X chromosome
inactivation involve

FMRI gene
SOX gene
SRY gene
XIST gene

࿿687࿿h Marcus and carmen have a child with ambiguous external genitalia, the
internal genitalia of female and chromosome analysis is 46xx, biochemical studies
revealed A deficiency of 21 hydroxylase, what is the most likely diagnosis

Androgen insensitivity
Camptomelic dysplasia
congenital adrenal hyperplasia
Turner syndrome

࿿688࿿h you have sent a blood sample of dysmorphic baby to the laboratory for
chromosomal analysis , the answer from the lab says that the baby 46,XY karyotype,
del(18)(q12).What is the reason the baby's parents should also do a blood analysis

to determine whether anomaly is acquired and inherited


to determine whether anomaly is paternal
to determine whether anomaly is maternal
to determine if there is a family history of the disease
࿿689࿿h which of the following statement is correct

X chromosome inactivation, all the genes of one X are switched off


female fetal development is solely dependent on normal functioning of SRY gene
male fetal development is solely dependent on normal functioning after SRY gene
females are mosaic with respect to X linked gene expression

࿿690࿿h which of the following statement is correct

X chromosome inactivation, all the genes of one X are switched off


Male fetal development is Solely depend on normal functioning of SRY gene
female fetal development is solely dependent on normal functioning of SRY gene
X chromosome inactivation may be linked to discordance in monozygotic twins Pairs

࿿691࿿h Where are pseudoautosomal regions(PAR) located on

PAR1 is located at the distal part and the PAR2 is located near the centromere of both X
and Y chromosomes
PAR1 and PAR2 are located close to the centromeres of both X and Y chromosomes
PAR1 and PAR2 are located on the distal parts of both arms of X and Y chromosome
PAR1 Is located near the centromere and PAR2 is located at the distal part of both X
and Y chromosome

࿿692࿿h which is not a feature of klinefelter syndrome

Gynecomastia
short height
long, thin fingers, arms and legs
small testis

࿿693࿿h which of the following is true about x inactivation

Embryos with 2 active X can survive but will be deceased


Embryos with 2 Active X cannot survive
Embryos with 2 inactive X can survive but will be deceased
Embryos with 2 Active X can survive and will be totally normal

࿿694࿿h When is barr body formed

Early embryogenesis
during spermatogenesis
during oogenesis
after birth
SET 1 NEW-1
Friday, July 17, 2020 1:57 AM

࿿695࿿h Which of the following is a "sense" strand of the DNA?


← Translated template;
← Nontranscribed template,
← Transcribed template:
← Nontranslated template.

࿿696࿿h The somatic cells of human males contain:


← 23 pairs of autosomes and 2 X chromosomes
← 22 pairs of autosomes, a Y chromosome and an X chromosomes
← 23 pairs of autosomes, a Y chromosome and an X chromosome.
← 22 pairs of autosomes and 2 X chromosomes,

࿿697࿿h Which DOES NOT describe DNA replication in eukaryotes?


← Replication is semiconservative:
← One new strand undergoes continuous synthesis, whereas the other undergoes discontinuous synthesis
← Replication begins at a single origin for each DNA molecule in eukaryotes;
← The replication fork is the site of strand separation

࿿698࿿h The 5’ and 3’ numbers are related to the


← length of the DNA strand
← Carbon rings in sugar
← The number of phosphates
← The base pair rule

࿿699࿿h Which describes restriction enzyme?


← They form covalent bonds between DNA fragments
← They are creations of genetic engineering and do not exist naturally
← Each act only on specific nucleotide sequences
← They remove introns

࿿700࿿h Which statement about DNA replication is true


DNA is synthesized in a conservative manner
DNA is synthesized in one direction
Uracil is inserted to pair with adenine

Set 1 new 1 Page 1


d. DNA helicase separates the double stranded DNA

࿿701࿿h How many times does the cell go through prophase during meiosis I and II?
Once only goes through prophase during meiosis I
Twice it goes through prophase during meiosis I and II
← Once -It only goes through prophase during meiosis I
←Twice - It goes through prophase during meiosis I and interphase

࿿702࿿h Which of the following stages is characterized by the replication of DNA?


Stage between two meiotic divisions
Metaphase of mitosis
Prophase of mitosis
Stage before I meiotic division

࿿703࿿h Which of the following processes take part in a nucleus of a cell?


Peptide bonds between amino acids
Binding of amino acids with transport RNA

࿿704࿿h The stage of the cell cycle where the cell is preparing to begin DNA replication is called

M
S
G2
G1
࿿705࿿h Which statement about base substitution is true?
← result in frameshift mutations
← cannot affect splicing
← May result in nonsense mutations
← Are always pathogenic
࿿706࿿h How many base pairs of DNA are there in human genome?
← 30 billion bp
← 3 billion bp
← 25000 bp
← Several thousand bp
࿿707࿿h one of the most useful methods for identifying a specific gene is
← Magnetic resonance imaging
← Eastern blot
← Western blot
← Southern blot
࿿708࿿h The following are directly involved in DNA repair
← Ribosomes
← Telomerases
← Splicing

Set 1 new 1 Page 2


← Ligases
࿿709࿿h which of the following processes take part in the cytoplasm of the cell?
← RNA processing
← Binding of amino acids with transport RNA
← Transcription
← Ligases
࿿710࿿h Okasaki fragments occur during
← Synthesis
← Replication
← Transformation
← Polymerase reaction
࿿711࿿h which of the following does asexual reproduction involve?
← Gamete formation
← fertilization
← Binary Fission
← meiosis
࿿712࿿h Which of the following describes the process of formation of phosphodiester bonds, when enzyme
binds two double stranded DNA sequences together?
← Restriction
← Hybridization
← Ligation
← Amplification

࿿713࿿h How is meiosis I reason from meiosis II?


← Meiosis I separates homologous chromatids, meiosis II separates sister chromatids
← Meiosis I separates homologous chromosomes, meiosis II separates sister chromatids
← Meiosis I separates sister chromatids, meiosis II separates

࿿714࿿h A diploid cell with 24 chrornosomes undergoes meiosis. It produces daughter cells with:
← 12 chromosomes each containing two chromatids
← 24 chromosomes each containing two chromatids
← 12 chromosomes each containing a single molecule of DNA
← 24 chromosomes each contatntng a single molecule of DNA

Which of the following are present in the processed pseudogenes?


← Exons
← Enhancers
← Introns
← Anticodons

࿿715࿿h Which of following undergoes posttranslational processing?


mRNA:
t-RNA:
DNA
Protein

࿿716࿿h Every polypeptide chain formed in translation starts with the amino acid:

Set 1 new 1 Page 3


← Alanine.
← Serine:
← Lysine:
← Methionine

࿿717࿿h All fragments cut by most restriction endonucleases have


← Complementary double-stranded ends;
← Double-stranded "sticky" ends:
← Comeplementary single stranded ends
← Double-stranded supplementary ends

࿿718࿿h folloqing statements about gametogenesis are all true EXCEPT:


← Four sperm ceas are produced from each diploid cell;
← Oogenesis produces one ovum and one polar body
← Sperm cells are produced during spermatogenesis:
← Polar bodies get onty a tiny amount of cytoplasm.

࿿719࿿h Which Of the following is true about formation of processed pseudogenes?


← They are formed by translocation:
← They are byproducts of evolution
← They are formed by retrotransposition
← They are formed by point mutation.

࿿720࿿h How many Chromosomes and chromatids are there in each human cell at the end of Il rneiotic
division?
← 23 and 23
← 23 and 46.
← 46 and 23:
← 46 and 46;

࿿721࿿h Which of the following refers to the autosomes?


Any chromosomes:
← Any Chromosome except the 22-th chromosome:
← Any sex chromosome:
← Any chromosome except sex chromosome

࿿722࿿h Transcription is the process of making a complementary RNA strand from a DNA strand. In
eukaryotic cells. where does this process take place?
← Smoothendoplasmic reticulum;
← Nucleus
← Cytoplasm;
← Ribosomes.
࿿723࿿h At which stage of mitosis chromosomes are best seen for analyses:
← Metaphase
← Anaphase;
← Prophase;
← Telophase.

࿿724࿿h Meiosis differs from mitosis in the following ways:


Meiosis is restricted to the gametes and mitosis occurs only in somatic cells
he prophase stage of mitosis is 1 step, in meiosis I there are 4 stages;
The prophase stage of mitosis is 1 step, in meiosis I there are 3 stages.
Daughter cells are haploid, not diploid

࿿725࿿h The cell that receives "foreign" DNA is called:


Sense strand.
Host cell
Donor cell;
Recombinant DNA;

࿿726࿿h Which of the following takes place during somatic reconstruction?


Set 1 new 1 Page 4
RNA splicing:
A new reconstruction of genes in the cells
RNA processing:
Posttranslational processing

࿿727࿿h Which of the are two oligonucleotides. One on each Side of a target sequence. designed so that
them is complementary to a segment of DNA on one strand of a double-stranded DNA molecule and the
other complementary to a segment of DNA on the other strand? A specific pair of oligonucleotides serves to
initiate Synthesis DNA in a PCR reaction.
Primers:
palindromes
Cornplementary DNAs:
Vectors.

࿿728࿿h When is the chemical "cap" added to the RNA?


During transcription,
After transcription
During translation
During splicing:

࿿729࿿h Which of the following is NOT true about mitosis?


allows growth and differentiation of somatic cells and effects tissue regeneration;
Results In two daughter cells with identical information;
Involves Ordinary somatic cell division
Results in the formation of reproductive celsls (gametes):

࿿730࿿h How many types probes are used for the stainig of metaphase chromosomes during SKY?
24
14.
23,
46

࿿731࿿h The first step the polymerase chain reaction (PCR) is


Denaturation
Annealing;
Primer extension;
Cooling.

࿿732࿿h Fluorescence In Situ hybridization using whole chromosome (painting ) or specific locus probes
enables routine detection of;
Triploidy
Gene expression:
Gene amplification
Reciprocal translocation

࿿733࿿h A Promotor is;


Stop signal for transcription
Start signal for transcription
Binding site for DNA Polymerase:
None of these.

࿿734࿿h What is the function of DNA polymerase?


It attaches the RNA primers to the DNA strand:
It seals the Okazaki fragments.
It unwinds the DNA
It adds single nucleotides to the growing DNA strand

࿿735࿿h Which of the following is true about Single-stranded proteins:


Attaches a small RNA primer to the single stranded DNA to acts as a substitute 3-OH for DNA polymerase to
Begin synthesis from

Set 1 new 1 Page 5


Bind to the single-stranded DNA and prevent supercoiling of the double helix:
Bind to the Single-stranded DNA and keep them separate in order to mantain stability Of the replication fork
Monitor the speed of replication

࿿736࿿h Which of the following processes occurs meiosis ll ?


Submetacentric and acrocentric chromosomes undergo recombination
Homologous chromosomes segregate to opposite poles .
Sister chromatids of the homologous chromosomes separate
DNA pairing is initiated, leading to the formation of the synaptonemal complex

࿿737࿿h Which Of the following statements about mitochondrial disorders is true:


All follow patrilinear inheritance.
All follow matrilinear inheritance
There are fewer than 50 gene products from the mitochondrial genome:
Leigh disease is always Caused by the same point mutation;

࿿738࿿h Which of the following types of inheritance best corresponds to the statement: an affected male
and a normal female have all affected daughters and healthy sons?
Y-linked.
X-linked dominant
Autosomal-recessive.
Autosomal-dominant;

࿿739࿿h A single nucleotide substitution (or point mutation) in a DNA sequence can alter the code in a triplet of
bases
and cause the replacement of one amino acid by another in the gene product.
Silence mutations:
Frameshift mutation
Nonsense mutations
Missense mutation

࿿740࿿h Which of the following nucleotide changes/alterations can cause the change of a purine base with
a pirimidine base or vise versa?
Deletion;
Transition
Transversion
Inversion.

࿿741࿿h Galactosemia is an autosomal recessive disorder. Symptoms include failure to thrive, vomiting,
jaundice, hepatomegaly, and cataracts. It is most commonly due to a deficiency of galactose-phosphate
uridyltransferase (GALT). A man and woman, who are both known to be carriers for galactosemia, marry and have a
child. To their relief. the standard newborn screening test results, which measures the blood level of galactose and
galactose -l-P, shows that their child is affected. Based upon this information, what is the probability that the child
has the disease?

0%
25%
50%
75%

࿿742࿿h Tay-Sachs disease is inherited as an autosomal recessive, deficiency of the enzyme


hexosaminidase A, Symptoms include blindness and retardation. Onset of symptoms begins at six months
of age, and death results in early childhood. Genetic counselling of at-risk populations (Ashkenazi Jews and
French Canadians in Quebec) has reduced the frequency of the disease, Alicia's parents both are normal, but had
a sister who died of Tay-Sachs. What is the probability that Alicia is a carrier of the Tay-Sachs allele?
50%
25%
0%
75%

࿿743࿿h What is a striking disorder that occurs nearly exclusively in females and meets all criteria for being
an X -Iinked dominant disorder that is usually lethal in hemizygous males
Set 1 new 1 Page 6
࿿744࿿h Fragile X syndrome:
࿿745࿿h Patau syndrome:
࿿746࿿h Down syndrorne:
࿿747࿿h Rett syndrome

← Which of the following type of inheritance is characterized by the fact, that the Parents of an affected child
are asymptomatic carriers of mutant alleles?
Autosomal Dominant Inheritance.
Autosomal Recessive inheritance,
Genomic Imprinting
X-Linked Recessive Inheritance.

← The incidence of the trait is much higher in males than in females


Autosomal Recessive inheritance,
Autosomal Dominant Inheritance,
X Linked Recessive Inheritance;
Genornic imprinting.

← The frequency of expression of a phenotype is less than 100%-that is, when some of those who have the
appropriate genotype completely fail to express it-the gene is said to show
࿿748࿿h Variable expressivity.
࿿749࿿h Expressivity;
࿿750࿿h Penetrance:
࿿751࿿h Reduced penetrance;

← Which of the following statement about Mitochondrial inheritance is true:


Mitochondrial diseases have nothing to do with nuclear genes.
Mitochondrial conditions only affect muscle and nerve tissue;
The risk of passing on a mitochondrial condition to the next generation may be as high as 100%
Mitochondrial genes mutate less often than nuclear genes:

← When number of bases involved is not a multiple of three (i.e., is not an integral number Of codons), and
when occurs in a coding sequence, the reading frame is altered beginning at the point of the insertion or
Nonsense mutations:
Silence mutations:
Frameshitf mutation
Missense mutations.

← The severiy of expression of the phenotype among individuals With the same disease-causing genotype
is referred to as
࿿752࿿h Variable expressivity
࿿753࿿h Expressivity
࿿754࿿h Penetrance

Galactosemia is an autosomal recessive disorder. Symptoms include failure to thrive, vomiting, jaundice.
hepatomegaly, and cataracts. It is most commonly due to a deficiency of galactose-phosphate uridyttransferase
(GALT). A man and woman. who are both known to be carriers for galactosemia, marry and have a child - To their
relief. the standard newborn screening test results, Which measures the blood level Of galactose and galactose -l-
P. shows that their child is not affected. Based upon this information. what is the probability that the child is
a carrier for galactosemia?
࿿755࿿h 50%
࿿756࿿h 0%
࿿757࿿h 25%;
࿿758࿿h 75%.

← Anticipation is characteristic of conditions caused by:


Mitochondrial inheritance;
Genomic imprinting
Trinucleotide repeat expansions
Microdeletions;

Set 1 new 1 Page 7


࿿759࿿h Which of the following is characteristic to autosomal-
recessive disease? a) Females are never affected.
b) It affects both sexes equally'
c) It affects females primarily; d)
It affects males primarily;

࿿760࿿h Which of the following antigenes is present in the erythrocytes of people with B
blood group? a) B
b) A c)
AB d)
None

࿿761࿿h What type of mutation does not change the sequence of the gene product?
࿿762࿿h Non-coding:
࿿763࿿h Nonsense;
࿿764࿿h Silent;
࿿765࿿h Quiescent.

The presence in an individual or a tissue of at least two cell lines that differ genetically but are derived from a
single zygote is refered to as
࿿766࿿h Mitochondrial inheritance:
࿿767࿿h Trinucleotide repeat expansions.
࿿768࿿h Mosaicism
࿿769࿿h imprinting;

Which statement about X-Iinked inheritance is true:


࿿770࿿h The risk of germline mosaicism does not need to be considered.
࿿771࿿h When dominant females are usually as severely affected as males;
࿿772࿿h Condition cannot be passed from an affected father to his son,
࿿773࿿h The condition can be passed from an affected father to his son;

← Which the following statements about mitochondrial disorders is true:


MERRF syndrome is always Caused by point mutation of t-RNA
All follow matrilinear inheritance
Leigh disease is always caused by the same point mutation;
All follow patrilinear Inheritance;

← Nearly every individual affected with neurofibromatosis type 1 (NF1 ) exhibits clinical symptoms. Some however
may present with cafe au lait spots and Lisch nodules, While others have life threatening tumors surrounding the
spinal cord. These represent an example of which Of the following?
Reduced penetrance:
Variable expressivity
Locus heterogeneity.
Sex-influenced expression.

← Severs inherited disorders are much more common in close-knit religious communities, such as the Amish
or Ashkenazi, than in the general population. This is at least partly due to the fact that:
࿿774࿿h shared environmental condtions such as diet can increase mutation rate.
࿿775࿿h modern medical Care is not widely available in such communities;
࿿776࿿h such communities are more likely to marry relatives
࿿777࿿h community members care for each other, and such disorders are contagious;

Which of the following statements is true:


࿿778࿿h in HD, those homozygous for the mutation are no more severely affected than those who are
heterozygous
࿿779࿿h In Huntington disease (HD) an earlier age of onset in the offspring is more likely if the gene is
passed from affected mother rather than an affected father;
࿿780࿿h From the onset of HD, the average duration of the illness until a terminal event is 25-30 years;
࿿781࿿h Myotonic dystrophy is a cause oi neonatal hypetonia
Set 1 new 1 Page 8
࿿782࿿h Changing the codon AGC to AGA represents a
______ mutation a) Missense
b) Frameshift;
c) Nonsense;
d) Deletion.

࿿783࿿h Which of the following statement is true:


࿿784࿿h The individual who seeks genetic counseling is the proband
࿿785࿿h Retinitis pigmentosa is not genetically heterogeneous;
࿿786࿿h Genetic counseling is all about recurrence risks;
࿿787࿿h Good counseling should not be measured by the patient/client's ability to remember generic
risks.

Ethnic Differences in disease frequencies are most apparent for.


࿿788࿿h Autosomal recessive conditions
࿿789࿿h X-Linked recessive conditions;
࿿790࿿h Sex Chromosome aneuploidies.
࿿791࿿h Autosomal dominant conditions;

← Which of the following is an individual with nonidentical pair of alleles?


Diheterozygote.
Hemizygote
Heterozygote
Homozygote

← If a father is Rh-. and a mother is Rh-. then the child can be:
Rh- ONLY (100%) sure

࿿792࿿h Which of the following diseases is characterized With maternal


genome imprinting? a) Miotonic dystrophy:
b) Prader-Willi syndrome
c) Angelmann syndrome d)
Fragile X syndrome;

࿿793࿿h Ellis-van Creveld syndrome is a type Of dwarfism inherited as an autosomal recessive disease. It
is rare in general population of the United States. but it is unusually common in the Amish people of
Pennsylvania. with this disease have a normal body length. but have shortened limbs and appendages as well as
extra
or toes. Dick suffers from Ellis-van Creveld syndrome. Margie has a normal stature, but her father suffered
Ellis-van Creveld syndrome. What is the probability that Dick and Margie will produce a dwarf child?
࿿794࿿h 25%
࿿795࿿h 50%
࿿796࿿h 75%
࿿797࿿h 0%

← Which one of the following statements regarding treatment of genetic diseases is true?
Genetic diseases cannot be treated if they are congenital;
Genetic diseases can only be treated if they are understood at least in part at the biochemical level
Diagnostic precision is often critically important in designing effective treatment for genetic diseases
Treatment of genetic diseases requires proper regulation of the therapeutic gene

← In a case when two related individuals in a family have the same disease, they are said to be:
Quantitative traits;
Qualitative trait.
concordant
Discordant ;

← What blood type is compatible for the transfusion of red blood cells in an O Rh negative individual
࿿798࿿h A Rh (D) negative;
࿿799࿿h O Rh (D) positive;
࿿800࿿h O Rh (D) negative
࿿801࿿h None of these.
Set 1 new 1 Page 9
86. Which of the following statements is true about mutations?
࿿802࿿h They are never as simple as an error in a single codon in a DNA molecule;
࿿803࿿h They can produce new alleles of existing genes
࿿804࿿h They can be inherited if they are in somatic cell"
࿿805࿿h The changing of a cell from one type to another

What is the definition of translocation?


← A change in the amino acid sequence.
← A change in the DNA;
← An exchange of chromosome parts
← Moving things around;

← Which of be following statements about venous thrombosis is true:


← Testing for factor V Leiden and the prothrombin variant will change the management of a patient with venous
thrombosis
Mela five times more likely to be affected with venous thrombosis than female
the factor V Leiden mutation causes reduced expression of the factor V gene
compound heterozygotes for factor V Leiden and prothrombin variant have a 20 fold increased risk of venous
thrombosis

࿿806࿿h Which of the following types of mutations causes the trisomy of the 21st chromosome?
Genome
Chromosomal
Gene
Somatic

← Elizabeth is a 25 year old woman with colour blindness, an X-Linked recessive disorder, Aside from being colorblind
medical history is unremarkable, her father is also colour blind, there is no history of colour blindness on her
mother's side of the family. what is likely cause of Elizabeth colour blindness
← XY sex reversal
← Skewed X inactivation
← Turner syndrome
← klinefelter syndrome

← which of the following mutations changes the DNA sequence of a nuclear or mitochondrial genome
← Chromosomal
← Genome
← Dynamic
← Gene

← Single allele that controls more than one character is said to be


← Pleiotropic
← linked
← Polygenic
← Autotrophic

← which of the following antigens is present in the erythrocytes of people with B blood group
← AB
← A
← B
← None

← what is the name of the member for home a family with a genetic disorder was first brought to the attention of the
geneticist
࿿807࿿h Sibs
࿿808࿿h Parents
࿿809࿿h Proband
࿿810࿿h Consultant

Set 1 new 1 Page 10


࿿811࿿h which statement about autosomal recessive
inheritance is true a) Amgleman syndrome follows this pattern
b) Usually only single generation has affected individuals
c) If both parents are carriers there is a risk that at conception any child might be a carrier
d) Females are more likely to be affected than males

96.
Only variable Expressivity
࿿812࿿h Age-dependent expressivity
࿿813࿿h only incomplete penetrance

97. most known human mutations


࿿814࿿h appear to be spontaneous
࿿815࿿h are caused by radiation
࿿816࿿h are due to viruses
࿿817࿿h are due to microorganisms

what happens to the extra X chromosome in patients with additional X chromosome


࿿818࿿h region near the centromere is inactivated
࿿819࿿h the entire chromosome is inactivated (ALMOST)
࿿820࿿h P arm is inactivated
࿿821࿿h q arm is inactivated

← changing the codon AGC to AGA represents a ______ mutation


Deletion
Nonsense
Missense
Frameshift

← which of the following diseases is caused by missense mutation


Sickle Cell anaemia
Tay-Sachs disease
PKU
cystic fibrosis

Set 1 new 1 Page 11


Set 1 new 1 Page 12
Set 1 new 1 Page 13
Set 1 new 1 Page 14
Set 1 new 1 Page 15
Portal Questions

Which of the following diseases is characterized with X-linked inheritance, is


severe, progressive quite common and incurable?

࿿822࿿h Thalassemia:
࿿823࿿h Cystic fibrosis:
C. Duchenne muscular dystrophy:
D. Phenylketonuria.

Which type of rearrangement results from breakage of nonhomologous chromosomes, with


exchange of broken off segments?

࿿824࿿h Inversions
࿿825࿿h Duplications
C. Reciprocal translocations
D. Deletions

A genetic disease that is either present or absent is referred to as

࿿826࿿h Concordance
࿿827࿿h Discordance
࿿828࿿h Quantitati
ve traits D.
Qualitative Trait

Which is NOT characteristic to mtDNA?

A. Is a circular chromosome
B. Is inherited both maternally and paternally
࿿829࿿h Has no reparation system
࿿830࿿h Is 16,5 kb long
Mutations

࿿831࿿h Are permanent changes in the DNA sequence or structure


࿿832࿿h Produce allelic variation
࿿833࿿h Are more likely to be harmful than beneficial
D. All of the above

Which of the following types of thrombosis is most common in whites?

࿿834࿿h Cerebral vein thrombosis


࿿835࿿h Placental artery
thrombosis C. Deep vein
thrombosis
࿿836࿿h Peripheral vein thrombosis

Which type of inheritance is characterized with the transmission of a disease from parents to
children in every generation

࿿837࿿h Autosomal recessive


࿿838࿿h X-linked
࿿839࿿h Y linked
D. Autosomal dominant

Which of the following types of inheritance best corresponds to the statement: each affected
individual has an affected parent?

A. Autosomal-dominant
࿿840࿿h Autosomal recessive
࿿841࿿h X-linked recessive
࿿842࿿h Y-linked

According to the X-linked genes males are

࿿843࿿h Homozygotes
࿿844࿿h Hetero
zygotes C.
Hemizygotes
࿿845࿿h Compounds.
Which of the following definitions corresponds to the description given as the
chromosome constitution of individuals:

࿿846࿿h Genotype
࿿847࿿h Ge
nome C.
Karyotype
࿿848࿿h Phenotype.

Once transcription has been completed, which of the following is NOT necessary for
protein synthesis to occur?

࿿849࿿h tRNA
࿿850࿿h Ribosomes
࿿851࿿h mRNA
D. DNA

What are two modular elements that appear as consensus sequences upstream from the gene?

࿿852࿿h Microsatellites and transposons


࿿853࿿h DNA and nucleolar
organizers C. TATA and CAT
࿿854࿿h TTAA and CCTT.

Which of the following processes take part in the cytoplasm of a cell?

࿿855࿿h Transcription
࿿856࿿h RNA processing
C. Binding of amino acids with transport RNA
D. RNA splicing

Which of the following corresponds to an individual with identical pair of alleles?

A. Homozygote
࿿857࿿h Heterozygote
࿿858࿿h Hemizygote
࿿859࿿h Diheterozygote
In a case when two related individuals in a family have the same disease, they are said to be

A. Concordant
࿿860࿿h Discordant
࿿861࿿h Quantitative traits
࿿862࿿h Qualitative trait.

Translation begins

࿿863࿿h At the replication fork


࿿864࿿h On the lagging
strand C. at the start
codon
࿿865࿿h In the nucleus.

If a mother cell has 14 chromosomes prior to mitosis, how many chromosomes will the
daughter cell have?

࿿866࿿h
28 B.
14 C.
7
D. Any number between 7 and 28

Which of the following can cause frameshift mutation?

࿿867࿿h Translocation
࿿868࿿h Transversion
࿿869࿿h Deletion
of 3 bp D. Insertion
of 1 bp

If a fragment of a chromosome breaks off and then reattaches to the original chromosome but
in the reverse direction, the resulting chromosomal abnormality is called

࿿870࿿h Deletio
n B. Inversion C.
Translocation D.
Nondisjunction
Which of the following is a gene coding region?

࿿871࿿h Int
ron B. Exon
C. Codon D.
Anticodon

An octamer of 4 histones complexed with DNA forms

࿿872࿿h Endosome
࿿873࿿h Meso
some C.
Nucleosome
࿿874࿿h Centromere

Which disease is most commonly associated with a mutation-induced alteration of a protein


by expansion of an unstable repeat

࿿875࿿h Cystic fibrosis


(CF) B. Huntington
disease C.
Phenylketonuria (PKU) D.
Tay-Sachs disease.

The triplet base sequence of tRNA nucleotides that is complementary to an mRNA codon
is called

A. Anticodon
࿿876࿿h Codon
࿿877࿿h Exon
࿿878࿿h Cistron

Which of the following is NOT true about polymorphism?

࿿879࿿h Usually it has no adverse effects on individual's health


࿿880࿿h It promotes diversity
࿿881࿿h Is transmitted from generation to generation
D. It is a type of mutation
Meiosis is involved in which of the following life cycle events?

A. Spermatogenesis
࿿882࿿h Cell regeneration
࿿883࿿h Growth
࿿884࿿h Development

Which change in the polypeptide chain is characteristic to HbS that causes sickle-cell disease?

࿿885࿿h Beta chain


GLu66Lys B. Beta chain
Glu6Val C. Beta chain
His92Tyr D. Beta chain
Glu2Lys

The most common heritable form of moderate mental retardation and is second only to
Down syndrome among all causes of mental retardation in males

࿿886࿿h Patau syndrome


࿿887࿿h Edward syndrome
࿿888࿿h Myotonic
dystrophy D. Fragile X
syndrome

Anticipation is characteristic of conditions caused by

࿿889࿿h Microdeletions
࿿890࿿h Mitochondrial inheritance
࿿891࿿h Genomic imprinting
D. Trinucleotide repeat expansions

Pseudogenes

A. Are nonfunctional genes


࿿892࿿h Are expressed genes
࿿893࿿h Are incomplete genes
࿿894࿿h Are only found in prokaryotes
Blood types are
A. a Genetically inherited
࿿895࿿h Common criteria for selecting marriage partners
࿿896࿿h Genetically inherited but may be changed after birth as a result of natural selection
࿿897࿿h Acquired after birth as a result of environmental factors

How long does a cell cycle last?

A. It depends on the cell type


࿿898࿿h It is approximately 36 hours
࿿899࿿h It varies greatly among cells of the same type
࿿900࿿h It is always about 24 hours

A frameshift mutation:

࿿901࿿h Replaces one amino acid with another


࿿902࿿h Removes part of the protein
C. Introduces a section of amino acids not normally found
D. Joins two different proteins

Which of the following corresponds to the alternative form of a gene?

࿿903࿿h Non-allele
࿿904࿿h Compound
࿿905࿿h He
mizygote D.
Allele

The diploid number of chromosomes:

࿿906࿿h was in the mother cell and is in the two daughter cells, following mitosis
࿿907࿿h Is the 2N number
࿿908࿿h varies according to the particular organism
D. all of the above
What happens to the extra X chromosome in patients with additional X chromosome?

A. Almost entire chromosome is inactivated


࿿909࿿h p arm is inactivated
࿿910࿿h Region near the centromere is inactivated
࿿911࿿h q arm is inactivated

According to the central dogma of biology

A. DNA is a template for all RNA production


࿿912࿿h DNA is a template only for DNA replication
࿿913࿿h Translation precedes transcription
࿿914࿿h Replication is the process where RNA is decoded to form a protein

What is the number of human mitochondrial genes?


A.37
࿿915࿿h 500
࿿916࿿h 1500
࿿917࿿h 20 000

Which of the following is characteristic to the expansion of unstable repeat sequences?

A. Dynamic nature
࿿918࿿h Translocation
࿿919࿿h Insertion
࿿920࿿h Deletion.

Which amino acid accumulates in Body fluids during phenylketonuria?

࿿921࿿h Valine
࿿922࿿h Thyrosine
C. Phenylalanine
D. Glutamate
Which of the following is responsible for the development of respiratory problems in
cystic fibrosis?

A. Lack of vitamins
B. Accumulation of thick mucus
࿿923࿿h Increased immunity
࿿924࿿h Anemia

A couple has a female child with Tay-Sachs disease, and three unaffected children. Neither
parent nor any of the four biological grandparents of the affected child has had this disease.
The most likely genetic explanation is that Tay-Sachs disease is inherited as:

࿿925࿿h Autosomal dominant


disorder B. Autosomal recessive
disorder C. Sex-linked recessive
disorder D. Sex-linked dominant
disorder.

The expression of the disease phenotype depends on whether the mutant allele has
been inherited from the father or from the mother, a phenomenon known as

࿿926࿿h X-Linked Recessive Inheritance


࿿927࿿h Autosomal Dominant Inheritance
࿿928࿿h Autosomal Recessive
Inheritance D. Genomic imprinting

Which of the following statements about mitochondrial genome is true

࿿929࿿h There are 13 tRNA gene in the mitochondrial genome


࿿930࿿h X There are 13 protein coding gene in the mitochondrial
genome C. There are 22 tRNA gene in the mitochondrial genome
࿿931࿿h There are 22 protein coding gene products from the mitochondrial genome
In which of the following the short arms of two acrocentric chromosomes break and the long
ams join, forming an unusual, long chromosome:
A. Reciprocal translocation
B. Robertsonian translocation
࿿932࿿h Inversion
࿿933࿿h Duplication

Which of the following is the characteristic of autosomal dominant inheritance?

A. The phenotype usually appears in every generation, each affected person having an
affected parent
࿿934࿿h The incidence of the trait is much higher in males than in females
࿿935࿿h Heterozygous females are usually unaffected
࿿936࿿h Affected males with normal mates have no affected sons and no normal
daughters

Change in nucleotide sequence in DNA is called:

࿿937࿿h M
utagen B.
Mutation
C. Recombination
D. Translation

Which of the following type of inheritance is characterized by the fact that the parents of an
affected child are asymptomatic carriers of mutant alleles?

࿿938࿿h Autosomal Dominant


Inheritance B. Autosomal Recessive
Inheritance C. Genomic imprinting
D. X-Linked Recessive Inheritance

What is the name of the process, when one segment of a chromosome moves into the other
chromosome and at the same time maintains or inverts its orientation

࿿939࿿h Translocation
࿿940࿿h Duplication
࿿941࿿h Inversion
D. Insertion
Fragile X syndrome and Huntington disease are caused by

࿿942࿿h Tandem duplication


࿿943࿿h Fusion gene
C. Expanding triplet repeats
D. Deletion

Which of the following chromosomal trisomies causes Down syndrome?

࿿944࿿h 18 chromosome trisomy


࿿945࿿h 13 chromosome
trisomy C. 21 chromosome
trisomy
࿿946࿿h 22 chromosome trisomy

What treatment strategy should be used in order to prevent neurological complications


in phenylketonuria?

࿿947࿿h Enzyme replacement therapy


࿿948࿿h Gene therapy
C. Diet
D. Growth hormone

The most common autosomal aneuploidy in liveborn children is:

࿿949࿿h Trisomy 18
࿿950࿿h Trisomy 13
࿿951࿿h Tris
omy 15 D.
Trisomy 21

What is an intron?

࿿952࿿h It is a region of DNA located between genes that is not translated


into protein B. It is region of DNA located within a genes that are not translated
into protein C. It is a regulatory gene
D. It is a structural gene
The growth of a polypeptide chain during translation is called

࿿953࿿h Initiation
࿿954࿿h Elongation
࿿955࿿h Translation
࿿956࿿h Termination

In the genetic dictionary, there are 64 codons because:

࿿957࿿h 64 amino acids are to be coded


࿿958࿿h 64 types of tRNAs are present
࿿959࿿h There are 44 nonsense codons and 20 sense
codons D. Genetic code has triplet nature

Which of the following chromosomal pathology is caused by the duplication of part of the
chromosome?

A. Partial trisomy
࿿960࿿h Partial monosomy
࿿961࿿h X chromosome trisomy
࿿962࿿h 12th chromosome trisomy

A promoter site on DNA:

A. Initiates transcription
࿿963࿿h Regulates termination
࿿964࿿h Codes for RNA
࿿965࿿h Transcribes repressor

Which type of bond joins two strands of DNA?

࿿966࿿h Covalent bond


࿿967࿿h Phosphodiester Bond
࿿968࿿h Ionic bond
D. Hydrogen bond
The base pair rules states that

࿿969࿿h Replication is
semiconservative B. A pairs with T,
G pairs with C
࿿970࿿h DNA is a double helix held together by hydrogen bonds
࿿971࿿h A pairs with G, T pairs with C

At which stage of mitosis chromosomes are best seen for analysis

࿿972࿿h Inte
rphase B.
Metaphase C.
Telophase D.
Anaphase

Which of the following definitions best describes set of alleles that form a genetic constitution of
an individual?

࿿973࿿h Phen
otype B.
Genotype C.
Compound D.
Heterozygote

How would you expect DNA methylation to change gene expression?

࿿974࿿h Extremely increase expression


࿿975࿿h Moderately increase expression
࿿976࿿h No change in expression
D. Decrease expression

Which statement about transcription is true:

࿿977࿿h Describes the production of polypeptides from the mRNA


template B. Occurs in the nucleus
C. Produces single-stranded mRNA using the transcribed DNA strand as a template
D. Results in the doubling of the DNA molecule
During which autosomal-recessive disease is mental retardation caused by the accumulation of
a certain amino acid in blood and urine?

࿿978࿿h Tay-Sachs
࿿979࿿h Cystic fibrosis
࿿980࿿h Muscular
dystrophy D.
Phenylketonuria

Which one of the following combinations of active and inactive X chromosomes is found in
49 XXXY individuals?

A. 1 active and 2 inactive X chromosomes


࿿981࿿h 2 active and 2 inactive X chromosomes
࿿982࿿h 3 active and 1 inactive X chromosomes
࿿983࿿h 4 active and no inactive X chromosomes

Which of the following may be a cause of cerebral vein thrombosis?

࿿984࿿h Mutation of prothrombin and FVL genes at the same time


࿿985࿿h Mutation of prothrombin gene and use of oral contraceptives
࿿986࿿h Mutation of FVL gene and use of oral contraceptives
D. Any of the above

Which body system is damaged during hyperphenylalaninemia?

࿿987࿿h M
uscle B.
Nervous
C. Cardiovascular
D. Respiratory

How are mtDNA distributed to the daughter cells during mitosis?

࿿988࿿h Non-randomly
࿿989࿿h Both cells receive exactly same amount and type of
mitochondria C. Randomly
࿿990࿿h Larger sıze mitochondria go to one cell and smaller ones go to another cell
Which histone protein attaches to DNA strands between nucleosomes?

࿿991࿿h H2A
࿿992࿿h
H4 C.
H1
࿿993࿿h H3

If at the end of meiosis, the daughter cell has 6 chromosomes, how many chromosomes were
in the mother cell?

࿿994࿿h 3
࿿995࿿h 6
C. 12
࿿996࿿h 24

The replication of DNA is possible due to

࿿997࿿h Hydrogen bonds


࿿998࿿h Phosphate backbone
C. Complementary base pair rule
D. The helical nature of the DNA double strand

DNA synthesis from a DNA template is called

࿿999࿿h Translation
࿿1000࿿h Repli
cation C.
Transcription
࿿1001࿿h Transduction

What is the number of Barr bodies (inactive X) in Klinefelter syndrome?

࿿1002
࿿h 0
B. 1
C. 2
D. 3
Which of the following is the characteristic of X-linked recessive inheritance?

A. The gene responsible for the condition is transmitted from an affected man
through all his daughters
࿿1003࿿h Affected females are about twice as common as affected males
࿿1004࿿h Any child of an affected parent has a 50% risk of inheriting the trait
࿿1005࿿h Males and females are equally likely to be affected

Changing the codon AGC to AGA represents a mutation


A. Missense
B. Nonsense
C. Frameshift
D. Deletion

Which of the following disorder corresponds to the single-gene disorder?

A. Cystic fibrosis
࿿1006࿿h Turner syndrome
࿿1007࿿h Cleft lip and cleft palate
࿿1008࿿h Down syndrome

Which of the following processes causes inhibition of transcription in Fragile X syndrome?

࿿1009࿿h Deletion
࿿1010࿿h Duplication
C. Methylation of cytosines
D. Inversion
Exchange of segments between non-homologous chromosomes is called:

࿿1011࿿h Crossing over


࿿1012࿿h Inversion
࿿1013࿿h Duplic
ation D.
Translocation

If one strand of a DNA molecule has the base sequence 3 ATTGCAT5’, its
complementary strand will have the sequence

࿿1014࿿h 5'ATG
CAAT B.
5’TAACGTA3’ C.
3"TAACGTAS

Which of the following mutation affects only a single nucleotide?

࿿1015࿿h Conditional mutation


࿿1016࿿h Germline
mutation C. Point
mutation
࿿1017࿿h Regional mutation

The complex of DNA and protein in chromosomes is called

࿿1018࿿h Centromeres
࿿1019࿿h Histone
࿿1020࿿h Cent
rosome D.
Chromatin

Which of the following definitions best describes location of a gene in a chromosome?

A. Locus
࿿1021࿿h Site
࿿1022࿿h Promoter
࿿1023࿿h Primer
Which of the following definitions best corresponds to an individual's morphological
characteristic?

࿿1024࿿h Heterozygote
࿿1025࿿h Hom
ozygote C.
Phenotype
࿿1026࿿h Genotype

Which of the following refers to the autosomes?

A. Any chromosomes
B. Any chromosome except sex chromosome
࿿1027࿿h Any sex chromosome
࿿1028࿿h Any chromosome except the 22-th chromosome

Each nucleotide triplet in mRNA that specifies a particular amino acid is called

࿿1029࿿h M
utagen B.
Codon C.
Anticodon D.
Exon

Which of the following statements is true about sex determination?

࿿1030࿿h It is the presence of the second X chromosomes that determines the


femaleness B. It is the presence of Y chromosome that determines the maleness
C. It is the lack of X chromosome that determines the maleness
D. All the statements are correct

Which of the following is NOT true about cleft lip/palate (CL/P)?

࿿1031࿿h It may be syndromic or nonsyndromic,


࿿1032࿿h It may be caused by both genetic and environmental
factors C. It is a very rare congenital malformation
࿿1033࿿h Most cases of nonsyndromic CL/P demonstrate familial aggregation
Given the following DNA strand. TACGGGATT, which of the following is its
complementary mRNA?

A. AUGCCCUAA
࿿1034࿿h AAUCAGUTA
࿿1035࿿h ATG TAA
࿿1036࿿h UACGA

A point mutation that changes a codon specifying an amino acid into a stop codon is called

࿿1037࿿h Missense
mutation B. Nonsense
mutation C. Frameshift
mutation D. Deletion
mutation

Loss of a sequence of DNA from a chromosome is called

࿿1038࿿h Cross
ing-over B.
Deletion
C. Duplication
D. Inversion

In general, full monosomes are not compatible with life and cause fetal death. Which of the
following chromosomal monosomies is an exception and can produce a liveborn infant?

A. X chromosome monosomy
࿿1039࿿h 21 chromosome monosomy
࿿1040࿿h 22 chromosome monosomy
࿿1041࿿h 17 chromosome monosomy

Separation of homologous chromosomes is called:

࿿1042࿿h Dispersion
࿿1043࿿h Bivalent
formation C.
Disjunction
࿿1044࿿h Crossing over
What type of mutation does not change the sequence of the gene product?

A. Silent
࿿1045࿿h Nonsense
࿿1046࿿h Non-coding
࿿1047࿿h Missense

During normal adult life the major form of hemoglobin produced in red blood cells is.

࿿1048࿿h Hb H
࿿1049࿿h Hb S
࿿1050࿿h

Hb F D.
Hb A

Which of the following mutations arise without exposure to external agents?

A. Spontaneous mutations
࿿1051࿿h Analogous mutations
࿿1052࿿h Induced mutations
࿿1053࿿h None of these

Which of the following is an individual with identical pair of alleles?

࿿1054࿿h Homoz
ygote B.
Heterozygote C.
Hemizygote D.
Diheterozygote

Couples who have one or more ancestors in common are called:

࿿1055࿿h Pedigree
࿿1056࿿h Kindred
࿿1057࿿h Sibship
D. Consanguineous
DNA methylation may be a significant mode of genetic regulation in eukaryotes.
Methylation refers to

࿿1058࿿h Altering RNA polymerase activity by methylation


࿿1059࿿h Changes in DNA-DNA hydrogen bonding
࿿1060࿿h Altering translational activity, especially of highly
methylated tRNAs D. Addition of methyl groups to the cytosine of CG
doublets
SET 2-4
Final exam — GENETICS

࿿1061࿿h What is different about interphase before and after meiosis I?

࿿1062࿿h DNA replication occurs at half the rate before meiosis I;

࿿1063࿿h DNA replication occurs at double the


rate before meiosis I; There is DNA replication before,
but not after, meiosis I;

࿿1064࿿h There is DNA replication after, but not before, meiosis I

2. What is the complementary strand for the DNA of the following RNA. strand: 5‘GCACGUUUACCGA 3"?

3' CGTGCAAATGGCT 3'; B. 3' UACGCAAAUGGCUS’;


C. 3' AGCCAUUUGCGUA 5’: D. 3’ AUGCGUUUACCGA 5.

3. Antiparallel strands of a DNA molecule mean that

A. One strand turns clockwise B. One strand turns anti-clockwise;


C. The phosphate groups of two DNA strands, at their ends, share the same poles

D. The phosphate groups at the start of two DNA strands are in opposite position (pole).

࿿1065࿿h Which of the following stages is characterised by the


replication of DNA? A. Stage between two meiotic divisions; B. Metaphase
of mitosis; C.Stage before I meiotic division;D. Prophase of mitosis.
࿿1066࿿h Which of the following statements about mitochondrial
genome is true: A. There are 13 tRNA gene in the mitochondrial genome;

B. There are 13 protein coding gene in the mitochondrial genome; C.

There are 22 tRNA gene in the mitochondrial genome;

D. There are 22 protein coding gene products from the mitochondrial genome.

࿿1067࿿h DNA exists in a double-stranded form whereas RNA is mainly a single stranded
molecule. What is the likely reason for DNA being double stranded?

A. DNA cannot exist in the single stranded form;

B. RNA strands cannot form base pairs;

C. It is easier to replicate double stranded DNA than single stranded RNA;


D. Double stranded DNA is a more stable structure.

࿿1068࿿h Okazaki fragments:


A. Are formed in the lagging strand;B. Are formed in the leading strand;
C. Remove the primer; D. Are synthesized by primase.

8. Which of the following statements about DNA replication is TRUE?

A. The leading strand is replicated discontinuously, while the lagging strand is replicated continuously

B. The leading strand is replicated continuously, while the lagging strand is replicated discontinuously

C. Both the leading and the lagging strands are replicated continuously
D. Both the leading and the lagging strands are replicated discontinuously

࿿1069࿿h Which of the following processes take part in a nucleus of a cell?

Transcription
Peptide bonds between amino acids
Translation
Binding with amino acids with transport RNA
࿿1070࿿h Which statement about transcription is true
Produces single stranded mRNA using the sense DNA strand as a template
Occurs in the nucleus
IS regulated by transcription factors that bind to the 3’ UTR
Describes the production of polypeptides from the mRNA template

࿿1071࿿h If at the end of meiosis, the daughter cell has 12 chromosomes, how many
chromosomes were in the mother cell?

3
6
12
24

࿿1072࿿h Which of the following processes is called a segregation between homological


chromosomes during meiosis?

Fusion
Disjunction
Exchange of the segments
Formation of bivalents

࿿1073࿿h During metaphase all of the following takes place EXCEPT:

Chromosomes move to opposite spindle poles;


Chromosomes line up on the spindle equator
Microtubules attach to sister chromatids of each chromosome
Chromosomes are in their most condensed form.

࿿1074࿿h In what way do mitosis and meiosis differ:

A. Replication of DNA precedes mitosis, it does nor precede meiosis


B. Mitosis produces cells which are identical, meiosis produces cells which are not identical;

C. Centrioles form during mitosis, they do not form during meiosis;

D. Chromosomal spindles shorten during mitosis, they do not during meiosis.

15. The human haploid genome is composed of approximately:


A. 6 million base pairs of DNA and 2000 protein coding genes
B. 6 billion base pairs of DNA and 20000 protein coding genes

C. 3 billion base pairs of DNA and 20000 protein coding genes


D. 3 million base pairs of DNA and 20000 protein coding genes

16, In eukaryotes, many genes may have to interact with each other, requiring more interacting
elements than can fit around a single promoter. This physical limitation is overcome by:

A, Alternating promoters and operators

B. Placing promoters on both sides of each gene

C. The use of very long promoters


D. Distant sites in a chromosome controlling transcription of a gene.

࿿1075࿿h Which of the following is NOT an example of repetitive DNA?


A. Promoter region;B. SINE DNA; C. Centromeric DNA; D. Telomeric DNA.

࿿1076࿿h DNA methylation may be a significant mode of genetic regulation in eukaryotes.


Methylation refers to:

A. Altering RNA polymerase activity by methylation;

B. Changes in DNA-DNA hydrogen binding;

C. Altering translational activity, especially of highly methylated tRNAs


D. Addition of methyl groups to the cytosine of CG doublets.
࿿1077࿿h When considering the initiation of transcription, one often finds consensus
sequences located in the region of the DNA where RNA polymerase(s) binds. Which of the following is
a common consensus sequence?

GGTTC
TATA
Satellite DNAs
TTTTAAAA

࿿1078࿿h Approximately 5% of the cytosine residues are methylated in the genome of


any given eukaryote. In what way is DNA methylation related to genetic regulation?

There is an inverse relationship between the degree of methylation of DNA and the degree of
gene expression

࿿1079࿿h X chromosome inactivation is a process by which one of two X chromosomes in human


females is condensed and inactivated to prevent overexpression of X-linked genes. What would most
likely be true about the degree of DNA methylation and histone acetylation on the inactivated X
chromosome?

Cytosines in CG islands would be hypomethylated and histone proteins would be acetylated


Cytosines in CG islands would be hypermethylated and histone proteins would be deacetylated
Cytosines in CG islands would be hyperacetylated and histone proteins would be
hypomethylated
Cytosines in CG islands would be deacetylated and histone proteins would be hypermethylated

࿿1080࿿h What is an exon?

It is a regulatory gene
It is a DNA spacer between genes
It is a segment of a split gene that codes for amino acids in a protein
It is a segment of a split gene that has no corresponding amino acids in a protein

࿿1081࿿h Which statement about base substitutions is true:


A. May result in nonsense mutations;B. Can not affect splicing:
C. Are always pathogenic; D. Result in frameshift mutations.
.
࿿1082࿿h Which of the following does not regulate normal gene
expression? A. Promotor; B. Enhancer; C. Locus control region; _D.
Poly-A tail.

࿿1083࿿h Which of the following processes take part in the cytoplasm of a cell?
A. Transcription; B RNA. processing; C. RNA splicing D. Binding of amino acids with transport RNA.

26. RNA processing varies in different cells. This is an example of


A. Transcriptional control of gene expression;B. Post-transcriptional control of gene expression;
C. Translational control of gene expression; D. Post-translational control of gene expression.

27. Which of the following processes causes diversity of antibodies?


A. Transcription; B. Translation;C. Somatic reconstruction;D. Posttranslational processing.

࿿1084࿿h What effect can methylation of CpG islands in the promoter region have
on gene expression?
A. Transcription level will be reduced;B. Transcription level is sharply increased; B.
It will affect translation; D. It will affect splicing process.

࿿1085࿿h Which of the following does NOT occur as a result of posttranslational modifications of a
protein?

←Addition of phosphate groups;


← Qassii of metals to create tertiary or quaternary structures;
← Removal of introns;
D. Removal of N-terminus amino acid.

࿿1086࿿h Which of the following undergoes processing?


← m-RNA;B. DNA; C. t-RNA D. r-RNA.
࿿1087࿿h An octamer of 4 histones complexed with DNA forms:
← Endosome; B. Mesosome; C. Centromere D. Nucleosome
࿿1088࿿h Which of the following is true about histone proteins:
They are positively charged;
They contain methionine and valine;
They bind to the base pairs of DNA;
There are about 20 histone types associated with DNA.

࿿1089࿿h What is the first stage of chromatin packing?

A.Coiling around nucleosomes


B. Formation of a 300-nm fiber;
C. Looping of 300-nm fibers;
D. Formation of a solenoid.

࿿1090࿿h Which of the following statements is true regarding transposons?

They are sequences of mRNA that can move around in the genome;
They exist in corn, but are not found in the human genome;

They are the most abundant type of repeat in the genome;


All of the above are true regarding transposons.

࿿1091࿿h In addition to highly repetitive and unique DNA sequences, a third category of DNA
sequences exists. What is it called, and which types of elements are involved in it?

A. Moderately repetitive DNA; SINEs, LINEs, and VNTRs;

B. Dominant DNA; euchromatin and heterochromatin;


C. Multiple gene family DNA; hemoglobin and 5. OS RNA:

D. Composite DNA; telomeres and heterochromatin


36. Pseudogenes are:
A. Recent beneficial mutations B. Nonfunctioning genes;C. Randomly transposed genes;
D. Artificial DNA sequences constructed by geneticists and inserted into the DNA of living organisms.

37. What is the name of the sequence that is the most common in a population?
A. Private sequence; B.Reference sequence;C. Variant sequence; D. All of the above.

࿿1092࿿h Substitution of one or the other of two bases at one location is


referred to as: A. Indel;
B.Single Nucleotide polymorphism SNP;
C. Simple;

D, Short tandem repeat polymorphism STRP

࿿1093࿿h ½. Which of the following is true about indels?

They have only 2 alleles;

There are several millions of indels in the genome;


They are crucial for identity testing;

The DNA sequences of indels are approximately 200-500 bp.

࿿1094࿿h When only one member of the pair of relatives is affected and the other is not, the
relatives are: A. Concordance B. Quantitative traits C.DiscordanctD. Qualitative trait.

࿿1095࿿h When should RhIG be injected to the Rh- woman?

A. After miscarriage or medical abortion B. After amniocentesis or chorionic villus sampling;


C. After delivery during the first 72 hours D. All of the above

࿿1096࿿h Because of its high level of informativity, the Federal Bureau of Investigation (FBI) uses a
set of
࿿1097࿿h of this type of marker for identity testing.
A. Variable number of tandem repeats (VNTR); B. Single nucleotide polymorphisms (SNPs);
C. Copy number polymorphisms (CNPs); D. all of above.
42. Which of the following statements is true of DNA damage?

A. Any DNA damage results in diseases such as cancer;


B. Any DNA damage is caused by physical, chemical or biological agents;

C. Most DNA damage is repaired by the cell;


D. Most DNA damage is advantageous to the cell.’

43. VNTRs are:


A. Used in DNA fingerprinting; B. Variable in number; C. Tandem repeats; D. All of the above are correct.

࿿1098࿿h DNA proofreading reparation takes place during which of the


following process? A. During G1 phase; B. After DNA replication; C. During DNA
synthesis; (D) A and C

No Answer wdk

45. Which of the following is directly involved in DNA repair:


A. Topoisomerase; B. Ribosomes;DNA polymerases;D. Telomerases.

46. Which of the following is a consequence of mutation?

A. Prevention of protein formation; B. Lowering in the amount of a protein;


Increases in the amount of a protein; D. Any of the above can occur

࿿1099࿿h Those mutations that arise in the absence of known mutagen are known as:
A. Induced mutations; B. Fused mutations; C. spontaneous mutations;D. None of the above. –

48. The mutations are mainly responsible for:


A. Variation in organisms; B. Maintaining genetic continuity;
A. Constancy in organisms; D. Increasing the population rate.
49. Most known human mutations:
A. Are caused by radiation; B. Appear to be spontaneous,C. Are due to viruses D. Are germline.
Nirmanie says so :)

50. A nucleotide deletion in DNA replication:


A. Causes the amino acids inserted after the deletion to be incorrect

B. Causes all of the amino acids of the protein to be incorrect


C. Causes one amino acid of the protein to be incorrect

D. Causes the amino acids inserted before the deletion to be incorrect

࿿1100࿿h Which of the following illustrates a single point mutation on a segment of


DNA which reads GTGGCACAT?
A. GIGGGACAU B. GTGGCAGAT C. TACACGGTG D. GIGGGCCACAT.

࿿1101࿿h A single nucleotide substitution (or point mutation) in a DNA sequence can alter the
code in a triplet of bases and cause the replacement of one amino acid by another in the gene
product.
A. Missense mutations;B. Nonsense mutations; C. Frameshift mutation; D. Silence mutations.

53. nonsense mutation introduced into the DNA sequence of a gene may:

A. Cause premature termination of the mRNA B. Shorten the length of the protein encoded by the gene
choose one. both are correct.

C. Have no effect on the transcript or protein made D. Cause a shift in reading frame

࿿1102࿿h The mutation causing sickle-cell disease is GAG-> GTG in the beta globin gene. This is
an example of A. Frameshift mutation B. Missense mutationC. Nonsense mutation; D. Dynamic
mutation.

࿿1103࿿h According to the X-linked genes males are


A. Homozygotes B. HemizygotesC. Heterozygotes; D. Compounds.
࿿1104࿿h Population carries two or more different alleles at a particular locus.
Which of the following definitions best corresponds to the above mentioned statement?
A. Homozygote; B. Heterozygote; C. Compound; D. Polymorphism.

࿿1105࿿h A genotype in which two different mutant alleles of the same gene are present,
rather than one es one mutant is referred to as:
A. Homozygote; B. Heterozygote C. Hemizygote D. Compound heterozygote

࿿1106࿿h What is called a set of alleles at a locus or locus clasters in chromosomes?


A. Wild type; B. Polymorphism; C. Homozygote; D. Haplotype.

࿿1107࿿h The probability that a gene will have any phenotypic expression at all is referred
to as:
A. Penetrance;B. Expressivity; C. Variable expressivity; D. reduced penetrance.

࿿1108࿿h Which statement about X-linked inheritance is true:

A. condition cannot be passed from an affected father to his son;

B. The condition can be passed from an affected father to his son;

C. When dominant, females are usually as severely affected as males;


D. The risk of germline mosaicism does not need to be considered.

࿿1109࿿h Which term is used to describe differences in gene expression based on the parent
of origin of an allele?
A. Parental determination; B. Sex-determination; C. Sex-linked dominance; D. Imprinting.

࿿1110࿿h The severity of a disease differs in people who have the same genotype. In this
case the phenotype is said to have:
A. Penetrance B. Expressivity; C. Reduced penetrance; D. Variable expressivity.
64, In familial hypercholesterolemia, individuals homozygous for the allele causing the disorder
completely lack receptors on liver cells that take up cholesterol from the blood stream. Heterozygotes
have one-half the number of receptors while individuals homozygous for the normal allele are
phenotypically normal. This is an example of:
A. Codominance; B. Epistasis C.Inccomplete dominance;D. Complete dominance.

࿿1111࿿h The presence in an individual or a tissue of at least two cell lines that differ
geneticaliy but are derived from a single zygote is referred to as:
A. Mosaicism B. Trinucleotide repeat expansions; C. imprinting; D. Mitochondrial inheritance.

66. Which of the following clinical phenotypes is characteristic to the mutations of mtDNA?

A. Neural and parenchymal disorders; B. Disorders of the sensory organs;


C. Neuromuscular disorders; D. Skeletal

67. Which of the following is characteristic to the expansions of unstable repeat sequences?
A. Haploinsufficiency; B. Anticipation;C. Locus heterogeneity; D. Mozaicism.

࿿1112࿿h Huntington's disease (HD) is inherited as an autosomal dominant, Patients first exhibit
symptoms as adults (90% between the ages 25 and 60, with the average in the early 40s). They suffer a
deterioration of the central system and lose mental abilities and motor control of the'limbs. The disease
is fatal within about 10 years after the onset of symptoms. What is the probability two individuals
homozygous for HD will have a child with HD?
A. 25%; B. 50%; C. 75%; D. 100%
࿿1113࿿h Galactosemia is anautosomal recessivedisorder. Symptoms include failure to thrive,
vomiting, jaundice, hepatomegaly, and cataracts. It is most commonly due to a deficiency of galactose-
phosphate uridyltransferase (GALT). A man and woman, who are both known to be carriersfor
galactosemia, marry and have a child. To their relief, the standard newborn screening test results,
which measures the blood level of galactose and galactose-1-P, shows that their childis affected.Based
upon this information, what is the probability that the child has the disease?
A. 0% B.25%;C. 50%; D. 75%

࿿1114࿿h An autosomal dominant trait exhibited by some Caucasians is called “woolly hair).
Mutation in this allele produces extremely brittle hair that breaks off before it grows very long. What
is the probability two individualsheterozygousfor this traithaving a normal child?
A. 0%:B. 25%:C.50%: D. 75%

࿿1115࿿h Which of the following processes, characteristic to the homological Chromosomes,


may cause a genetic disease? (homologous) eunata hadannepa
A. Non disjunction B. Conjugation C. Formation of bivalents D. Crossing over.

࿿1116࿿h Tay-Sachs disease shows autosomal recessive inheritance.Parents of a newly


diagnosed affected child are referred for genetic counseling. It would be correct to tell them
that:

A. the probability that their next child will be affected is 1 in 2 .

B. the probability that the older unaffected sister of the affected child is a carrier is 1

C. the fact that their last child was affected means that their next three children will not be affected
D. The probability that each parent is a carrier is 1

࿿1117࿿h Ellis-van Creveld syndrome is a type of dwarfism inherited as an autosomal recessive di


sease. It is rare in the general population of the United States, , but it is unusually common in the
Amish people of Pennsylvania. People with this disease have a normal body length, but have shortened
limbs and appendages as well as extra fingers or toes. Dick suffers from Ellis-van Creveld syndrome.
Margie has a normal stature, but her father suffered from Ellis-van Creveld syndrome. What is the
probability that Dick and Margiewill produce a dwarf child?
A.0% B. 25% C. 50% D. 100%

73. In which of the following conditions cytogenetic analysis is used?


A. Cataract B. Anemia C. Congenital anomalies D. Diabetes
࿿1118࿿h An 80-year-old man presented withimpairment of higher intellectual functionand
alterations In mood and behavior. His family reported progressive disorientation andmemory lossover
the last 6 months. There is no family history of dementia. The patient was tentatively diagnosed with
Alzheimer disease. Which one of the following best describes Alzheimer disease?

A. It is associated with the deposition of neurotoxic amyloid B peptide aggregates

B. It is an environmentally produced disease not influenced; by the genetics of the individual


C. It is associated with the accumulation of amyloid precursor protein

D. It is caused by the infectious Beta-sheet form of host-cell protein

75. Which of the following statements about venous thrombosis is true.

࿿1119࿿h Testing for factor V Leiden and the prothrombin variant will change the
management of a patient with on thrombosis;

Male are five time more likely. to be affected with venous thrombosis than females;
C. The factor V Leiden mutation causes reduced expression of the factor V gene;
D. The prothrombin variant causes increased expression of the prothrombin gene.

࿿1120࿿h What type of chromosomal aberration is indicated by the following


cytogenetic abbreviation: i(X)(q10)?
A. X monosomy; B: Isochromosome;C. Transversion; D. Terminal deletion.

badagini!!! - kaveesha issochromosome - dilina

࿿1121࿿h In some chromosomes the centromere is not in the middle of the chromosome,
however there is no significant difference in the length. of the arms. Which of the following
corresponds to this type of chromosome?
A. Metacentric; B. Submetacentric;C. Acrocentric; D. Telocentric.

࿿1122࿿h In which of the following diseases is cytogenetic analysis most frequently


recommended? A. PKU B. Galactozemia; C. Diabetes mellitus; D. Malignant neoplasia.
࿿1123࿿h In general, full monosomies are not compatible with life. Which of the
following chromosomal monosomies is an exception?

A. X chromosome monosomy B. 21 chromosome monosomy

C. 22 chromosome monosomy D. 17 chromosome monosomy.

࿿1124࿿h You have sent a blood sample of a dismorphic baby to the lab for the chromosomal
analysis. The answer from the lab says that the baby has 46,XY,del(18)(q12) karyotype. What type of
anomaly is in the karyotype?

࿿1125࿿h The long arm of chromosome 18 is shorter that it

should be B.The arms of chromosome 18 are equally short

C.One arm of chromosome 18 is very long compared to the other


The arms of chromosome 18 are equally long

࿿1126࿿h What happens to the extra X chromosome in patients with additional X chromosome?
← Almost entire chromosome is inactivated;

← p arm is inactivated;

← q arm is inactivated

← Region near the centromere is inactivated.

࿿1127࿿h Chromosomal aberrations include a change in the sequence of genes on a


chromosome. This is known as

← A carrier
←an inversion
←a duplication;
← a translocation.
84, Which of the following causes the bipotential gonad to differentiate into one of the sexes?

← Presence of SRY on the X chromosome leads to female development;

Presence of SRY on the Y chromosome leads to male development;

Recombined PAR regions of X and Y lead to male development;


←Formation of female gonads in the absence of Y chromosome inhibits male development.

࿿1128࿿h Fluorescence in situ hybridization using whole chromosome (painting) or specific


locus probes aa a detection of

← Gene expression;
← Triploidy;
← Gene amplification
← Reciprocal translocation

࿿1129࿿h Which of the following statement is correct:

← Female fetal development is solely dependent on normal functioning of the SRY gene;

← X chromosome inactivation, all the genes of one X are switched off,

← Male fetal development is solely dependent on normal functioning of the SRY gene;

← In most males with karyotype 46.XX the SRY gene is present and found on one of the X
chromosomes.

࿿1130࿿h Which of the following definitions corresponds to the description


given as: the chromosome constitution of individuals:

Genotype
Genome
Karyotype;
Phenotype.

࿿1131࿿h Which of the following statement is correct:

← X chromosome inactivation may be linked to discordance in monozygotic twin pairs;

← Male fetal development is solely dependent on normal functioning of the SRY gene;

← Female fetal development is solely dependent on normal functioning of the SRY gene;

← X chromosome inactivation, all the genes of one X are switched off.


࿿1132࿿h Which method is used to detect relative copy number gains and losses in a
genome-wide manner by hybridization

G-banding
Fluorescent in situ hybridization (FISH)
CGH array

D. Allele specific oligonucleotide hybridization

࿿1133࿿h Dosage compensation in mammals typically involves the random inactivation of


one of the two X chromosomes relatively early in development. In a such X. chromosome
inactivation involve: A. FMRI gene; B. SOX gene; C. SRY gene; D. XIST gene.

࿿1134࿿h Marcus and Carmen have a child with ambiguous external genitalia. The internal
genitalia are female and chromosome analysis is 46,XX. Biochemical studies reveal a deficiency of 21-
hydroxylase. What is the most likely diagnosis?

A. Androgen insensitivity;
B. Camptomelic dysplasia;
C. Congenital adrenal hyperplasia;/
D. Turner syndrome.

࿿1135࿿h If you have an aunt or uncle with a polygenic disorder


You will be more likely to develop the disorder than your mother or father;

You are not at risk to have children with the disorder;

You are at increased risk of developing the disorder than the average member of your
population

You will eventually develop that disorder.

࿿1136࿿h Which of the following statements about normal phenotypic variation is true?

Most normal characteristics in bays are inherited from their fathers and most normal
characteristics in girls are inherited from their mothers;

The genes involved are the same in all populations;


Environmental factors play a role;

← Most normal characteristics such as eye or hair color are transmitted as autosomal
recessive trait.
94, Which of the following statements about twins and twin studies is FALSE?

࿿1137࿿h Twin studies have been used to try to determine the role of heredity versus
environment;
࿿1138࿿h Dizygotic twins are an important control for any study using monozygotic twins:

࿿1139࿿h If the concordance values for monozygotic twins are much higher than for
dizygotic twins, the trait probably has a strong genetic basis;

࿿1140࿿h Dizygotic twins are as genetically similar as monozygotic twins,

࿿1141࿿h What is Barr body?


A. Inactive Y; B. Active X;inactive X;D. Active Y.

96. Which of the following statements is TRUE about concordance?


A. We can talk about concordance only when considering monozygotic twins;

B. Concordance means when any two individuals develop the same disease;
C. Concordance always results from the same genotypes;

D. Concordance may be a result of genetic and/or environmental factors.

࿿1142࿿h Which of the following is NOT a teratogen (an environmental agent that causes
malformation of an embryo or fetus) for cleft lip/palate?

Anticonvulsants (anti-epilepsy drugs).

࿿1143࿿h Which of the following is TRUE of a concordant trait between twins?

It is more common for the males to have the trait; B. Its

more common for the females to have the trait;

C. It is more common for the older twin to have the trait

D. Both share the trait.


࿿1144࿿h Which of the following is NOT a symptom of placental artery thrombosis?
Increased levels of alpha-fetoprotein (AFP) in amniotic fluid Again Nirmanie says so. n she is
correct . Jayawewa!

b. Intrauterine growth retardation [himeshata meka b. karanna ba] eunata mama kara :)
← Premature separation of the placenta from the uterine wall

← Preeclampsia

࿿1145࿿h Which of the following is a risk factor for Alzheimer’s disease?

Oral contraceptives;

Arthritis;

Use of anticonvulsants

Head trauma
Final exam - GENETICS SET 2-1

࿿1146࿿h During metaphase all of the following takes place EXCEPT:


Chromosomes are in their most condensed form
Chromosomes line up on the spindle equator
Microtubules attach to sister chromatids of each chromosome
Chromosomes move to opposite spindle poles
࿿1147࿿h If at the end of meiosis, the daughter cell has 12 chromosomes, how many
chromosomes were in the mother cell?
3
6
12
24
࿿1148࿿h Which of the following processes is called a segregation between homological
chromosomes during meiosis?
Fusion
Disjunction
Exchange of the segments
Formation of bivalents
࿿1149࿿h In what ways do mitosis and meiosis differ?
Mitosis produces cells which are identical, meiosis produces cells which are not
Replication of DNA precedes mitosis, they do not form during meiosis
Centrioles form during mitosis, they do not form during meiosis
Chromosomal spindles shorten during mitosis, they do not during meiosis
࿿1150࿿h What is different about interphase before and after meiosis I
DNA replication occurs at half the rate before meiosis I
DNA replication occurs at double the rate before meiosis I
There is DNA replication before, but nor after meiosis I
There is DNA replication after, but not before meiosis I
࿿1151࿿h The human haploid genome is composed of approximately
6 billion base pairs of DNA and 20000 protein coding genes
6 million base pairs of DNA and 2000 protein coding genes
3 million base pairs of DNA and 2000 protein coding genes
3 billion base pairs of DNA and 20000 protein coding genes
࿿1152࿿h Antiparallel strands of DNA molecule mean that
One strand turns clockwise
One strand turns anti-clockwise
The phosphate groups of two DNA strands at their ends, share the same position
The phosphate groups at the start of two DNA strands are in opposite position (pole)
࿿1153࿿h What is the complementary strand for the DNA of the following RNA strand: 5’
GCACGUUUACCGA 3’
3’ AUGCGUUUACCGA 3’
3’ UACGCAAAUGGCU 5’
3’ AGCCAUUUGCGUA 5’
3’ CGTGCAAATGGCT 5’
࿿1154࿿h Which of the following statements about mitochondrial genome is true:
There are 13 tRNA gene in the mitochondrial genome
There are 13 protein coding gene in the mitochondrial genome
There are 22 tRNA gene in the mitochondrial genome
There are 22 protein coding gene products from the mitochondrial genome
࿿1155࿿h Which of the following stages is characterized by the replication of DNA?
Stage before I meiotic division
Stage between two meiotic divisions
Metaphase of mitosis
Prophase of mitosis
࿿1156࿿h DNA exists in a double stranded form whereas RNA is mainly a single stranded
molecule. What is the likely reason for DNA being double stranded?
DNA cannot exist in the single stranded form
RNA strands cannot form base pairs
Double stranded DNA is a more stable structure
It is easier to replicate double stranded DNA than single stranded RNA
࿿1157࿿h Okazaki fragments
Are formed in the lagging strand
Are formed in the leading strand
Remove the primer
Are synthesized by primase
࿿1158࿿h Which of the following processes take part in a nucleus of a cell?
Binding of amino acids with transport RNA
Transcription
Peptide bonds between amino acids
Translation
࿿1159࿿h WHich of the following statements about DNA replication is true?
The leading strand is replicated continuously, while the lagging strand is replicated
discontinuously
The leading strand is replicated discontinuously, while the lagging strand is replicated
continuously
Both the leading and the lagging strands are replicated continuously
Both the leading and the lagging strands are replicated discontinuously
࿿1160࿿h Which statement about base substitutions is true?
May result in nonsense mutation
Can not affect splicing
Are always pathogenic
Result in frameshift mutations
࿿1161࿿h Which statement about transcription is true?
Describes the production of polypeptides from the mRNA template
Occurs in the nucleus
Produces single stranded mRNA using the sense DNA strand as a template
Is regulated bu transcription factors that bind to the 3’ UTR
࿿1162࿿h What is an exon?
It is a regulatory gene
It is a DNA spacer between genes
It is a segment of a split gene that codes for amino acids in a protein
It is a segment of split gene that has no corresponding amino acids in a protein
࿿1163࿿h Which of the following does not regulate normal gene expression?
Promotor
Enhancer
Poly-A tail
Locus control region
࿿1164࿿h RNA processing varies in different cells. This is an example of
Transcriptional control of gene expression
Post-transcriptional control of gene expression
Translational control of gene expression
Post-translational control of gene expression
࿿1165࿿h Which of the following processes take part in the cytoplasm of a cell?
Transcription
RNA processing
Binding of amino acids with transport RNA
RNA splicing
࿿1166࿿h What effect can methylation of CpG islands in the promoter region have on gene
expression?
Transcription level will be reduced
Transcription
level is sharply increased
It will affect translation
It will affect splicing process
࿿1167࿿h Which of the following processes causes diversity of antibodies?
Transcription
Somatic reconstruction
Translation
Post-translational processing
࿿1168࿿h Which of the following does NOT occur as a result of posttranslational modification of a
protein?
Removal of introns
Addition of metals to create tertiary or quaternary structures
Addition of phosphate groups
Removal of N terminus amino acid
࿿1169࿿h Which of the following undergoes processing?
mRNA
DNA
tRNA
rRNA
࿿1170࿿h Which of the following is true about histone proteins?
They are positively charged
They contain methionine and valine
They bind to the base pairs of DNA
There are about 20 histone types associated with DNA
࿿1171࿿h An octamer of 4 histones complexed with DNA forms
Endosome
Mesosome
Nucleosome
Centromere
࿿1172࿿h What is the first stage of chromatin packing?
Formation of a solenoid
Formation of a 300nm fiber
Looping of 300nm fibers
Coiling around nucleosome
࿿1173࿿h Which of the following is not an example of repetitive DNA?
Telomeric DNA
SINE DNA
Centromeric DNA
Promoter region
࿿1174࿿h Which of the following statements is true regarding transposons?
They are sequences of mRNA that can move around in the genome
THey exist in corn, but are not found in human genome
They are the most abundant type of repeat in the genome
All of the above are true regarding transposons
࿿1175࿿h Pseudogenes are
Nonfunctioning genes
Recent beneficial mutations
Randomly transposed genes
Artificial DNA sequences constructed by geneticists and inserted into living organisms
࿿1176࿿h In addition to highly repetitive and unique DNA sequences, a third category of DNA
sequence exists. What is it called? And which rules if elements are involved in it?
Composite DNA; telomeres and heterochromatin
Dominant DNA; euchromatin and heterochromatin
Multiple gene family DNA; hemoglobin and 5.0S RNA
Moderately repetitive DNA; SINEs, LINEs, and VNTRs
࿿1177࿿h In eukaryotes, many genes may have to interact with each other, requiring more
interacting elements than can fit around a single promoter. This physical limitation is
overcome by
Alternating promoters and operators
Placing promoters on both sides of each gene
The use of very long promoters
Distant sites in a chromosome controlling transcription of a gene
࿿1178࿿h When considering the initiation of transcription, one often finds consensus sequences
located in the region of the DNA where RNA polymerase(s) binds. Which of the following is a
common consensus sequence?
TATA
GGTTC
TTTTAAAA
Satellite DNAs
࿿1179࿿h DNA methylation may be a significant mode of genetic regulation in eukaryotes.
Methylation refers to
Altering RNA polymerase activity by methylation
Changes in DNA-DNA in hydrogen binding
Altering translational activity, especially of highly methylated tRNAs
Addition of methyl groups to the cytosine of CG doublets
࿿1180࿿h X chromosome inactivation is a process by which one of the two X chromosomes in a
human females is condensed and inactivated to prevent over expression of x linked genes.
What would most likely be true about the degree of DNA methylation and histone acetylation
on the inactivated chromosome?
Cytokines in CG islands would be hypermethylated and histone proteins would be
deacetylated
Cytosines in CG islands would be hypomethylated and histone proteins would be
acetylated
Cytokines in CG islands would be hyperacetylated and histone proteins would be
hypomethylated
Cytosines in CG islands would be deacetylated and histone proteins would be
hypermethylated
࿿1181࿿h Approximately 5% of the cytosines residues are methylated in the genome of
any given eukaryote. In what way is DNA methylation related to genetic regulation?
There is an inverse relationship between the degree of methylation of DNA and the degree
of gene expression
Methylation of DNA promotes upregulation of gene expression
To the methylated regions of DNA specific transcription factor proteins attached
Active chromatin configuration is characterized by methylation of CpG
࿿1182࿿h The mutations are mainly responsible for
Increasing population size
Maintaining genetic continuity
Constancy in organisms
Variation of organisms
࿿1183࿿h Which of the following is a consequence of mutation?
Prevention of protein formation
Lowering in the amount of a protein
Increases in the amount of a protein
Any of the above can occur
࿿1184࿿h Most known human mutations
Are caused by radiation
Appear to be spontaneous
Are due to viruses
Are germline
࿿1185࿿h Those mutations that arise in the absence of known mutagen are known as
Induced mutations
Fused mutations
Spontaneous mutations
None of the above
࿿1186࿿h Which of the following illustrates a single point mutation on a segment of DNA which
reads
GTGGCACAT?
GTGGGACAU
GTGGCAGAT
TACACGGTG
GTGGGCCACAT
࿿1187࿿h A nucleotide deletion in DNA replication
Causes one amino acid of the protein to be incorrect
Causes all of the amino acids of the protein to be incorrect
Causes the amino acids inserted after the deletion to be incorrect
Causes the amino acids inserted before the deletion to be incorrect
࿿1188࿿h A nonsense mutation introduced into the DNA sequence of a gene may:
Cause premature termination of the mRNA
Shorten the length of the protein encoded by the gene
Have no effect on the transcript or protein made
Cause a shift in reading frame
࿿1189࿿h A single nucleotide substitution (or point mutation) in a DNA sequence can alter the
code in a triplet of bases and cause the replacement of one amino acid by another in the gene
product
Nonsense mutations
Silence mutations
Frameshift mutations
Missense mutations
࿿1190࿿h The mutation, causing sickle-cell disease is GAG → GTG in the beta-globin gene.
This is an example of
A frameshift mutation
Missense mutation
Nonsense mutation
Dynamic mutation
࿿1191࿿h Population carries two or more different alleles at a particular locus. Which of the
following definitions best corresponds to the above mentioned statement?
Homozygote
Heterozygote
Polymorphism
Compound
࿿1192࿿h Substitution of one or the other of two bases at one location is referred to as
Indel
Single nucleotide polymorphism SNP
Simple
short tandem repeat polymorphism STRP
࿿1193࿿h What is the name of the sequence that is most common in a population?
Private sequence
variant sequence
reference sequence
all of the above
࿿1194࿿h when only one member of the pair of relatives is affected and the other is not, the
relatives are
Concordant
Discordant
quantitative traits
qualitative traits
࿿1195࿿h which of the following is true about indels?
they are crucial for identity testing
There are several millions of indels in the genome
They have only 2 alleles??
the DNA sequences of indels are approximately 200 to 500 BP
࿿1196࿿h because of its high level of informativity the Federal Bureau of Investigation uses a set
of 13 of this type of Marker for identity testing
Variable number of tandem repeats VNTR
Single nucleotide polymorphisms SNPs
Copy no. polymorphisms CNPs
All of the above
࿿1197࿿h When should RhIG be injected to the Rh”-” woman?
After miscarriage or medical abortion
after amniocentesis or chorionic villus sampling
after delivery during the first 72 hours
all of the above
࿿1198࿿h VNTRs are
used in DNA fingerprinting
variable the number
tandem repeats
all of the above are correct
࿿1199࿿h Which of the following statements is true of DNA damage?
any DNA damage results in diseases such as cancer
most DNA damage is repaired by the cell
any DNA damage is caused by physical chemical or biological agents
most DNA damage is advantageous to the cell
࿿1200࿿h Which of the following is directly involved in DNA repair
Topoisomerase
Ribosomes
O9 Telomerases
࿿1201࿿h Dna proofreading reparation takes place during which of the following process?
During G1 phase
During DNA synthesis
After DNA replication
A and C
࿿1202࿿h A genotype in which two different mutant allele of the same gene are present, rather
than one normal and one mutant is referred to as
Homozygote
Heterozygote
Hemizygote
Compound heterozygote
࿿1203࿿h According to the X-linked genes males are
Hemizygotes
Homozygotes
Heterozygotes
Compounds
࿿1204࿿h Galactosemia is an autosomal recessivedisorder. Symptoms include Failure to thrive
vomiting jaundice hepatomegaly and cataracts. it is most commonly due to A deficiency of
galactose phosphate uridyltransferase (GALT). A man and woman, who are both known to be
carriers for galactosemia, marry and have a child. To their relief the standard newborn screening
test results which measures the blood level of galactose and galactose-1-p, shows that their child
is affected. Based upon this information, what is theprobability that the child has the disease?
0%
25%
50%
75%
࿿1205࿿h Huntingdon’s disease (HD) is inherited as an autosomal dominant.Patients first exhibit
symptoms as adults (90% b/w the ages 25 and 60, with average in the early 40s). They suffer a
deterioration of the CNS and lose mental abilities and motor control of the limbs. The disease is
fatal within about 10 years after the onset of symptoms. What is the probability two individuals
homozygous for HDwill have a child with HD?
25%
50%
75%
100%
࿿1206࿿h Anautosomal dominanttrait exhibited by some caucasians is called woolly hair.
Mutation in this allele produces extremely brittle hair that breaks off before it grows very
long. What is the probability of twoindividuals heterozygous for this traithaving a normal
child?
0%
25%
50%
75%
࿿1207࿿h Ellis-van Creveld syndrome is a type of dwarfism inherited as an autosomal recessive
disease. It is rare in the general population of the united states, but it is unusually common in the
Amish people of pennsylvania. People with this disease have a normal body length, but have
shortened limbs and appendages as well as extra fingers/toes. Dick suffers from Ellis-van
Creveld syndrome. Margie has a normal stature, but her father suffered from Ellis-van Creveld
syndrome. What is the probability that dick and margie will produce a dwarf child?
0%
25%
50%
100%
࿿1208࿿h Tay-sachs disease shows autosomal recessiveinheritance. Parents of a newly diagnosed
affected child are referred for genetic counselling. It would be correct to tell him that
The probability that their next child will be affected is 1 in 2
The probability that the older unaffected sister of the affected child is a carrier is 1
The fact that their last child was affected means that their next three children will not be
affected
The probability that each parent is a carrier is 1(100%)
࿿1209࿿h What is called a set of alleles at a locus or locus clusters in chromosomes
Wild type
Polymorphism
Homozygote
Haplotype
࿿1210࿿h Which statement about X linked inheritance is true
The condition cannot be passed from an affected father to his son
The condition can be passed from an affected father to his son
When dominant, females are usually as severely affected as males
The risk of germline mosaicism does not need to be considered
࿿1211࿿h The probability that a gene will have any phenotypic expression at all is referred to as
Expressivity
Penetrance
Reduced penetrance
Variable expressivity
࿿1212࿿h The severity of a disease differs in people who have the same genotype. In this
case the phenotype is said to have
Penetrance
Expressivity
Reduced penetrance
Variable expressivity
࿿1213࿿h Which term is used to describe differences in gene expression based on the parent of
origin of an allele?
Parental determination
Sex-determination
Imprinting
Sex-linked dominance
࿿1214࿿h The presence in an individual or a tissue of at least two cell lines that differ
genetically but are derived from a single zygote is referred to as
Mosaicism
Trinucleotide repeat expansions
Imprinting
Mitochondrial inheritance
࿿1215࿿h In familial hypercholesterolemia, individuals homozygous for the allele causing the
disorder completely lack receptors in liver that take up cholesterol from the blood stream.
Heterozygotes have one-half the number of receptors while individuals homozygous for the
normal allele are phenotypically normal. This is an example of
Codominance
Incomplete dominance
Epistasis
Complete dominance
࿿1216࿿h Which of the following statements about normal phenotypic variation is true?
Most normal characteristics in bays are inherited from their fathers and most normal
characteristics in girls are inherited from their mothers
The genes involved are the same in all populations
Environmental factors play a role
Most normal characteristics such as eye or hair color are transmitted as autosomal
recessive trait
࿿1217࿿h Which of the following is characteristic to the expansions of unstable repeat sequences?
Haplosufficiency
Anticipation
Locus heterogeneity
Mosaicism
࿿1218࿿h Which of the following clinical phenotypes is characteristic to the mutations of mtDNA?
Neural and parenchymal disorders
Neuromuscular disorders
Disorders of the sensory organs
Skeletal
࿿1219࿿h If you have an aunt or uncle with a polygenic disorder
You will be more likely to develop the disorder than your father or mother
You are at increased risk of developing the disorder than the average member of your
population
You are not at risk to have children with the disorder
You will eventually develop the disorder
࿿1220࿿h Which of the following statements is true about concordance?
We can talk about concordance only when considering monozygotic twins
Concordance means when any two individuals develop the same disease
Concordance always results from the same genotypes
Concordance may be a result of genetic and/or environmental factors
࿿1221࿿h Which of the following statements about twins and twin studies is FALSE?
Dizygotic twins are important control for any study using monozygotic twins
Twin studies have been used to try determine the role of heredity versus environment.
If the concordance values for monozygotic twins are much higher than for dizygotic
twins, the trait probably has strong genetic basis
Dizygotic twins are as genetically similar as monozygotic twins
࿿1222࿿h Which of the following is TRUEof a concordant trait between twins?
It is more common for the males to have the trait
It is more common for the females to have the trait
It is more common for the older twin to have the trait
Both share the trait
࿿1223࿿h Which of the following is NOT a teratogen (an environmental agent that causes
malformation of an embryo or fetus) for cleft lip/palate?
Anticonvulsants (anti-epilepsy drugs)
Rubella virus
Maternal diabetes
Oral contraceptives
࿿1224࿿h Which of the following is a risk factor for Alzheimer’s disease
Oral contraceptives
Arthritis
Use of anticonvulsants
Head trauma
࿿1225࿿h Which of the following is NOT a symptom of placental artery thrombosis
Premature separation of the placenta from the uterine wall
Intrauterine growth retardation
Increased levels of alpha-fetoprotein (AFP) in amniotic fluid
Preeclampsia
࿿1226࿿h In which of the following conditions cytogenetic analysis is used?
Cataract
Anemia
Congenital anomalies
Diabetes
࿿1227࿿h In some chromosomes the centromere is not in the middle of the chromosome,
however there is no significant difference in the length of the arms. Which of the following
corresponds to this type of chromosome?
Metacentric
Submetacentric
Acrocentric
Telocentric
࿿1228࿿h Which of the following processes, characteristic to the homological chromosomes,
may cause a genetic disease?
Crossing over
Conjugation
Formation of bivalents
Nondisjunction
࿿1229࿿h Which of the following statements about venous thrombosis is true?
Testing factor V leiden and the prothrombin variant will change the management of a
patient with venous thrombosis
Males are five times more likely to be affected with venous thrombosis than females
The factor V leiden mutation causes reduced expression of the factor V gene??
The prothrombin variant causes increased expression of the prothrombin gene

࿿1230࿿h An 80 year old man presented with impairment of higher intellectual function and
alterations in mood and behaviour. His family reported proogressive disorientation and memory
loss over the last 6 months. There is no family history of dementia. The patient was tentatively
diagnosed with alzheimer disease. Which one of the following describes alzheimers disease?
It is caused by the infections beta sheet form of host-cell protein
It is an environmentally produced disease not influenced by the genetics of the individual
It is associated with the accumulation of amyloid precursor protein
It is associated with the deposition of the neurotoxic amyloid beta peptide
aggregates
࿿1231࿿h What type of chromosomal aberration is indicated by the following cytogenetic
abbreviation: i(X)(q10)?
Isochromosome
X monosomy
Transversion
Terminal deletion
࿿1232࿿h In general, full monosomies are not compatible with life. Which of the following
chromosomal monosomies is an exception?
X chromosome monosomy
21 chromosome monosomy
22 chromosome monosomy
17 chromosome monosomy
࿿1233࿿h In which of the following diseases is cytogenetic analysis most frequently recommended?
PKU
Galactosemia
Malignant neoplasia
Diabetes mellitus
࿿1234࿿h Chromosomal aberrations include a change in the sequence of genes on a
chromosome. This is known as
A carrier
An inversion
A duplication
A translocation
࿿1235࿿h You have sent a blood sample of a dysmorphic baby to the lab for the chromosomal
analysis. The answer from the lab says that the baby has 46,XY,del(18)(q12) karyotype. What
type of anomaly is in the karyotype?
The arms of chromosome 18 are equally long
The arms of chromosome 18 are equally short
One arm of chromosome 18 is very long compared to the other
The long arm of chromosome 18 is shorter than it should be
࿿1236࿿h Fluorescence in situ hybridization using the whole chromosome (painting) or
specific locus probes enables routine detection of
Gene expression
Triploidy
Gene amplification
Reciprocal translocation
࿿1237࿿h Which of the following causes the bipotential gonad to differentiate into one of the sexes?
Presence of SRY on the Y chromosome leads to male development
Presence of SRY on the X chromosome leads to female development
Recombined PAR regions of X and Y lead to male development
Formation of female gonads in the absence of y chromosome inhibits mae development
࿿1238࿿h Which of the following statements is incorrect
In most males with karyotype 46,XX the SRY gene is present and found on X
chromosome
X chromosome inactivation, all the gene of one X are switched off
Male fetal development is solely dependent on normal functioning of the SRY gene
Female fetal development is solely dependent on normal functioning of the SRY gene
࿿1239࿿h Which of the following definitions corresponds to the description given as: the
chromosome constitution of individuals
Genotype
Genome
Karyotype
Phenotype
࿿1240࿿h What happens to the extra x chromosome in patients with additional x chromosome
Almost entire chromosome is inactivated
P arm is inactivated
Q arm is inactivated
Region near the centromere is inactivated
࿿1241࿿h Which method is used to detect relative copy number gains and losses in a genome-
wide manner by hybridization?
CGH array
FISH
G-banding
Allele specific oligoneucleotide hybridization
࿿1242࿿h Which of the following statements is correct?
X chromosome inactivation, all genes of one X are switched off
Male fetal development is solely dependent on normal functioning of the SRY gene
Female fetal development is solely dependent on normal functioning of the SRY gene
X chromosome inactivation may be linked to discordance in monozygotic twin pairs
࿿1243࿿h What is a barr body?
Inactive Y
Inactive X
Active X
Active Y
࿿1244࿿h Dosage compensation in mammals typically involves the random inactivation of one of
the two x chromosomes relatively early in development. In such X chromosome inactivation
involve:
FMR1 gene
SOX gene
SRY gene
XIST gene
࿿1245࿿h Marcus and Carmen have a child with ambiguous genitalia. The internal genitalia are
female and chromosome analysis is 46,XX. Biochemical studies reveal a deficiency of 21-
hydroxylase. What is the most likely diagnosis?
Congenital adrenal hyperplasia
Camptomelic dysplasia
Androgen insensitivity
Turner syndrome
Final exam — GENETICS SET 1-3

࿿1246࿿h During telophase all of the above mentioned takes place EXCEPT:

DNA is duplicated;
The chromosomes become less dense and more thread-like;
New nuclear envelopes form;
The chromosomes are released from the microtubules.

࿿1247࿿h In which stage of the meiotic division does crossing over take place?
A.I prophase;B. II prophase; C. II anaphase; D. I anaphase.

3. An important and distinct feature of meiosis is:


A. Chromosome condensation; B.Recombination;C. Chromosome congression; D) spindle formation.

࿿1248࿿h If at the end of meiosis,the daughter cell has 12 chromosomes,how many


chromosomes were in the mother cell?

࿿1249࿿h 3

࿿1250࿿h 6
࿿1251࿿h 12

࿿1252࿿h 24

࿿1253࿿h After which of the following divisions will there be 23 chromosomes and 46
chromatids in the cell? A. MitosisB. I meiotic division;C. II meiotic division; D. Amitosis.

࿿1254࿿h The human haploid genome is composed of approximately:

A. 6 billion base pairs of DNA and 20000 protein coding genes;

B. 6 million base pairs of DNA and 2000 protein coding genes;


C. 3 million base pairs of DNA and 20000 protein coding genes;

D. 3 billion base pairs of DNA and 20000 protein coding genes.


࿿1255࿿h During the process of oogenesis:

A. Four ovum are produced; B. Two ovum and two polar bodies are produced;
C. Two polar bodies are produced; laawata D. Only one ovum is produced

࿿1256࿿h By which of the following bonds are DNA nucleotides of the same strand connected
with each other? A. hydrogen bonds; B. Hydrophobic bonds; C. Sugar-phosphate bonds; D. Peptide
bonds.

࿿1257࿿h Which of the following units corresponds to the 1 thousand nucleotide base
pairs in the genomic DNA?
A. microbase; B. 1 milibase C.1 kilobase;D. 1 megabase.

࿿1258࿿h DNA exists in a double-stranded form whereas RNA is mainly a single stranded
molecule. What is the likely reason for DNA being double stranded?

A. DNA cannot exist in the single stranded form;

B. RNA strands cannot form base pairs;

C. It is easier to replicate double stranded DNA than single stranded RNA;

D. Double stranded DNA is a more stable structure.

࿿1259࿿h What is the complementary strand for the DNA of the following RNA strand: 5‘
AGCCAUUUGCACG 3”?

A. 3‘ CGUGCUUUGGCUS’, B. 3' ATGCGTTTACCGA 3’;


C. 5' CGUGCAAUGGCU 3'; D. 3' TCGGTTTCGTGC S'.
12. Which of the following statements is true regarding Okazaki fragments?
A. They are synthesized by ligase; B. They are short fragments of RNA joined by DNA ligase;
C. They add nucleotides to the elongating DNA. D. They are short fragments of DNA joined by DNA
ligase;

13. When examining the genetic code, it is apparent that:

A. There can be more than one amino acid for a particular codon;
B. AUG is a terminating codon;

C. The code is ambiguous in that the same codon can code for two or more amino acids;
D. There can be more than one codon for a particular amino acid.

14, Which one of the following makes use of RNA template to synthesize DNA?
DNA polymerase; B. RNA polymerase; C. Reverse transcriptase;D.DNA dependent
RNA polymerase.

࿿1260࿿h Which statement about base substitutions is true:

A. Can not affect splicing; B. Are always pathogenic;


C. can affect gene expression; D. Result in frameshift mutations.

࿿1261࿿h Which of the following processes take part in the


nucleus of a cell? A. Post Translational processing; B. RNA processing;

C. Translation; D. Binding of amino acids with transport RNA.


࿿1262࿿h Which statement about
transcription is true: A. Precedes 5' capping and
polyadenylation;

B Produces single-stranded mRNA using the sense DNA strand as a


template; C. Is regulated by transcription factors that bind to the 3' UTR;

D. Describes the production of polypeptides from the mRNA template.

࿿1263࿿h During protein synthesis, an anticodon on


tRNA pairs with: A. DNA nucleotide bases; B. rRNA nucleotide
bases;

C. mRNA nucleotide bases;D. Other tRNA nucleotide


bases,

࿿1264࿿h What effect can methylation of CpG islands in the promoter region have on gene
expression?
Transcription level will be reduce; B. Transcription level is sharply increased;
It will affect translation; D. It will affect splicing process,

࿿1265࿿h Which of the following unprocessed RNA transcripts of a gene colinears with the
genomic DNA and contains introns as well exons?
A.t-RNA; B. r-RNA; C. mature RNA D. primary RNA.

21. Which of the following is NOT an outcome of posttranslational modifications of a protein?

A. Removal of introns; B. Addition of metals to create tertiary or quaternary structures.

C. Addition of phosphate groups; D. Removal of N-terminus amino acid.

࿿1266࿿h During somatic rearrangement which part of the


DNA is excised? A. Exons; B. Introns C. Gene segments;D.
Promoter sequence.
࿿1267࿿h Which of the following types of histones does NOT constitute an octamere?
← H1
← H2a and H2B
← H3
← H4

࿿1268࿿h Which of the following contains protein-encoding genes?


A. Heterochromatin; B. Telomeric regions; C. Euchromatin; D. Centromeric regions.

࿿1269࿿h Which of the following is the fundamental unit of chromatin organization? .


A. Solenoid; B. Scaffold; C. Matrix; D. Histone
࿿1270࿿h Which of the following elements are present in the processed
pseudogenes? A. Anticodons; B. Introns; C, Enhancers;D: Exons.

Approximately 5% of the cytosine residues are methylated in the genome of any given
eukaryote. In what way is DNA methylation related to genetic regulation?

࿿1271࿿h To the methylated regions of DNA specific transcription factor proteins attached:
࿿1272࿿h Methylation of DNA promotes upregulation of gene expression;

࿿1273࿿h There is an inverse relationship between the degree of methylation of DNA and the
degree of gene expression;

࿿1274࿿h Active chromatin configuration is characterized by methylation of CpG.

࿿1275࿿h 2Which of the following does NOT regulate normal gene


expression? A. Enhanser; @ Promoter; C. Silencer; D. ”Cap”,
29. Which is NOT true regarding variable number tandem repeats (VNTRs)?

A. They are about 15 to 100 base pairs long B. They are found within and between genes
C. They are transposable elements D. They are useful for identification of individuals

࿿1276࿿h What are two modular elements that appear as Consensus sequences
upstream from RNA polymerase II transcription start sites?

A. Microsatellites and transposons; B. rDNA and nucleolar organizers;


C. TTAA and CCTT; D. TATA and CAAT.

࿿1277࿿h Transposable sequencesfall into a unique group of repetitive DNA sequences


that are identified by their ability to:
A. Stop viruses from infecting a cell B. Move from place to place within the genome
C. Interfere with telomere function; D. Prevent proper chromosome segregation during meiosis.

࿿1278࿿h A mutation is defined as:


The growth of an abnormal cell Structure;
A change in an organism's DNA sequence;
The changing of a cell from one type to another;
A way of changing mRNA to proteins.

࿿1279࿿h The mechanism of silencing of genes through epigenetics

Specifically at (C) in RNA located in CpG islands;


Specifically on amino acids on histone tails;
Specifically at cytosine (C) in DNA located in areas where strings of C are located;
Specifically at cytosine (C) in DNA located in CpG islands
࿿1280࿿h Which of the following would result in the highest rate of mutation?
Small genes, few hot spots, old age;
Large genes, many hot spots, old age
Small genes, many hot spots, young age;
Large genes, few hot spots, old age.

࿿1281࿿h Why are male mutation rates higher than female?


Greater number of germ cell divisions;
More cells so more DNA;
The Y chromosome is unstable;
Men have only 1 X chromosome.

࿿1282࿿h 3Which of the following illustrates a single point mutation on a segment of


DNA which reads TACACGGTG?
TACACGCGCTG; B. TAGACGGTC; C. TACAUGTG; D. TTCACGGAG.

࿿1283࿿h Which of the following statements regarding spontaneous mutation rates is FALSE?

The spontaneous mutation rate cannot be measured in complex animals such as mammals;
The spontaneous mutation rate varies from gene to gene within an organism;
The spontaneous mutation rate is very high in some organisms, but they have efficient repair
system;
The spontaneous mutation rate varies considerably among different organisms.

࿿1284࿿h What is the consequence of a nonsense mutation?


Introduces a stop codon;
Codes for the wrong amino acid;
Deletes promoter region
Prevents DNA replication.
࿿1285࿿h A frameshift mutation is one of the most severe types of
mutations because: A. More than one gene is affected;

B. They occur only in gametes;


C. Translation is stopped;

D. More than one amino acid or entire proteins are affected.

࿿1286࿿h The mutation, causing sickle-cell disease is GAG —> GTG in the Beta-globin gene.
This is an example of:
Frameshift mutation; B. Nonsense mutation; C. Missense mutation;D. Dynamic mutation.

࿿1287࿿h When the number of bases involved is not a multiple of three (i.e., is not an integral
number of codons), and when it occurs in a coding sequence, the reading frame is altered beginning at
the point of the insertion or deletion:
Nonsense mutations; B. Silence mutations:C. Frameshift mutation;D. Missense mutations.

࿿1288࿿h Which of the following is NOT true about indels?

Indels are never inherited (not sure)


Their DNA sequences may vary from 2 bp to 200 bp
There are hundreds to thousands of indels in the genome
The number of repeats correspond to the number of alleles

࿿1289࿿h What is the name of the sequence that is the most common in a population?
Private sequence; B. Variant sequences C. Reference sequence;D. All of the above.

࿿1290࿿h Which polymorphism is represented with a multiple alleles and used for identity testing:
SNPs B. STRs;C. LINE-1; D. CNVs.
࿿1291࿿h An individual's ABO blood type is normally
determined by: A. Genetic inheritance and environmental
influences during life;

B. Environmental influences alone;

C. Whether mother has been injected by RhIG during pregnancy or not; D.

The inheritance of 1 of 3 possible alleles (A, B, or O) from each parent.

࿿1292࿿h In a case when two related individuals in a family have the same disease, they are said to be:

Concordant
Discordant
Quantitative traits;
D. Qualitative trait.

࿿1293࿿h A polymorphism is:

A. Any change in the DNA sequence,


B, The most common variation of a gene or

C. The least common variation of a gene or marker peers


D. variation of gene or marker sequence present in >1% of the population

࿿1294࿿h Which of the following biomolecule has


self-repair mechanisms?

A.DNA, RNA and


protein; B. DNA and RNA;

C. DNA and proteins;


D. DNA only.
49, An Rh - female a birth to an Rh “+” child. She was not injected Rh immune globulin after the delivery.
Her husband is Rh “+”. Will she have problems with her second pregnancy (remember, Rh*-“ is a
recessive trait)

A. Yes, she will definitely have problems;


B. It depends of the gender of the conceived child

C. No, she will not have problems;


D. Maybe, depending upon the father's genotype.

࿿1295࿿h What type of enzyme removes damaged DNA from the rest of the DNA
molecule? A. .Polymerase;B.Ligase; C. Nuclease; D. Primase.

࿿1296࿿h Which of the following is directly involved in DNA repair:


A. Topoisomerase; B. Ribosomes; DNA polymerases;D. Telomerases.

࿿1297࿿h Tay-Sachs disease is inherited as an autosomal recessive,characterises deficiency of the


enzyme hexosaminidase A. Symptoms include blindness and retardation. Onset of symptoms begins at
about six months of age and death results in early childhood. What is the probability that a
homozygous normal man and a carrier female will have a child with Tay-Sachs disease?

0%;
25%
50%
75%.
࿿1298࿿h Galactosemia is an autosomal recessive disorder. Symptoms include failure to thrive,
vomiting, jaundice hepatomegaly, and cataracts. It is most commonly due to a deficiency of galactose-
phosphate uridyltransferase (GALT). A man and woman, who are both known to be carriers for
galactosemia, marry and have a child. To their relief, the standard newborn screening test results, which
measures the blood level of galactose and galactose-1-P, shows that their child is not affected. Based
upon this information, what is the probability that the child is normal phenotypically?
0%
25%
50%
75%

࿿1299࿿h Red-green colour-blindness also known as daltonism is a relatively benign condition.The


gene responsible for this condition is located on the X chromosome and is inherited as a recessive
trait. Approximately 7-10% of men and 0.5-1% of women are affected. What is the chance that a
daltonic father and a normal mother will produce a color-blind son?

0%
25%
50%
100%

࿿1300࿿h Hemophilia A is a disease in which patients lack a clotting factor in the blood
(factor VIII). It is inherited as an X-linked recessive trait. What is the probability of a haemophilic
maleand a career female producing a hemophilic son?
← 0%
← 25%;
← 50%
← 75%.

࿿1301࿿h Robin is affected by an autosomal dominant disorder inherited from her mother.
She is married to Chad, whos unaffected and has no history of the disorder in his family. Robin and
Chad have two unaffected children. Studies suggest that for every 100 individuals who inherit
mutations in the gene of interest, only 50 actually show symptoms. The new mutation rate for this
disorder is essentially zero. Based on this, what is the probability that their next child will present
with the clinical signs of the disease?

¾
½
¼
1/8
࿿1302࿿h Which of the following definitions best describes a disease, caused by one or two
mutant alleles?
Multifactorial
Monogenic
Chromosomal
Polygenic

࿿1303࿿h Ethnic differences in disease frequencies are most apparent for:

A. Autosomal dominant conditions;

B. X-linked recessive conditions;

C. Autosomal recessive conditions;

D. Sex chromosome aneuploidies.

࿿1304࿿h | Which of the following definitions best corresponds to an individual’s genotype,


which is expressed in its morphological characteristic?

࿿1305࿿h Heterozygote
࿿1306࿿h Homozygote;
࿿1307࿿h Phenotype
࿿1308࿿h Genotype.

࿿1309࿿h The frequency of expression of a phenotype is less than 100%-that is, when some of
those who have the appropriate genotype completely fail to express it the gene is said to show

Penetrance
Expressivity
Variable expressivity
Reduced Penetrance
࿿1310࿿h The presence in an individual or a tissue of at least two cell lines that differ
genetically but are derived from a single zygote is referred to as

Mosaicism
Trinucleotide repeat expansions
Imprinting
Mitochondrial inheritance

࿿1311࿿h Which of the following definitions corresponds to the disease, that is caused by
different types of mutations in the same locus?

← Genetic heterogeneity
← Locus heterogeneity:
← Allelic heterogeneity
← Phenotypic heterogeneity

࿿1312࿿h Nearly every individual affected with neurofibromatosis type 1 (NF1) exhibits clinical
symptoms. Some, however, may present with cafe au lait spots and Lisch nodules, while others have
life- threatening tumors surrounding the spinal cord. These represent an example of which of the
following?

Variable expressivity
Reduced penetrance
Locus heterogeneity
Sex-influenced expression

࿿1313࿿h Which of the following is true about incomplete dominance?

The dominant allele is expressed only in heterozygotes;


The hybrid shows a phenotype that is intermediate between the two homozygotes;
The hybrid expresses the traits carried by both alleles;
Patients homozygous and heterozygous for a mutant allele will have equally severe disease.
࿿1314࿿h In 1991, it was discovered that the fragile X syndrome was caused by a mutation
in the fragile X mental retardation-1 (FMR-1) gene. An area of CGG trinucleotide repeats just
upstream of the coding area was found to be variable in size. All the following statements
regarding the FMR1 gene are true, EXCEPT:

“Premutations” may expand to full mutations in future generations;


Offspring of male carriers inherit a permutation:
Offspring of female carriers may inherit a premutation or a full mutation;
Individuals with premutation will also have intellectual disability.

࿿1315࿿h The expression of the disease phenotype depends on whether the mutant allele
has been inherited from the father or from the mother, a phenomenon known as:

X-Linked Recessive Inheritance


Autosomal recessive
Autosomal dominant
Genomic imprinting

࿿1316࿿h The disease appears to develop at an earlier and earlier age when it is
transmitted through the pedigree, a phenomenon referred to as

Anticipation
X-Linked recessive
Autosomal dominant
Genomic imprinting.

࿿1317࿿h Which of the following is characteristic to the mitochondrial type of inheritance?

The disease is more often in males


The disease is transmitted only maternally
All children of the affected males are also affected
Only females are affected.

࿿1318࿿h Which of the following is TRUE about inheritance of complex diseases?

A. It appears in every second generation of the family;


B. Concordance rate is same in both monozygotic and dizygotic twins;

C. If a person has a predisposition gene, he will develop the disease;


D. The disease is more common is close relatives of the proband than in far relatives.
࿿1319࿿h Which of the following statements is true about polygenic trait?
Several different genes influence one trait
Several different genes influence many traits;
One gene influences many traits;
Several alleles exist for one trait.

࿿1320࿿h Which of the following is true about a modifier gene?

← It is a nonallelic gene that changes the manifestation of another gene


← It is a nonallelic gene that inhibits the function of another gene when certain enzymes
are absent
← It is a nonallelic gene that enhances the function of another gene when certain enzymes
are present
← It is a gene that, according to the demands of the organism, increases or decreases the
function of a nonallelic gene.

࿿1321࿿h Which of the following is TRUE of a concordant trait between twins?

It is more common for the males to have the trait


It is more common for the females to have the trait
It is more common for the older twin to have the trait
Both share the trait

࿿1322࿿h Which of the following diseases are NOT characterized by anticipation?

Huntington disease
Myotonic dystrophy
Fragile X syndrome
Alzheimer’s disease

࿿1323࿿h Which of the following is true about conditions that show a multifactorial inheritance
pattern?

The recurrence risk is lower if more than one family member is affected
If the expression of the disease in the proband is more severe, the recurrence risk is lower
The recurrence risk for the disease is quite high even in remotely related relatives
The recurrence risk is higher if the proband is of the less commonly affected sex.
࿿1324࿿h Which of the following is TRUE about cleft lip/palate (CL/P)?
Syndromic cleft lip/palate is always inherited as a single-gene disorder
It causes mental retardation
Most of the patients with cleft lip/palate are males
In isolated cases of CL/P the recurrence risk in relatives does not always correlate with
proband’s severity.

࿿1325࿿h Which statement is TRUE concerning Alzheimer disease patients?

All people with Alzheimer disease have the £4 allele


All people who have E4 allele will develop the disease
Everyone who carries the APOE gene will develop Alzheimer disease
E4 allele increases the risk of developing Alzheimer disease, but it does not cause the disease.

࿿1326࿿h An 80-year-old man presented with impairment of higher intellectual


function and alterations in mood and following best describes Alzheimer disease?

← It is caused by the infectious B-sheet form of host-cell protein


← It is an environmentally produced disease not influenced; by the genetics of the individual
← It is associated with the accumulation of amyloid precursor protein
← It is associated with the deposition of neurotoxic amyloid beta peptide aggregates

࿿1327࿿h Which of the following statements about venous thrombosis is true

Testing for factor V Leiden and the prothrombin variant will change the management of a
patient with venous thrombosis
Factor V Leiden and the prothrombin G20210A variant are common causes of venous
thrombosis
Male are five time more likely to be affected with venous thrombosis than females
The factor V Leiden mutation causes reduced expression of the factor V gene.

A hari D hari. Deken ekak. Kemathi ekak. :)


࿿1328࿿h Which of the following may be a cause of cerebral vein
thrombosis? A. Mutation of prothrombin and FVL genes at the same time;

B. Mutation of prothrombin gene and use of oral contraceptives;


C. Mutation of FVL gene and use of oral contraceptives;

D. Any of the above.

࿿1329࿿h Which of the following would be most likely to produce an imbalance in the
amount of essential genetic material in the carrier?

Isochromosome
Pericentric inversion
Reciprocal translocation
Robertsonian translocation

81 Which process is primarily responsible for aneuloidy?


← Meiotic nondisjunction
← Unequal crossing over
← Formation of Barr bodies
← Errors during replication

࿿1330࿿h Which of the following types of chromosomes correspond to the dicentric


chromosomes? A. Duplicated; Homological; C. Abnormal;D. Diploid.

࿿1331࿿h Which phase of mitosis can high quality G or R banding of the


chromosomes be accomplished,
Interphase; B. Metaphase;C. Anaphase; -D, Telophase.
࿿1332࿿h To be stable, a rearranged chromosome must have which of
the following? A. No inverted segments

࿿1333࿿h Centromere
࿿1334࿿h Centromere and two telomeres

D. Balanced composition (no losses or gains of genetic material)

DIlina maru! -quizlet

࿿1335࿿h This type of acrocentric chromosome hasattached to their short


arms by stalks, A. Satellites;B, Fragile sites; C. LINE sequences; D. Alu sequences;

࿿1336࿿h Which of the following is a chromosomal mutation?


A. Inversion B. Triploidy; C. Aneuploidy; —_D, Polyploidy.

࿿1337࿿h Which of the following is characterized by the junction of two acrocentric


chromosomes near the centromere with the deletion of short arms?
A. Robertsonian translocation B. Duplication; C. Reciprocal translocation; D. Deletion.

࿿1338࿿h Fluoroscence in situ hybridization using whole chromosome (painting) or


specific locus probes enables routine detection of

← Gene expression
← Gene amplification;
← Trisomy;
← Triploidy.

࿿1339࿿h Short arms of acrocentric chromosomes contains

A. t-RNA genes; B. r-RNA genes;C. Telomerase genes; D. none of above.


࿿1340࿿h You have sent a blood sample of a dismorphic baby to the laboratory for the
chromosomal analysis. The answer from the lab says that the baby 46,XY karyotype. What is the
reason the baby’s parents should also do a blood analysis?

To determine whether anomaly is acquired or inherited;


B. To determine whether anomaly is paternal
To determine whether anomaly is maternal
To determine if there is a family history of the disease.

࿿1341࿿h Which of the following does NOT cause Down syndrome?

Reciprocal translocation between 21q and chromosome 15


Robertsonian translocation between 21q and chromosome 22
Robertsonian translocation between 21 and chromosome 14
Isochromosome 21q21q

࿿1342࿿h Where are pseudoautosomal regions (PAR) located on?

PARI and PAR2 are located close to the centromeres of both X and Y chromosomes;
PAR1 and PAR2 are located on the distal parts of both arms of X and Y chromosomes,...
PAR1 is located at the distal part and PAR2 is located near the centromere of both X
and Y chromosomes
PAR1 is located near the centromere and PAR2 is located at the distal part of both X
and Y chromosomes.

࿿1343࿿h Which method is used to detect relative copy number gains and losses in a
genome-wide manner by Hybridization

← CGH array
← Fluorescent in situ hybridization (FISH)
← G-banding
← Allele specific oligonucleotide hybridization
࿿1344࿿h Which of the following statement is correct
← X chromosome inactivation, all the genes of one X are switched off;
← Male fetal development is solely dependent on normal functioning of the SRY gene;
← Females are mosaic with respect to X-linked gene expression;
← Female fetal development is solely dependent on normal functioning of the SRY gene.

࿿1345࿿h Which of the following is true about X-inactivation?

Embryos with 2 active X can survive, but will be diseased;


Embryos with 2 active X cannot survive;
Embryos with 2 inactive X can survive, but will be diseased;
Embryos with 2 inactive X can survive and will be totally normal

࿿1346࿿h Which is NOT a feature of Kleinfelter syndrome?


A. Long, thin fingers, arms and legs; B. Gynecomastia;C. Short height;D. Small testes.

97. When is Barr body formed?


A. During oogenesis; B. During spermatogenesis; C. After birth D. Early embryogenesis

࿿1347࿿h Which of the following statement is correct:

X chromosome inactivation, all the genes of one X are switched off;


Male fetal development is solely dependent on normal functioning of the SRY gene;
Female fetal development is solely dependent on normal functioning of the SRY gene;
X chromosome inactivation may be linked to discordance in monozygotic twin pairs.

࿿1348࿿h Marcus and Carmen have a child with ambiguous external genitalia. The internal
genitalia are female and chromosome analysis is 46,XX. Biochemical studies reveal a deficiency of 21-
hydroxylase. What is the most likely diagnosis?
A. Camptomelic dysplasia; B. Congenital adrenal hyperplasia;
C. Androgen insensitivity; D. Turner syndrome.
Dosage compensation in mammals typically involves the random inactivation of one of the two X
chromosomes relatively early in development. In a such X chromosome inactivation involve:

࿿1349࿿h FMR1 gene; B.SOX gene; C. SRY gene;D. XIST gene.


࿿1350࿿h

You might also like